You are on page 1of 179

AMPLIACIÓN DE CÁLCULO (Curso 2001/2002) Examen Final de Febrero 31.01.

02

Solución del
PROBLEMA 2
(3 puntos)

Calcular el flujo del campo vectorial F~ (x, y, z) = z 2 cos(x2 + y 2 )~k a través de la semiesfera

x2 + y 2 + z 2 = a 2 , z ≥ 0,

según la normal exterior a la esfera.

Respuesta:
Utilizando coordenadas esféricas, una parametrización de la semiesfera viene dada por las ecua-
ciones:
~ := Σ(θ,
Σ ~ φ) = (x(θ, φ), y(θ, φ), z(θ, φ)),

donde,
x(θ, φ) = a cos θ sen φ, y(θ, φ) = a sen θ sen φ, z(θ, φ) = a cos φ,
con 0 < θ < 2π y 0 < φ < π/2.
Un vector en la dirección de la normal a la semiesfera es entonces
~
∂Σ ~
∂Σ ¡ ¢
~n := × = a2 cos θ (sen φ)2 , a2 (sen φ)2 sen θ, a2 cos φ sen φ
∂φ ∂θ
que, además, tiene el sentido de la normal exterior.
Por otra parte, el valor que toma el campo vectorial sobre la semiesfera es:

F~ (x(θ, φ), y(θ, φ), z(θ, φ)) = (0, 0, a2 (cos φ)2 cos(a2 (sen φ)2 )).

Por lo tanto, el flujo pedido es, por definición:


Z Z 2π Z π/2
Φ := ~ ~
F · ds = dθ a2 (cos φ)2 cos(a2 (sen φ)2 ) a2 cos φ sen φ dφ
Σ 0 0
Z π/2
= 2πa4 (cos φ)3 cos(a2 (sen φ)2 ) sen φ dφ
0

Para resolver esta integral efectuamos el cambio de variable

u = a2 (sen φ)2 =⇒ du = 2a2 cos φ sen φ

del que se obtiene:


Z a2
Φ := π (a2 − u) cos u du = π(1 − cos a2 )
0
que es el resultado pedido.
AMPLIACIÓN DE CÁLCULO (Curso 2001/2002) Examen Final de Junio 24.06.02

Solución del
PROBLEMA 2
(3 puntos)

Una loxodroma sobre un cilindro es aquella curva que corta las generatrices del cilindro formando
con cada una de ellas un mismo ángulo α. Se considera el cilindro de ecuación implı́cita

x2 + y 2 = r 2 .

1) Determı́nense las funciones f para las cuales la curva



x = r cos t 
y = r sent t ∈ R,

z = f (t)

es una loxodroma de ángulo α. ¿Qué nombre reciben estas curvas ?


2) Considérense las loxodromas del apartado anterior que pasan por los puntos (r, 0, 0) y (r, 0, 2π)
y, para cada una de ellas, calcúlese su longitud entre dichos puntos.

Respuesta:
1) El ángulo que forman dos curvas que se cortan en un punto es el menor ángulo α que forman sus
tangentes en ese punto (α ∈ [0, π]).
Las generatrices del cilindro dado son paralelas al eje OZ, por tanto un vector unitario en la
dirección de la tangente a cualquiera de ellas es k = (0, 0, 1). En cuanto a la curva dada, un vector
unitario en la dirección de la tangente es
(x0 (t), y 0 (t), z 0 (t)) (− r sen t, r cos t, f 0 (t))
t= = .
[(x0 (t))2 + (y 0 (t))2 + (z 0 (t))2 ]1/2 [r2 + (f 0 (t))2 ]1/2
El producto escalar entre ambos vectores unitarios da lugar a la expresión:
f 0 (t)
t·k = = cos α . (1)
[r2 + (f 0 (t))2 ]1/2
Por otra parte, su producto vectorial es:
(r sen t, − r cos t, 0)
t×k =
[r2 + (f 0 (t))2 ]1/2
Por tanto el módulo de este producto vectorial da lugar a la relación:
r
|t × k| = = sen α . (2)
[r2 + (f 0 (t))2 ]1/2
Dividiendo miembro a miembro (1) entre (2) resulta:

f 0 (t) = r ctg α .

Puesto que r 6= 0, esta operación siempre puede realizarse ya que, en este caso, se tiene sen α 6= 0.
Integrando miembro a miembro en la última ecuación se obtiene el valor de la función f (t)
buscado, que es:
f (t) = (r ctg α) t + C
siendo C una constante arbitraria.
Ası́ pues, las ecuaciones paramétricas de la curva buscada son:

x = r cos t 
y = r sent t ∈ R,

z = (r ctg α) t + C

que corresponden a una familia de hélices.

2) Se trata de determinar la familia de hélices que pasa por los puntos dados.
Si la hélice pasa por (r, 0, 0), entonces debe existir un t0 ∈ R tal que:

x(t0 ) = r cos t0 = r =⇒ cos t0 = 1 ,


y(t0 ) = r sent0 = 0 =⇒ sen t0 = 0 .

De ambas expresiones se deduce que

t0 = 2nπ , n ∈ Z.

Además,
z(t0 ) = (r ctg α) t0 + C = 0 =⇒ C = − (r ctg α) 2nπ .
Por otra parte, si la hélice pasa por (r, 0, 2π), entonces debe existir un t1 ∈ R (t1 6= t0 ) tal que:

x(t1 ) = r cos t1 = r =⇒ cos t1 = 1 ,


y(t1 ) = r sent1 = 0 =⇒ sen t1 = 0 ,

con lo cual,
t1 = 2mπ , m ∈ Z y m 6= n .
Además,
1
z(t1 ) = (r ctg α) t1 − (r ctg α) 2nπ = 2π =⇒ r ctg α = .
m−n
Es decir, la hélice que pasa por los dos puntos dados es:

x = r cos t 
y = r¡ sent¢ t ∈ R y n, m ∈ Z (n 6= m).
1 2nπ 
z = m−n t − m−n

La longitud de arco pedida viene dada por:


¯ ¯ s 
¯ Z 2mπ s µ ¶2 ¯ µ ¶2
¯ 1 ¯ 1
¯ r2 + dt ¯¯ =  r2 +  |m − n| 2 π .
¯ m − n m − n
¯ 2nπ ¯

Si se considera n = 0 y m = 1 entonces la longitud de arco es r2 + 1 2 π.
AMPLIACIÓN DE CÁLCULO (Curso 2001/2002) Examen Final de Junio 24.06.02

Solución del
PROBLEMA 3
(3 puntos)

Sea U (r) (r = xi + yj + zk) un campo escalar de clase dos y armónico en un dominio M ⊂ R3 . Sea
Ω ⊂ M un subconjunto medible - Jordan compacto, cuya frontera S está formada por un número
finito de superficies regulares.
Se pide:
1) Demostrar la igualdad
Z Z Z Z Z
2
kgradU k dxdydz = U gradUdσ (∗)
Ω S

en donde la superficie S está orientada según la normal saliente.


Indicación: div(V F) = V divF + F · gradV .
2) Comprobar la fórmula anterior (calculando explı́citamente las dos integrales de la igualdad (∗) )
para el campo escalar armónico
1
U (r) =
krk
y el recinto Ω ⊂ R3 limitado por las superficies:

S1 : esfera de centro el origen y radio 1.


S2 : superficie cónica de revolución alrededor del eje z de vértice el origen
y semiángulo en el vértice.
S3 : plano z = 1/2.

Respuesta:
1) Sea F = U gradU que es un campo de clase uno en M . Mediante el uso del teorema de Gauss,
el flujo de F sobre la superficie S puede ser expresado, de la siguiente forma
Z Z Z Z Z Z Z Z
U gradU ds = div(U gradU )dxdydz = (U div(gradU ) + gradU · gradU ) dxdydz =
S Ω Ω
Z Z Z Z Z Z
¡ 2¢
= U ∆U + kgradU k dxdydz = kgradU k2 dxdydz
Ω Ω

donde se ha utilizado que ∆U = 0 en Ω por definición de campo armónico.


2) El conjunto Ω se muestra en la figura.


1.4

1.2
Σ1 1

0.8

0.6

0.4
Σ2 0.2

Σ3 -1 -0.5 0.5 1
RRR ° ° 1 °
°2 RR 1 1
Sean I = Ω°
grad( )
krk °
dxdydz y J = S krk
grad( krk )ds.
p
Sea r := krk = x2 + y 2 + z 2 . En primer lugar,

1 −grad r −r
grad( ) = 2
= 3
r r r
donde se ha utilizado que grad r = − rr . Es inmediato comprobar, aunque no lo pide el enunciado, que
° °2
° 1 °
1
r
es armónico en R − {0}. Ahora, °grad( krk )° = r14 = (x2 +y12 +z2 )2 y krk
3 1 1
grad( krk ) = −r
r4
= (x2−(x,y,z)
+y 2 +z 2 )2
.
A) Cálculo de I. Z Z Z
1
I= dxdydz
Ω (x2 + y 2 + z 2 )2
Por la geometrı́a del recinto y la forma de la función subintegral, lo más adecuado es hacer un cambio
a esféricas.
x = r cos θsenφ ; y = rsenθsenφ ; z = r cos φ
Obviamente, el ángulo θ variará entre 0 y 2π y el ángulo φ entre 0 y π/4. En cuanto a r,
éste variará entre r0 y 1 donde r0 se puede calcular expresando la ecuación del plano z = 1/2 en
coordenadas esféricas
1 1
r0 cos φ = ⇒ r0 =
2 2 cos φ
Por ello,
Z Z Z Z 2π Z π Z 1
1 4 1 2
I = 2 dxdydz = dθ dφ r senφdr =
2 2
Ω (x + y + z )
2
0 0 1
2 cos φ
r4
Z π Z 1 Z π · ¸1
4 1 4 1
= 2π dφsenφ 2
dr = 2π dφsenφ − =
0 1
2 cos φ
r 0 r 1
2 cos φ
Z π Z π Z π
4 4 4
= 2π senφ (2 cos φ − 1) dφ = 4π senφ cos φdφ − 2π senφdφ =
0 0 0
1 ³√ ´
π/4
= 2π sen2 φ |0 −2π cos φ |0π/4 = π 2−1
2
B) Cálculo de J. La integral de superficie se calcula descomponiéndola en las integrales corres-
pondientes a Σ1 , Σ2 y Σ3 (ver figura).

Z Z Z Z µZ Z Z Z Z Z ¶
r r r r r
J =− ds = − · nds = − · nds+ · nds+ · nds
S r4 S r4 Σ1 r4 Σ2 r4 Σ3 r4

donde n es el vector unitario normal saliente a la superficie.


B.1. Como en la superficie esférica se tiene n = r y r = 1 entonces
Z Z Z Z Z Z Z Z ° °
r 1 1 ° ∂Φ ∂Φ °
· nds= ds = 2 ds = Área(Σ1 ) = ° °
r 4 r 2 1 ° ∂φ × ∂θ ° dφdθ
Σ1 Σ1 Σ1 G

donde (x, y, z) = Φ(u, v), (u, v) ∈ G es una parametrización de Σ1 . Como parametrización de Σ1 se


puede tomar

x = cos θsenφ ; y = senθsenφ ; z = cos φ


h πi
θ ∈ [0, 2π] ; φ ∈ 0,
4
° °
° ∂Φ °
para la cual ° ∂Φ
∂φ
× ∂θ °
= senφ luego
Z Z Z 2π Z π/4 Z π/4 √
r
· nds = dθ senφdφ = 2π senφdφ = π(2 − 2)
Σ1 r4 0 0 0

r
B.2. En Σ2 los vectores normales a la superficie son perpendiculares al campo r4
(nótese que es
un campo central) con lo que Z Z
r
4
· nds = 0
Σ2 r

B.3. En Σ3 , n =(0, 0, −1) luego


Z Z Z Z
r z
· nds = − ds
Σ3 r4 Σ2 (x2 + y 2 + z 2 )2

y ahora, parametrizando Σ3 en la forma


1 1
x=x; y=y; z= ; x2 + y 2 ≤
2 4
° °
° ∂Φ °
se tiene, al ser ° ∂Φ
∂φ
× ∂θ °
= 1,
Z Z Z Z Z Z 1
r z 2
· nds = − ds = − ¢2 dxdy =
¡
Σ3 r4 Σ2 (x + y 2 + z 2 )2
2
x2 + y 2 + 14
x2 +y 2 ≤ 14
Z Z Z Z 1/2
1 1 1 2π ρ
= − ¡ ¢ 2 dxdy = − dθ ¡ ¢2 dρ =
2 x2 +y 2 ≤ 14 x2 + y 2 + 14 2 0 0 ρ2 + 14
Z 1/2
ρ
= − ¡ ¢ dρ = −π
1 2
0 ρ2 + 4

Por ello, se tiene finalmente


³ √ ´ √
J = − π(2 − 2) + 0 − π = π( 2 − 1)

con lo que se compueba que, efectivamente, I = J.


AMPLIACIÓN DE CÁLCULO (Curso 2001/2002) Examen Final de Septiembre 9.9.02

Solución del
PROBLEMA 1
(4 puntos)

Sea f : (0, ∞) → R una función de clase uno, sea a ∈R3 un vector fijo no nulo y sea el campo
F :R3 \ {0} → R3 definido por
F(r) =f (r) a × r
p
donde r = xi + yj + zk y r = x2 + y 2 + z 2 .
Se pide:
a) Calcular la divergencia del campo F y simplificar el resultado.
b) Sea Γ la curva definida por las ecuaciones

x2 + y 2 + z 2 = 1
x−z =0

Calcular Z
a×r
I= dr
Γ r2
donde la orientación de Γ es tal que su proyección sobre el plano xy está orientada en sentido contrario
a las agujas del reloj.
c) Se considera el tetraedro T de vértices A ≡ (0, 0, 1/2), B ≡ (0, 1, 0), C ≡ (1, 0, 0) y V ≡ (1, 1, 1).
Calcular la integral Z
J= r2 (a × r) dσ
Σ

siendo a = (1, −1, 0) y Σ la superficie formada por las tres caras de T que pasan por el punto V ,
orientada según la normal saliente a T .

Respuesta:

a) En primer lugar calculamos

a × r = (a2 z − a3 y, a3 x − a1 z, a1 y − a2 x)

y ahora, utilizando que si U es un campo escalar y V un campo vectorial se cumple div (U V) = grad
U · V + U div V, se tiene

div (f (r)a × r) = grad (f (r)) · (a × r) + f (r) div (a × r) =


= f 0 (r) gradr · (a × r) + f (r) div (a × r) =
r
= f 0 (r) · (a × r) + f (r) div (a × r) = 0
r
pues:
- gradr = rr
- div (a × r) = 0 al ser su componente i-ésima independiente de la i-ésima variable, i = 1, 2, 3.
- el producto mixto r · (a × r) es nulo al ser los vectores linealmente dependientes.
Por ello, para cualquier f el campo F es solenoidal en su dominio de definición.
b) Se puede proceder de dos formas:
b.1) Calculando drectamente la integral curvilı́nea. Para parametrizar Γ calculamos su
proyección sobre el plano xy eliminando la z entre las dos ecuaciones que definen la curva. Esta
proyección es 2x2 + y 2 = 1 y entonces la curva se puede escribir

2x2 + y 2 = 1
z = x

Parametrizamos la proyección en la forma


1
x = √ cos t ; y = sen t , t ∈ [0, 2π]
2
(nótese que lleva la orientación que indica el enunciado), y entonces una posible parametrización
para Γ viene dada por
1
x = √ cos t
2
y = sen t
1
z = √ cos t con t ∈ [0, 2π]
2
Ahora, como r2 = x2 + y 2 + z 2 vale uno en todos los puntos de Γ, la expresión de I se puede
simplificar. En efecto,
Z Z Z
a×r
I= 2
dr = (a × r) · dr = (a2 z − a3 y)dx + (a3 x − a1 z)dy + (a1 y − a2 x) dz
Γ r Γ Γ

y entrando con la parametrión anterior se tiene


Z 2π µ µ ¶ ¶µ ¶
a2 cos t sen t a3 cos t a1 cos t a2 cos t sen t
I= √ − a3 sen t) − √ +( √ − √ ) cos t + (a1 sen t − √ − √ dt
0 2 2 2 2 2 2
R 2π R 2π R 2π R π/2
Por simetrı́a, la integral 0 cos tsentdt vale cero mientras que 0 cos2 dt = 0 sen2 tdt = 4 0 cos2 dt
por lo que se puede escribir
µ ¶ Z π/2
2a3 2a1 √
I=4 √ − √ cos2 dt = 2π (a3 − a1 )
2 2 0

b.2) Aplicando el teorema de Stokes. Como rot(a × r) = 2a tiene una expresión muy simple
y la curva Γ es la frontera de la superficie (también muy sencilla) S correspondiente a la porción del
plano x = z contenido en el interior de la esfera x2 + y 2 + z 2 = 1 parece razonable aplicar el teorema
de Stokes para calcular I. Ası́,
Z Z Z
a×r
I= 2
dr = (a × r) · dr = 2a · dσ
Γ r Γ S

donde S está orientada por el vector normal unitario n = √12 (−1, 0, 1). De esta forma
Z Z √ Z √
1
I = 2 a · dσ =2 a · √ (−1, 0, 1) dσ = 2(a3 − a1 ) dσ = 2(a3 − a1 )Área(S)
S S 2 S

y como S es un cı́rculo de radio 1 se tiene Área(S) = π y se llega al resultado final I = 2π (a3 − a1 ).
c) El calcular J integrando sobre Σ parece complicado debido a la complejidad de dicha superficie,
por lo que se busca otra alternativa para su cálculo. Puesto que el campo r2 (a × r) tiene la forma
del primer apartado, es solenoidal en R3 . De esta forma, y mediante el uso del teorema de Gauss, J
es igual a la integral del campo en cuestión sobre cualquier superficie que se apoye sobre el borde de
Σ y esté orientada coherentemente con dicho borde. Por ejemplo, podemos elegir por ejemplo la cara
S del tetraedro T correspondiente a los vértices A, B y C. Esta cara está situada sobre el plano

x + y + 2z = 1

y se debe considerar la orientación correspondiente al vector normal unitario n = √16 (1, 1, 2).
Entonces
Z Z
J = r (a × r)dσ = r2 (a × r) · ndσ =
2
Σ Z Σ
1 ¡ 2 ¢
= √ x + y 2 + z 2 ((1, −1, 0)×(x, y, z)) · (1, 1, 2)dσ =
6 ZΣ
2 ¡ 2 ¢
= √ x + y 2 + z 2 (x + y − z)dσ
6 Σ

Ahora parametrizamos S en la forma estandar, es decir,

x=x
Φ: y=y ; (x, y) ∈ M = {(x, y) : 0 ≤ x ≤ 1 ; y = 1 − x}
1
z = h(x, y) = 2 (1 − x − y)

Entonces
° ° s µ ¶2 µ ¶2 √
° ∂Φ ∂Φ ° ∂h ∂h 6
° °
° ∂x × ∂y ° = 1 + ∂x + ∂y =
2
y se tiene
Z Z µ ¶ √
2 1 1 6
J = √ x2 + y 2 + (1 − x − y)2 (x + y − (1 − x − y) ) dxdy =
6 M 4 2 2
Z Z µ ¶
2 2 1 2 1
= x + y + (1 − x − y) (x + y − (1 − x − y) )dxdy = · · · =
4 2
Z ZM µ ¶
15 3 15 3 21 2 21 2 11 2 11 2 7 5 5 1
= x + y + xy + x y − x − y − xy + x + y − dxdy
M 8 8 8 8 8 8 4 8 8 8
RR
Ahora hay que evaluar 10 integrales del tipo M
xpRy qRdx. En primerR lugar
R podemos utilizar que,
p q q p
al ser M simétrico respecto de la recta y = x, entonces M
x y dx = M
x y dx (*) con lo que
Z Z µ ¶
15 3 21 2 11 7 5 1
J= x + x y − x2 − xy + x − dxdy
M 4 4 4 4 4 8

y hay que calcular 6 integrales. Estas se pueden calcular de forma conjunta evaluando, para p y q
genéricos, la integral
Z Z Z 1 Z 1−x Z 1 Z 1
p q p q 1 p q+1 1
x y dxdy = x dx y dy = x (1 − x) dx = xp (1 − x)q+1 dx =
M 0 0 0 q + 1 q + 1 0
1 1 Γ(p + 1)Γ(q + 2) 1 p!(q + 1)! p!q!
= β(p + 1, q + 2) = = =
q+1 q + 1 Γ(p + q + 3) q + 1 (p + q + 2)! (p + q + 2)!

Nótese que esta fórmula demuestra la la validez de la igualdad (*) que se estableció anteriormente
por un argumento de simetrı́a.
Ası́,
Z Z Z Z
3 3! 1 2 1
x dxdy = = ; x2 ydxdy = =
5! 20 5! 60
Z ZM Z ZM
2! 1 1 1
x2 dxdy = = ; xydxdy = =
4! 12 4! 24
Z ZM Z Z M
1 1 1
xdxdy = = ; dxdy =
M 3! 6 M 2
y entonces
15 1 21 1 11 1 7 1 51 11
J= + − − + −
4 20 4 60 4 12 4 24 4 6 8 2
19
y operando se llega al resultado final J = 160 .
AMPLIACIÓN DE CÁLCULO (Curso 2002/2003) Examen Final de Junio 23.06.03

Solución del
PROBLEMA 1
(4 puntos)

Sean φ : R3 → R un campo escalar de clase C 2 y Ω ⊂ R3 un sólido limitado por una superficie S


regular y orientada según la normal saliente n.
Se pide:
1) Calcular div (φ grad φ).
2) Aplicando el teorema de Gauss, transformar la integral de superficie
ZZ
∂φ
φ dσ
S ∂n

∂φ
en una integral triple extendida a Ω. (Se recuerda que = grad φ · n).
∂n
3) Determinar la expresión en Ω de todos los campos vectoriales F : R3 → R3 , F ∈ C 1 , que sean
irrotacionales, solenoidales y con componente normal sobre S igual a cero en todos los puntos.
4) Sea S una superficie regular orientable que encierra un volumen acotado Ω ⊂ R3 ¿Queda unı́vo-
camente determinado en Ω un campo vectorial G : R3 → R3 , G ∈ C 1 , si se conocen rot G y div G
en R3 y la componente normal de G sobre todos los puntos de S? Razonar la respuesta.

Respuesta:
1) Utilizando las definiciones de gradiente de un campo escalar y de divergencia de un campo vecto-
rial, se tiene:
· ¸ · ¸ · ¸
∂ ∂φ ∂ ∂φ ∂ ∂φ
div (φ grad φ) = φ + φ + φ
∂x ∂x ∂y ∂y ∂z ∂z
µ ¶µ ¶ 2
µ ¶µ ¶ µ ¶µ ¶
∂φ ∂φ ∂ φ ∂φ ∂φ ∂ 2φ ∂φ ∂φ ∂2φ
= +φ 2 + +φ 2 + +φ 2
∂x ∂x ∂x ∂y ∂y ∂y ∂z ∂z ∂z
µ ¶2 µ ¶2 µ ¶2 · 2 2 2
¸
∂φ ∂φ ∂φ ∂ φ ∂ φ ∂ φ
= + + +φ + +
∂x ∂y ∂z ∂x2 ∂y 2 ∂z 2

= kgrad φk2 + φ ∆ φ ,

donde k.k representa la norma dos y ∆ es el operador laplaciano.


2) En las condiciones establecidas en el enunciado es posible aplicar el teorema de Gauss a la integral
de superficie dada. Para ello basta tener en cuenta que
ZZ ZZ
∂φ
φ dσ = φ grad φ · n dσ .
S ∂n S

La aplicación del teorema de Gauss a esta integral da lugar a la igualdad


ZZ ZZZ
∂φ
φ dσ = div [φ grad φ] dV .
S ∂n Ω
siendo Ω el volumen limitado por la superficie cerrada S. Finalmente, teniendo en cuenta el resultado
obtenido en el apartado anterior, se obtiene la igualdad:
ZZ ZZZ ZZZ
∂φ 2
φ dσ = kgrad φk dV + φ ∆ φ dV .
S ∂n Ω Ω

3) Que el campo F sea irrotacional en R3 (que es un dominio estrellado) significa que es el gradiente
de un potencial escalar. Es decir, existe un campo escalar φ : R3 → R tal que
F = grad φ (en R3 ) .
Además, F es solenoidal (es decir, tiene divergencia nula), por lo tanto:
div (F) = div (grad φ) = ∆ φ = 0 (en R3 ) ,
donde ∆ vuelve a representar el operador laplaciano.
Podemos concluir, por tanto, que todo campo irrotacional y solenoidal en R3 es el gradiente de
un campo escalar armónico (que satisface la ecuación de Laplace ∆ φ = 0 en R3 ).
Por otra parte, puesto que F = grad φ, su componente normal sobre la superficie cerrada S viene
dada por grad φ · n = (∂φ/∂n), siendo n un vector dirigido según la normal saliente a S en cada
punto. Ahora bien, esta componente normal es cero y, en consecuencia, la integral de superficie
ZZ ZZ
∂φ
φ F · n dσ = φ dσ
S S ∂n
debe también anularse.
El resultado final obtenido en el apartado 2) anterior nos permite entonces concluir que
ZZ ZZZ ZZZ
∂φ 2
φ dσ = kgrad φk dV + φ ∆ φ dV = 0 .
S ∂n Ω Ω

De aquı́, teniendo en cuenta que (como se acaba de demostrar) φ es un campo escalar armónico
(∆φ = 0), resulta ZZZ
kgrad φk2 dV = 0

y como kgrad φk2 es una función continua no negativa, obtenemos finalmente
kgrad φk = 0 en Ω ⇐⇒ F = grad φ = 0 en Ω .
Ası́ pues, el único campo vectorial que satisface todas las condiciones establecidas en este tercer
apartado es el campo nulo en Ω, que deriva de un potencial escalar constante.

4) Se trata de determinar si en el volumen Ω limitado por la superficie cerrada S, el conocimiento del


rotacional y la divergencia de un campo vectorial G, junto con los valores de su componente normal
sobre S, determina dicho campo de forma única.
Para ello supongamos que existe otro campo vectorial H tal que su rotacional, su divergencia y
los valores de su componente normal sobre S, coinciden con los de G.
En estas condiciones, la diferencia entre ambos campos vectoriales, F = G − H, es a su vez un
campo vectorial con las siguientes propiedades:
rotF = rotG − rotH = 0, divF = divG − divH = 0 .
Además, la componente normal de F sobre S es también nula.
Según se ha aprobado en el apartado anterior, el único campo en Ω que cumple estas condiciones
es el idénticamente nulo. Es decir , G y H tienen que ser iguales, de lo que concluimos que, en efecto,
las condiciones dadas sobre G en este apartado 4) lo determinan unı́vocamente en Ω.
AMPLIACIÓN DE CÁLCULO (Curso 2002/2003) Examen Final de Junio 23.06.03

Solución del
PROBLEMA 2
(3 puntos)

Se considera el astroide de ecuación cartesiana:


x2/3 + y 2/3 = a2/3 , (a > 0).
Se pide:
1) Calcular su longitud.
2) Calcular el área que encierra.

Respuesta:
1) La curva Γ en cuestión es cerrada y presenta simetrı́a respecto de los dos
ejes de coordenadas. Su aspecto es el que muestra la figura.
Para calcular la longitud de la curva se debe encontrar una parametrización
de la misma. Para ello, se puede utilizar el hecho de que sen2 t + cos2 t = 1,
obteniendo ası́ la parametrización r(t) = (x(t), y(t)) dada por

x = acos3 t
y = asen3 t, t ∈ [0, 2π]

Se tiene entonces
Z 2π Z 2π p Z 2π √
0
L= kr (t)k dt = x0 (t)2 + y 0 (t)2 dt = 3a sen2 t cos4 t + sen4 t cos2 tdt =
0 0 0
Z 2π p Z 2π √
= 3a 2 2 2 2
sen t cos t(sen t + cos t)dt = 3a sen2 t cos2 tdt =
0 0
Z 2π Z π π
2
= 3a |sent| |cos t| dt = 12a sent cos tdt = 6asen2 t |02 = 6a
0 0

donde en la antepenúltima igualdad se ha√utilizado la periodicidad de la función |sent| |cos t| (nótese


que si se comete el error de escribir que sen2 t cos2 t es igual a sent cos t, se obtiene que la longitud
es nula, lo cual, evidentemente, no tiene sentido).
2) La parametrización anterior puede utilizarse para el cálculo del área mediante una integración
curvilı́nea haciendo uso de la siguiente expresión
Z Z
1 1 2π
A= xdy − ydx = [x(t)y 0 (t) − y(t)x0 (t)] dt =
2 Γ 2 0
Z Z
1 2π 0 0 3 2 2π £ 2 ¤
= [x(t)y (t) − y(t)x (t)] dt = a sen t cos4 t + sen4 t cos2 t dt =
2 0 2 0
Z 2π Z π
3 2 1 3 3 3
= a2 sen2 t cos2 tdt = 6a2 sen2 t cos2 tdt = 6a2 β( , ) = · · · = πa2
2 0 0 2 2 2 8
Nótese que a debe tener dimensiones de longitud para que x e y representen una posición. Se
comprueba que los resultados obtenidos son dimensionalmente correctos, puesto que L y A tienen
dimensiones de longitud y área respectivamente.
AMPLIACIÓN DE CÁLCULO (Curso 2002/2003) Examen Final de Septiembre 8.09.03

PROBLEMA 1 (4 puntos) Contestar razonadamente a cada una de las siguientes cuestiones: (las
cuestiones 1, 2 y 3 no son actual del programa)
4) Sea Ω = R2 \ {(±1, 0)} y sean P, Q : Ω → R dos funciones de clase C 1 tales que

∂P ∂Q
(x, y) = (x, y) , (x, y) ∈ Ω
∂y ∂x
y Z Z
P dx + Qdy = 4 , P dx + Qdy = 7 ,
Γ1 Γ2

en donde Γ1 es el cuadrado de vértices (±3, ±3) con orientación positiva y Γ2 es la circunferencia


(x − 1)2 + y 2 = 1 con
Z orientación positiva.
Se pide calcular P dx+Qdy siendo Γ cada una de las siguientes curvas, con orientación positiva:
Γ

a) Circunferencia (x + 1)2 + y 2 = 1.

b) Elipse 4x2 + y 2 = 1.

c) Cuadrado de vértices (0, ±1), (2, ±1).

Respuesta:

Aplicaremos el siguiente resultado (consecuencia del teorema de Green):

Sea F = (P, Q) un campo vectorial de clase 1 en un dominio Ω (múltiplemente conexo) de R2 y


∂Q ∂P
tal que = . Sea R una región cerrada en Ω cuya frontera está formada por un número finito
∂x ∂y
de curvas de Jordan Γ, γ1 , . . . , γm de forma que Γ contiene a γ1 , . . . , γm en su interior. Entonces
Z m Z
X
P dx + Qdy = P dx + Qdy .
Γ k=1 γk

recorridas las curvas con la misma orientación.


Llamemos C a la circunferencia del apartado a), E a la elipse de
4
b) y G al cuadrado de c) (véase la figura); apliquemos el resultado
Γ1
enunciado: 3

a) Consideramos la región cuya frontera exterior es el cuadrado 2

Γ1 y cuyas fronteras interiores son las circunferencias Γ2 y C, E


1
se tiene:
Z Z Z Z -4 -3 -2 -1 1 2 3 4
Γ2
= + ⇒ P dx + Qdy = 4 − 7 = −3 . -1
Γ1 Γ2 C C C G
-2

b) En la región
Z encerrada por la elipse el campo es conservativo, -3
entonces P dx + Qdy = 0.
E -4

c) Consideramos la región cuya frontera exterior es G y cuya


frontera interior es Γ2 , entonces
Z Z
P dx + Qdy = P dx + Qdy = 7 .
G Γ2
AMPLIACIÓN DE CÁLCULO (Curso 2002/2003) Examen Final de Septiembre 8.09.03

Solución del
PROBLEMA 2
(3 puntos)

Se considera un elipsoide en el que las longitudes de los semiejes son a, b y c. A cada punto de la
superficie del elipsoide se le asocia la distancia, d(x, y, z), del centro del elipsoide al plano tangente al
elipsoide en ese punto. Calcúlese la integral de la función 1/d2 (x, y, z) sobre la superficie del elipsoide.

Respuesta: Puesto que la solución del problema no depende del sistema de referencia elegido,
tomaremos aquel en el que el centro del elipsoide está en el origen y sus ejes coinciden con los ejes
cartesianos. Ası́, la ecuación del elipsoide es

x2 y 2 z 2
+ 2 + 2 =1
a2 b c
En primer lugar calcularemos el plano tangente al elipsoide en un punto genérico (x, y, z) de su
x2 y2 z2
superficie. Puesto que la ecuación del elipsoide es F (x, y, z) = ¡0 con F (x, ¢ y, z) = a 2 + b2 + c2 − 1,

un
¡ x vector ¢ normal al plano viene dado por gradF (x, y, x) = 2x , 2y , 2z , o dividiendo por 2, por
a2 b2 c2
, y , z . De esta forma la ecuación de los puntos (X, Y, Z) del plano es
a2 b 2 c2

x y z
2
(X − x) + 2 (Y − y) + 2 (Z − z) = 0
a b c
es decir,
x y z x2 y 2 z 2
X + Y + Z = + 2 + 2
a2 b2 c2 a2 b c
y teniendo en cuenta que (x, y, z) pertenece al elipsoide, se tiene finalmente
x y z
2
X + 2Y + 2Z = 1
a b c
La distancia de un punto (x0 , y0 , z0 ) a un plano AX + BY + CZ + D = 0 es

|Ax0 + By0 + Cz0 + D|


d= √
A2 + B 2 + C 2
por lo que al ser (x0 , y0 , z0 ) = (0, 0, 0), la función distancia pedida viene dada por

|−1| 1
d(x, y, z) = √ =q .
A2 2
+B +C 2 x 2 2
+ yb4 + z2
a4 c4

De esta forma, la integral pedida es


Z Z µ 2 ¶
1 x y2 z2
I= 2
dσ = + 4 + 4 dσ
S d (x, y, z) S a4 b c
Utilizando la paridad de la función subintegral respecto de x, y y z y la simetrı́a del recinto
respecto de los tres planos coordenados se tiene
Z µ 2 ¶
x y2 z2
I=8 + 4 + 4 dσ
S0 a4 b c

donde S 0 es la porción de elipsoide comprendida en el primer octante.


Procederemos parametrizando S 0 en la forma

(x, y, z) = Φ(θ, φ), (θ, φ) ∈ Ω

y entonces la integral se calcula como una integral doble en la forma


Z Z ° °
1 ° ∂Φ ∂Φ °
I=8 ° (θ, φ) × (θ, φ)° dθdφ
2 ° °
Ω (d(Φ(θ, φ))) ∂θ ∂φ

La parametrización más cómoda para S 0 es

x = a cos θsenφ
y = bsenθsenφ
π h πi
z = c cos φ, θ ∈ [0, ], φ ∈ 0,
2 2
Operando se obtiene
° °
° ∂Φ ° p
° (θ, φ) × ∂Φ (θ, φ)° = b2 c2 cos2 θsen4 φ + a2 c2 sen2 θsen4 φ + a2 b2 cos2 φsen2 φ =
° ∂θ ∂φ °
r
cos2 θsen2 φ sen2 θsen2 φ cos2 φ
= abc |senφ| + +
a2 b2 c2
Por otro lado,
x2 y 2 z 2 cos2 θsen2 φ sen2 θsen2 φ cos2 φ
+ 4 + 4 = + +
a4 b c a2 b2 c2
y por ello,
Z µ ¶ Z π Z πµ 2 ¶3
x2 y 2 z 2 2 2 cos θsen2 φ sen2 θsen2 φ cos2 φ 2
I = 8 + 4 + 4 dσ = 8abc dθ + + |senφ| dφ =
S0 a4 b c 0 0 a2 b2 c2
Z π Z πµ 2 ¶3
2 2 cos θsen2 φ sen2 θsen2 φ cos2 φ 2
= 8abc dθ + + senφdφ
0 0 a2 b2 c2
£ ¤
donde se ha usado que el seno es no negativo en 0, π2 . Las integrales obtenidas no pueden resolverse
por los métodos habituales de integración, por lo que el resultado se deja indicado.
AMPLIACIÓN DE CÁLCULO (Curso 2003/2004) Examen Final de Febrero 5.02.04

Solución del
PROBLEMA 1
(4 puntos)

1) Sea Ω un dominio acotado de R3 limitado por una superficie regular Σ orientada según la
normal saliente. Sea F :R3 → R3 un campo vectorial regular del que se sabe que admite tanto un
potencial escalar como unR potencial
R vector y sea G un potencial vector cualquiera de F. Encontrar
una expresión para I = Σ
F × G dσ que dependa únicamente del valor del módulo de F en el
recinto Ω.
2) Calcular las constantes α, β,γ y δ para que el campo

F(r) =(x + αz)i+(βx + y)j+(γy + δz)k, r =xi+yj+zk

definido en R3 , admita tanto un potencial escalar como un potencial vector. Para los valores de α, β,γ
y δ obtenidos, calcular las constantes A, B, C, D y E para que el campo

G(r) =(yz + Azx, Bxz + Cxy, Dxy + Eyz)

sea un potencial vector de F.


3) Comprobar la validez de la fórmula obtenida en el apartado 1 en el caso en que Σ es la esfera
unidad orientada según la normal saliente y F y G son los campos calculados en el apartado 2.
4) Sea Γ la curva de ecuaciones cartesianas
¾
x2 + y 2 − 1 = 0
x+y+z−1 = 0

orientada de forma que su proyección sobre el plano xy se recorre en sentido positivo. Se pide:
4.1. Parametrizar Γ y calcular la circulación del campo G hallado en el apartado 2 sobre Γ.
4.2. Comprobar el resultado aplicando de forma adecuada el teorema de Stokes.

Respuesta: Se entregará esta hoja y, a lo sumo, una adicional.

1. Puesto que se pide relacionar una integral de superficie de un campo vectorial con una integral
triple parece razonable aplicar el teorema de Gauss. La regularidad de la superficie Σ y de los campos
F y G involucrados hacen que se verifiquen las condiciones suficientes para su aplicabilidad. Ası́,
Z Z Z Z
F × Gdσ = div(F × G)dv,
Σ Ω

Ahora se utilizará que div(F × G) = G·rotF − F·rotG, que al admitir F un potencial escalar en
R se sigue rotF = 0 en R3 y, por definición de potencial vector, rotG = F obteniéndose
3

Z Z Z Z Z Z Z
F × Gdσ = −F · Fdv = − kFk2 dv
Σ Ω Ω

que es el resultado pedido.


2. Para que F admita un potencial escalar en R3 , es decir, para que F sea un campo conservativo
en R3 , es condición necesaria que se cumpla rotF = 0 en R3 mientras que para que F admita un
potencial vector es condición necesaria que se cumpla divF = 0 en R3 (y ello es independiente
del hecho de que R3 sea un dominio estrellado). Imponiendo las condiciones anteriores se tiene
rotF = (γ, α, β) = (0, 0, 0) de donde se sigue α = β = γ = 0 y divF = 1 + 1 + δ = 0 de donde
se sigue δ = −2, es decir F = xi + yj − 2zk. Ahora bien, como R3 es estrellado las condiciones
necesarias anteriores también son suficientes y por ello se puede afirmar que el campo F anterior
verdaderamente admite un potencial escalar y un potencial vector.
Forzando a que G sea un potencial vector de F se tiene

∀(x, y, z) ∈ R3 , rotG = (Dx + Ez − Bx, y + Ax − Dy, Bz + Cy − z) = (x, y, −2z)

Como dicha igualdad es válida para todo (x, y, z) ∈ R3 , G es un potencial vector de F si y sólo si se
verifican las igualdades

D − B = 1, E = 0, 1 − D = 1
A = 0, B − 1 = −2, C = 0

que se cumplen si y sólo si


A = C = D = E = 0, B = −1
con lo que el campo G pedido es G(r) = yzi − xzj. R RRR
3) El enunciado del problema pide que se evalúen las integrales Σ F × Gdσ y − Ω
kFk2 dv
siendo F y G los campos del apartado 2 y Σ la esfera unidad con la orientación saliente, comprobando
que en ambos casos se obtiene el mismo resultado.
En primer lugar, F × G = − (2xz 2 , 2yz 2 , x2 z + y 2 z). Utilizando que al ser Σ la esfera unidad su
normal unitaria saliente es n = (x, y, z) se tiene
Z Z Z
¡ ¢
I = F × Gdσ = (F × G) ·ndσ = − 2xz 2 , 2yz 2 , x2 z + y 2 z (x, y, z)dσ=
ΣZ Σ Σ Z
¡ 2 2 2 2 2 2 2 2
¢ ¡ 2 2 ¢
= − 2x z + 2y z + x z + y z dσ = − 3x z + 3y 2 z 2 dσ
Σ Σ

R Ahora
2 2
se puede
R 2 2utilizar que, al ser la esfera
R 2invariante
2
ante permutaciones de los ejes coordenados,
Σ
x z dσ = Σ
y z dσ con lo que I = −6 Σ
x z dσ. Utilizando la paridad de la función subintegral
R
y la simetrı́a de Σ respecto de los tres planos coordenados se tiene finalmente I = −48 Σ0 x2 z 2 dσ
donde Σ0 es la porción de Σ contenida en el primer octante. Parametrizando Σ0 en la forma

(x, y, z) = Φ(θ, ϕ) = (cos θsenϕ, senθsenϕ, cos ϕ), θ ∈ [0, π/2], ϕ ∈ [0, π/2]
° °
° °
y utilizando que ° ∂Φ
∂θ
(θ, ϕ) × ∂Φ
∂ϕ
(θ, ϕ)° = senϕ se tiene
Z Z π Z π
2 2
I = −48 2 2
x z dσ = −48 dθ (cos θsenϕ)2 (cos ϕ)2 senϕdϕ =
Σ0 0 0
Z π Z π
2 2
= −48 cos2 θdθ sen3 ϕ cos2 ϕdϕ
0 0

Ahora,
Z π Z π Z π Z π
2
2 1 2 1 2 1 2 π π
cos θdθ = (1 + cos 2θ)dθ = dθ + cos 2θdθ = + 0 =
0 2 0 2 0 2 0 4 4
Z π
2 1 3 1 Γ(2)Γ(3/2) 2
sen3 ϕ cos2 ϕdϕ = β(2, ) = = ··· =
0 2 2 2 Γ(7/2) 15

y se obtiene finalmente −48 π4 15


2
= − 8π
5
.
Por otro lado, utilizando argumentos de simetrı́a y paridad análogos a los anteriores se tiene
Z Z Z Z Z Z Z Z Z Z Z Z
2 2 2 2 2
− kFk dv = − (x + y + 4z )dv = −6 z dv = −48 z 2 dv
Ω Ω Ω Ω0
donde Ω0 es la porción de la esfera unidad contenida en el primer octante. Haciendo un cambio a
esféricas
(x, y, z) = (r cos θsenϕ, rsenθsenϕ, r cos ϕ), θ ∈ [0, π/2], ϕ ∈ [0, π/2], r ∈ [0, 1]
con jacobiano igual a r2 senϕ se obtiene
Z Z Z Z Z Z Z π Z π Z 1
2 2
2 2
− kFk dv = −48 z dv = −48 dθ dϕ r2 cos2 ϕr2 senϕdr =
Ω Ω 0 0 0 0
Z π Z π Z 1
2 2 π 1 £ ¤0 1 π11 8π
= −48 dθ cos2 ϕsenϕdϕ r4 dr = −48 cos3 ϕ π/2 = −48 =−
0 0 0 23 5 235 5
con lo que se comprueba que el resultado es el mismo que en el caso de la integral de superficie.
4)
4.1. Γ es la curva cerrada resultante de la intersección del plano x + y + z = 1 con la superficie
cilı́ndrica x2 + y 2 = 1. Puesto que la proyección de la curva sobre el plano xy es precisamente la curva
x2 + y 2 = 1, se puede obtener una parametrización de Γ parametrizando dicha proyección mediante
x = cos t, y = sent, t ∈ [0, 2π] (nótese que la orientación concuerda con la fijada en el enunciado) y
entrando con dichos valores en z = 1 − x − y, es decir, la parametrización γ(t) dada por
x = cos t, y = sent, z = 1 − cos t − sent, t ∈ [0, 2π]
Por definición de integral curvilı́nea
Z Z 2π
Gdr = G(γ(t)) · γ 0 (t)dt
Γ
Z0 2π
= [sent(1 − cos t − sent)(−sent) − cos t(1 − cos t − sent)(cos t)] dt =
0
Z 2π Z 2π
£ 2 2
¤
= (sen t + cos t)(−1 + cos t + sent) dt = (−1 + cos t + sent)dt = −2π
0 0
R 2π R 2π
pues 0 cos tdt = 0 sentdt = 0. Nótese que si en vez de simplificar en la tercera igualdad se
desarrolla el producto se obtiene
Z Z 2π
Gdr = (− cos2 t − sen2 t + cos3 t + sen3 t + cos tsen2 t + cos2 tsent)dt =
Γ
Z0 2π
= (−1 + cos3 t + sen3 t + cos tsen2 t + cos2 tsent)dt = −2π
0
R 2π R 2π R 2π R 2π
pues 0 cos3 tdt = 0 sen3 tdt = 0 cos tsen2 t = 0 cos2 tsentdt = 0.
4.2. Aplicando el teorema de Stokes
Z Z Z
Gdr = rotGdσ = Fdσ
Γ S S

donde S es cualquier superficie regular que se apoya en Γ y cuya orientación es coherente con la de
Γ. Por simplicidad elegimos como S la porción del plano √ x + y + z = 1 contenida dentro de Γ y
orientada según el vector normal unitario n = (1, 1, 1)/ 3. Parametrizamos S en cartesianas en la
forma
(x, y, z) = Φ(x, y) = (x, y, f (x, y)), (x, y) ∈ D
donde f (x, y) = 1 − x − y y D = {(x, y) : x2 + y 2 ≤ 1}, y utilizando que
° ° s µ ¶2 µ ¶2 q
° ∂Φ ° √
° (x, y) × ∂Φ (x, y)° = 1 + ∂f +
∂f
= 1 + (−1)2
+ (−1)2
= 3
° ∂x ∂y ° ∂x ∂y
se obtiene
Z Z Z Z Z √
1 1
Gdr = F · ndσ = √ (x + y − 2z)dσ = √ (x + y − 2(1 − x − y)) 3dxdy =
Γ
ZS Z 3 S Z Z 3 D

= (−2 + 3x + 3y)dxdy = −2 dxdy = −2Area(D) = −2π


D D
RR RR
donde se ha utilizado que D
xdxdy = D
ydxdy = 0 puesto que la función subintegral es impar
en x (resp. en y) y D es simétrico respecto del eje x = 0 (resp. y = 0). Nótese que, como debe ser,
se ha obtenido el mismo resultado que en el apartado 4.1.
AMPLIACIÓN DE CÁLCULO (Curso 2003/2004) Convocatoria de junio 10.06.04

Solución del
PROBLEMA 1
(4 puntos)
Este ejercicio consta de cuatro preguntas independientes, cada una de las cuales vale 1 punto.

Este ejercicio consta de cuatro preguntas independientes, cada una de las cuales vale 1 punto.
1) Decidir razonadamente para qué valores del parámetro real p converge la integral impropia:
Z 1
t
p
dt.
0 | log t|

La integral es impropia en ambos extremos del intervalo; el logaritmo es negativo en (0, 1), ası́ pues
| log t| = − log t. Hagamos el cambio de variable x = − log t, la integral se escribe
Z ∞ −x Z ∞
e −x
I= p
e dx = e−2x x−p dx
0 x 0

y la homotecia y = 2x nos permite reconocer una gamma,


Z ∞
p−1
I=2 e−y y −p dy = 2p−1 Γ(1 − p)
0

que, como sabemos, es convergente para 1 − p > 0, es decir p < 1.

2) Se considera el sólido limitado por las superficies x2 + z 2 = y 2 , y = 0, y = 1, orientado por el


vector normal exterior. Calcular el flujo del campo F(x, y, z) = (y 2 + z 2 , −y 2 , 2yz) a través de su
superficie lateral cónica.
El campo F es polinómico y solenoidal, div F(x, y, z) = −2y + 2y = 0. Llamemos V al sólido, Σ
a la superficie lateral cónica y D a la superficie plana de ecuaciones x2 + z 2 ≤ 1, y = 1. En virtud
del teorema de Gauss se tiene:
ZZZ ZZ ZZ
div F = F+ F = 0,
V Σ D
ZZ
luego para calcular la integral pedida F basta con calcular el flujo a través del disco D orientado
Σ
por el vector j,
ZZ ZZ ZZ ZZ ZZ
2
F=− F=− F·j=− (−y ) dσ = dσ = Área (D) = π.
Σ D D D D
AMPLIACIÓN DE CÁLCULO (Curso 2003/2004) Convocatoria de junio 10.06.04

Solución del
PROBLEMA 2
(3 puntos)

Sea Γ la curva definida por las ecuaciones cartesianas

2x − y = 0, z − x3/2 = 0 (x, y, z ≥ 0)

y F el campo vectorial dado por

F(x, y, z) = (xy 2 , x2 y, −zx2 ) .

Se pide:
1) Hallar la longitud del arco de Γ determinado por los puntos (0, 0, 0) y (1, 2, 1).
2) Sea C la circulación de F sobre el arco de la curva Γ determinado por el punto (0, 0, 0) y un
punto P arbitrario de la misma. Estudiar si el valor de C alcanza un máximo. En caso afirmativo,
determinar las coordenadas del punto P correspondiente.
3) Sea α una función de clase C 1 en R tal que α(0) = 0 y Λ una curva definida por las ecuaciones
cartesianas
2x − y = 0 , z − α(x) = 0 .
Determinar las funciones α tales que se anule la circulación de F desde (0,0,0) hasta cualquier punto
de la curva Λ.

Respuesta: Se entregará esta hoja y, a lo sumo, una adicional.


1) En primer lugar se debe parametrizar la curva Γ. Puesto que Γ está definida por la intersección
de dos superficies dadas por ecuaciones del tipo f (x, y) = 0, g(x, z) = 0, es decir, en la primera no
interviene la z y en la segunda no interviene la y, la parametrización de la curva se puede llevar a
cabo de forma trivial utilizando la x como parámetro. Por ejemplo se puede tomar la parametrización
r(t) dada por
x = t, y = 2t, z = t3/2
con t ≥ 0 pues según el enunciado sólo se considera la curva en el primer octante (de hecho, en el
resto de R3 la curva no está ni siquiera definida).
Ahora, puesto que (0, 0, 0) corresponde a t = 0 y (1, 2, 1) a t = 1 se tiene que la longitud pedida
es, por definición,
s
Z 1 Z 1 µ ¶2 Z 1r µ ¶3/2
0 3 9 42 9 29 √ 40 √
L= kr (t)k dt = 2
1 +2 + 2 t1/2 dt = 5 + tdt = 5+ t |t=1
t=0 = 29− 5
0 0 2 0 4 93 4 27 27

2) Puesto que es fácil parametrizar la curva Γ y el campo F tiene una expresión sencilla, la
circulación de F entre dos puntos cualesquiera se puede calcular directamente mediante la definición,
es decir, mediante el cálculo de una integral curvilı́nea (si eso no fuese ası́, se podrı́a comprobar si
F es conservativo y en tal caso la circulación se podrı́a calcular a través de un potencial escalar de
F). Consideramos ası́ la curva entre el punto (0, 0, 0) y un punto genérico correspondiente a un valor
u ≥ 0 del parámetro (es decir, el punto (u, 2u, u3/2 )). Entonces, la circulación como función de u es
Z u Z u Z u
0
¡ 2 2 3/2 2
¢ 3 1/2 3
C(u) = F(r(t)) · r (t)dt = t(4t ), t (2t), −t t · (1, 2, t )dt = (8t3 − t4 )dt
0 0 2 0 2
Rt
(nótese que una expresión del tipo 0 F(r(t)) · r0 (t)dt serı́a incorrecta, pues t denotarı́a a la vez a
la variable de integración y al lı́mite de integración).
Obsérvese que puesto que lo que se quiere estudiar es la existencia de máximos de C(u), se puede
trabajar estudiando las derivadas de C(u), no hace falta calcular la integral anterior. En efecto, por el
teorema fundamental del cálculo, al ser la función subintegral continua, C(u) es derivable y además
3
C 0 (u) = 8u3 − u4
2
Los ceros de C 0 (u) son u = 0 y u = 16 3
. Para saber si son extremos podemos acudir a la segunda
derivada, C 00 (u) = 24u2 − 6u3 , comprobando que C 00 ( 16 3
) < 0, lo que indica que en u = 163
hay un
00
máximo local. En cuanto a u = 0, se tiene C (0) = 0 (con lo que se debe acudir a derivadas de
orden superior), C 000 (0) = 0 y C (4 (0) > 0 con lo que en u = 0 hay un mı́nimo local. Para estudiar
la existencia de máximo global, basta darse cuenta que el lı́mite de C(u) cuando u tiende a ∞ vale
−∞ y que C( 16 ) > C(0) para poder afirmar que el máximo global se da en u = 16 , que corresponde
3
16 32 64
√ 3
al punto ( 3 , 3 , 9 3).
3) Razonando de la misma manera que en el apartado 1, para la curva dada se puede tomar la
parametrización w(t) dada por
x = t, y = 2t, z = α(t)
donde t pertenece a un conjunto todavı́a sin determinar.
La circulación de F entre el punto (0, 0, 0) y un punto genérico al que asignamos un valor de
parámetro u es
Z u Z u Z u
0
¡ 2 2 2
¢ 0
H(u) = F(w(t)) · w (t)dt = t(4t ), t (2t), −α(t)t · (1, 2, α (t))dt = (8t3 − α(t)α0 (t)t2 )dt
0 0 0

Sabemos que H(u) debe ser nula para todo valor de u. La condición necesaria y suficiente es
que la función subintegral sea identicamente nula, es decir,
8t3 − α(t)α0 (t)t2 = 0 para todo t (A)
Para razonar esto rigurosamente, se puede proceder de la siguiente forma. Si se cumple (A) entonces
claramente H(u) es nula para todo valor de u. Recı́procamente, si H(u) = 0 para todo u, derivando y
aplicando el teorema fundamental del cálculo se obtiene (A), como se querı́a demostrar. Otra forma
de razonar lo anterior es que la única posibilidad de que H(u) sea nula para todo valor de u es que
F sea ortogonal a la curva en todos los puntos, es decir, F(w(t)) · w0 (t) = 0 para todo t, con lo que
se obtiene (A).
Se ha obtenido una ecuación diferencial de variables separadas para α. Se tiene ası́, dividiendo
por t2
α(t)α0 (t) = 8t
e integrando
α2 (t) √
= 4t2 + C ⇔ α2 (t) = 8t2 + K ⇔ α(t) = ± 8t2 + K
2
donde K es una constante. Como α(0) debe ser cero, se tiene que K = 0 y resulta ası́ que hay dos
soluciones dadas por √
α(t) = ±2 2t

BAREMO DE CALIFICACIÓN (sobre 10 puntos)


Apdo. 1 −→ 2 puntos
Apdo. 2. −→ 4 puntos
Apdo. 3. −→ 4 puntos
AMPLIACIÓN DE CÁLCULO (Curso 2003/2004) Convocatoria de septiembre 16.09.04

PROBLEMA 1 (4 puntos) Este ejercicio consta de tres preguntas independientes. La dos primeras
valen 1 punto y la tercera 2 puntos.

1) Decidir razonadamente para qué valores de los parámetros reales p y q converge la integral
impropia: Z ∞
tp (log t)q dt.
1

Respuesta a 1): Se trata de una integral impropia debido a que el integrando puede no estar
acotado en el extremo inferior del intervalo y también debido a que el intervalo de integración no
está acotado superiormente.
Para el estudio de su convergencia es conveniente realizar el cambio de variable: x = log t, del
que se obtiene: Z Z
∞ ∞
p q
t (log t) dt = xq e(p+1)x dx.
1 0
Ambas integrales se comportan de la misma manera en lo que a su convergencia se refiere. Conside-
ramos pues la segunda y distinguimos los siguientes casos:
A) p+1 > 0. Puesto que la integral es impropia en los dos extremos del intervalo, será convergente
si y sólo si lo son simultáneamente las dos integrales:
Z a Z ∞
q (p+1)x
I1 = x e dx , I2 = xq e(p+1)x dx,
0 a

donde a > 0 es cualquier número real (finito). Ahora bien, si p > −1, I2 es divergente para todo
valor de q ∈ R y, por tanto, la integral de partida es siempre divergente en este caso (lo que se puede
concluir sin necesidad de analizar el comportamiento de I1 ).
La demostración de que I2 es divergente se puede hacer de forma sencilla utilizando el siguiente
n
• Criterio de Comparación: Sea f integrable en [a, x] para todo
Z +∞x > a. Entonces si lı́m x→+∞ x f (x)
es finito y no nulo o infinito, para algún n ≤ 1, la integral f (x) dx es divergente.
a

Bata entonces tener en cuenta que, para p + 1 > 0 y para todo q ∈ R, se tiene
lı́m xq+1/2 e(p+1)x = ∞.
x→∞

B) p + 1 = 0. En este caso, la integral de partida será convergente si lo son simultáneamente:


Z a Z ∞
q
J1 = x dx , J2 = xq dx,
0 a

donde a > 0 es cualquier número real (finito). Sin más que aplicar la definición de integral impropia
se comprueba que J1 y J2 no convergen simultáneamente para ningún valor de q ∈ R\{0} y que,
para q = 0, la integral de partida es, por definición, divergente.
C) p + 1 < 0. En esta situación podemos utilizar la función Gamma de Euler:
Z ∞
Γ(α) = x(α−1) e−x dx
0
(que converge si y solo si α > 0), dado que el cambio de variable (p + 1)x = y (con p + 1 < 0)
transforma la integral de partida en
Z ∞
1 1
q+1
y q e−y dy = Γ(q + 1)
|p + 1| 0 |p + 1|q+1

Por tanto, para p < −1, la integral propuesta es convergente si y solo si q > −1 y diverge si q ≤ −1.
En resumen:

Z ∞
La integral tp (log t)q dt es convergente si y sólo si p < −1 y q > −1 y es divergente en otro
1
caso.

1 punto.

2) Calcular razonadamente todos los valores posibles que puede tomar el flujo del campo vectorial

F(x, y, z) := 2 z 2 x i + z 2 y j − z 3 k

a través de una superficie regular cuyo borde orientado es la curva que resulta de la intersección de
la esfera x2 + y 2 + z 2 = 1 y el cilindro x2 + z 2 = 1/4, y ≥ 0.
Respuesta a 2): En primer lugar y dado que se trata de calcular un flujo es conveniente comprobar
si el campo vectorial es solenoidal o no (es decir si su divergencia es o no nula). Con la notación
F(x, y, z) := (F1 (x, y, z), F2 (x, y, z), F3 (x, y, z)), se tiene:

∂F1 ∂F2 ∂F3


∇·F = + + = 2z 2 + z 2 − 3z 2 = 0
∂x ∂y ∂z

y, por tanto, el campo es, en efecto, solenoidal en R3 . Como R3 es un dominio estrellado y F es de


clase C 1 en R3 , podemos aplicar el Teorema de Gauss que, en particular, nos permite concluir que el
flujo de F a través de cualquier superficie cerrada es cero. De aquı́ se puede deducir que el flujo de F
a través de cualquier superficie con el mismo borde orientado es el mismo, lo que se puede razonar
como sigue:
Dos superficies regulares cualesquiera que tengan el mismo borde orientado siempre forman una
superficie cerrada. Sean entonces Σ+ +
1 y Σ2 dos superficies con un borde orientado común (que induce
un sentido para el vector normal a ambas) S y Σ− −
1 y Σ2 las mismas superficies pero con la otra
+ −
orientación posible. Sea ahora Σ+ = Σ1 Σ2 la superficie cerrada correspondiente (orientada según
el vector normal saliente o entrante). Entonces, el flujo de F a través de Σ+ es nulo y, por tanto:
ZZ ZZ ZZ ZZ ZZ ZZ
F dσ = F dσ + F dσ = 0 =⇒ F dσ = − F dσ = F.
Σ+ Σ+
1 Σ−
2 Σ+
1 Σ−
2 Σ+
2

ALTERNATIVA: Una vez comprobado que el campo es solenoidal, una alternativa más sencilla
al razonamiento (basado en el teorema de Gauss) que se acaba de dar consiste en utilizar el teorema
de Stokes de la siguiente forma.
Puesto que F es solenoidal en R3 , está garantizada la existencia de otro campo vectorial G, que es
un potencial vector de F en R3 , de forma que se cumple F = ∇ × G en R3 . Entonces, para cualquier
superficie Σ con un borde orientado Γ se puede aplicar el teorema de Stokes y, por tanto, se tiene:
ZZ ZZ Z
F dσ ≡ ∇ × G dσ = G ds
Σ Σ Γ
de donde se concluye que el flujo de F solo depende del borde orientado considerado y no de la
superficie que se considere (con ese borde).

Para calcular el flujo pedido, basta entonces considerar una superficie cualquiera con el √ borde
descrito en el enunciado que es la circunferencia x2 + z 2 = 1/4 situada en √ el plano y = 3/2.
Por sencillez, elegimos el cı́rculo C := {(x, y, z) ∈ R3 : x2 + z 2 ≤ 1/4 , y = 3/2}, que se puede
parametrizar mediante γ = (x(r, θ), y(r, θ), z(r, θ)) con:

3
x(r, θ) = r cos θ , y(r, θ) = , z(r, θ) = r sin θ (r ∈ (0, 1/2) , θ ∈ (−π, π)) ,
2
con lo que las dos posibilidades para el vector normal a C son: (0, ±r, 0), que van a dar lugar a dos
posibles valores que puede tomar el flujo pedido (siempre que sea distinto de cero).
Calculamos pues los dos valores posibles del flujo de F a través de C que se corresponden con las
dos orientaciones posibles ((0, r, 0) (C + ) y (0, −r, 0) (C − )) de su vector normal.
 
ZZ Z 1/2 Z π 0
¡ ¢
F dσ = 2 z(r, θ)2 x(r, θ) , z(r, θ)2 y , − z(r, θ)3 ·  r  dr dθ
C+ 0 −π 0
√ Z 1/2 Z π √
3 3π
= r3 sin2 θ dr dθ = ,
2 0 −π 128


y, por tanto, el otro valor que puede tomar el flujo es − .
128

ALTERNATIVA: Para calcular los valores posibles del flujo se puede utilizar el teorema de
Stokes en la forma antes mencionada, lo que exige la obtención de un potencial vector de F, que
siempre se puede elegir de la forma G(x, y, z) = (G1 (x, y, z), G2 (x, y, z), 0). Para calcularlo se identi-
fican componentes en la igualdad F = ∇ × G, de lo que resulta el sistema de ecuaciones en derivadas
parciales siguiente:
∂G2 ∂G1 ∂G2 ∂G1
− = 2z 2 x , = z2y − = −z 3 ,
∂z ∂z ∂x ∂y
de cuya resolución se obtiene: G1 (x, y, z) = − 23 z 3 y + M (x, y), G2 (x, y, z) = 31 z 3 x + N (x, y), donde
∂M ∂N
M (x, y) y N (x, y) son funciones arbitrarias que deben satisfacer la condición − = 0, que
∂x ∂y
se cumple si se elige M = N = 0. ¡ ¢
Ası́ pues, un potencial vector de F es G = − 23 z 3 y , 13 z 3 x , 0 y el flujo pedido vendrá dado por:
 
ZZ Z Z π µ ¶ −(1/2) sin θ
2 1
F dσ = G ds = − z(θ)3 y(θ) , z(θ)3 x(θ) , 0 ·  0  dθ
Σ Γ −π 3 3
(1/2) cos θ
√ Z π √
3 3π
= sin4 θ dθ = ,
48 −π 128

donde Γ es la circunferencia x2 +z 2 = 1/4 situada√ en el plano y = 3/2 para la que se ha considerado
la parametrizacı́on x(θ) = (1/2)
√ cos(θ), y(θ) = 3/2, z(θ) = (1/2) sin(θ) (θ ∈ (−π, π)).
El otro valor posible, − 3 π/128, se obtiene recorriendo la circunferencia en sentido contrario al
considerado en el cálculo anterior.
1 punto.
AMPLIACIÓN DE CÁLCULO (Curso 2003/2004) Convocatoria de septiembre 16.09.04

PROBLEMA 2 (3 puntos)
Sean f : R → R una función de clase C 1 , y V el campo vectorial definido por

V(x, y, z) = yzi + xzj + y f (x) k.

1) Determinar las funciones f para las cuales el campo V es solenoidal. En tal caso, hallar todos los
potenciales vectores de V que sean de la forma L(x, y, z)i + M (y, z)j.
2) Sea Γ la curva intersección de x2 + y 2 + z 2 = 1 (x, y, z ≥ 0) con los tres planos coordenados,
y sea Σ el cono formado por los segmentos de recta que parten del punto (1, 1, 1) y terminan en los
puntos de Γ.
Si f (x) = x + a, calcular la constante a para que el flujo de V a través de Σ sea nulo.

Respuesta: Se entregará esta hoja y, a lo sumo, una adicional.

1) Un campo regular es solenoidal si y sólo si su divergencia es nula. Dado que

∂(yz) ∂(xz) ∂(yf (x))


div (V) = + + = 0,
∂x ∂y ∂z

se tiene que V es solenoidal para toda función f ∈ C 1 (R).


Como V es solenoidal en el dominio estrellado R3 , admite un potencial vector F, es decir,
V = rot(F); si éste es de la forma F(x, y, z) = L(x, y, z)i + M (y, z)j debe cumplirse que
¯ ¯
¯ i j k ¯¯ µ ¶
¯ ∂M (y, z) ∂L(x, y, z) ∂L(x, y, z)
(yz, xz, yf (x)) = V = ¯¯ ∂
∂x

∂y
∂ ¯
∂z ¯ = − , ,− .
¯ L(x, y, z) M (y, z) 0 ¯ ∂z ∂z ∂y

Igualando las primeras componentes e integrando respecto a z se tiene que M (y, z) = −yz 2 /2 +
m(y). Ídem para las segundas componentes, de donde se obtiene que L(x, y, z) = xz 2 /2 + l(x, y);
introduciendo esta función en la tercera componente del rotacional, se deduce que

∂l(x, y) y2
yf (x) = − ⇔ l(x, y) = − f (x) + h(x).
∂y 2

Finalmente, todos los potenciales vectores pedidos son de la forma:


µ 2 ¶ µ ¶
xz y 2 f (x) yz 2
F(x, y, z)= − + h(x) i + − + m(y) j
2 2 2

donde h, m son funciones cualesquiera de C 1 (R) .


2) Como V = rot(F), F ∈ C 1 (R3 ) y Σ es unión de superficies regulares, podemos aplicar el Teorema
de Stokes: Z Z Z
V= rot(F) = F
Σ Σ Γ

puesto que la curva Γ es el borde de la superficie Σ. Como queremos que dicha integral sea nula, en
este caso es indiferente la orientación de la curva Γ, pues la orientación contraria cambiarı́a de signo
la integral, que seguirı́a valiendo cero.
Calculemos, por tanto, dicha integral curvilı́nea, parametrizando Γ. Esta curva es unión de tres
arcos, obtenidos por la intersección del triángulo esférico x2 + y 2 + z 2 = 1 (x, y, z ≥ 0) con cada uno
de los tres planos coordenados. Por tanto, consta de:


 Γ1 (t) = (cos(t), sen (t), 0) t ∈ [0, π/2] en el plano z = 0,
Γ2 (t) = (0, cos(t), sen (t)) t ∈ [0, π/2] en el plano x = 0,

Γ3 (t) = (sen (t) , 0, cos(t)) t ∈ [0, π/2] en el plano y = 0.

Por otro lado, escogemos la forma más sencilla del potencial vector F ya calculado tomando, por
ejemplo, las funciones h y m nulas. Además, obsérvese que, para f (x) = x + a, se tiene que
1¡ 2 ¢ 1 ¡ 2¢
F(x, y, z)= xz − y 2 (x + a) i − yz j
2 2
R
La integral curvilı́nea Γ
F es la suma de las tres integrales siguientes:
Z Z π/2
1 ¡ ¢
F = −sen 2 (t) (cos (t) + a), 0, 0 · (−sen (t) , cos (t) , 0) dt =
Γ1 2 0
Z
1 π/2 1 a
= sen 3 (t) (cos (t) + a)dt = +
2 0 8 3
Z Z π/2 Z
1 ¡ 2 2
¢ 1 π/2 1
F = −a cos (t) , − cos(t)sen (t) , 0 · (0, −sen (t) , cos (t)) dt = sen 3 (t) cos (t) dt =
Γ2 2 0 2 0 8
Z Z π/2 Z π/2
1 ¡ ¢ 1 1
F = sen (t) cos2 (t) , 0, 0 · (cos (t) , 0, −sen (t)) dt = sen (t) cos3 (t) dt. = .
Γ3 2 0 2 0 8
La suma de las tres integrales debe ser cero, de donde
3 a 9
0= + ⇔a=− .
8 3 8
OTRA FORMA: Si se cierra la superficie Σ mediante otra superficie S, de forma que Ω sea el
volumen contenido entre ellas, como V ∈ C 1 (R3 ) podemos aplicar el Teorema de la divergencia
de Gauss: Z Z Z
VdS + VdS = div (V) dV = 0
Σ S Ω
donde se ha utilizado
R que V siempre es solenoidal. Se pide que la integral sobre Σ sea nula, ası́ que
es suficiente que S VdS = 0 para alguna superficie regular S que también tenga por borde a la
curva Γ. Puede elegirse S, por ejemplo, como la unión de las porciones de los tres planos coordenados
limitados por la curva Γ en el primer octante; o bien, como el triángulo esférico del primer octante.
Hagamos
p los cálculos, por ejemplo, para el triángulo esférico parametrizado como S = {(x, y, z)/
z = 1 − x − y 2 , (x, y) ∈ D} siendo D = {(x, y)/ x2 + y 2 ≤ 1, x, y ≥ 0}. La normal dada por la
2

parametrización en cada punto es (x, y, z(x, y))/|z(x, y)|, luego:


Z Z Z
1
0= VdS = (yz, xz, y(x + a)) · (x, y, z) dxdy = (3xy + ay)dxdy.
S D z D

Calculando dichas integrales dobles, se llega al mismo resultado a = − 98 .


AMPLIACIÓN DE CÁLCULO (Curso 2004/2005) Convocatoria de febrero 15.02.05

PROBLEMA 1 (4 puntos) (Este problema consta de dos apartados independientes).


1) Se considera la función compleja de variable compleja definida por
sen πz cos πz
f (z) = + 0 ,
p(z) p (z)
donde p(z) = z 3 (z − α) (α ∈ C) y se supone que sen πα 6= 0 y cos πα 6= 0. Se pide:
1.1) Determinar la parte principal de los desarrollos de f en serie de Laurent alrededor de z = 0
y z = α convergentes en discos perforados con centro en z = 0 y z = α, respectivamente.
I
1.2) Calcular f (z)dz, siendo Γ la frontera de la intersección de los dos dominios del plano
Γ
complejo definidos por |z| + |z − α| ≤ 2|α| y 2|z| ≤ |α|, recorrida en sentido positivo.
(2 puntos)

2) Sea S la superficie de revolución engendrada al girar la curva de ecuación z = y con 1 ≤ y ≤ 2
(situada en plano Y Z) alrededor del eje OZ.
Calcular ángulo sólido de visión desde el origen de la porción de S situada en el primer octante.
(2 puntos)

Respuesta:

1.1) Para determinar la parte principal de la serie de Laurent de la función f alrededor de los dos
puntos considerados se procede, en primer lugar, a identificar el tipo de singularidad que presenta
f en cada uno de ellos. Esta información permite reconocer la forma de la serie de Laurent y, en
particular, permite saber qué coeficientes hay que calcular en cada caso.
(a) El punto z = 0 es un polo doble ya que se cumple:
1 + 3π
lı́m z 2 f (z) = − ,
z→0 3α
donde el lı́mite se calcula aplicando una vez la regla de L’Hopital. Por lo tanto, la serie de
Laurent de f convergente en un disco perforado de centro el punto z = 0 tiene la forma:
X∞
a−2 a−1
f (z) = 2 + + an z n ,
z z n=0

con lo que los coeficientes que hay calcular son a−2 y a−1 . Para ello se tiene:

X 1 + 3π
z 2 f (z) = a−2 + a−1 z + an z n+2 =⇒ a−2 = lı́m z 2 f (z) = − ,
n=0
z→0 3α
donde el lı́mite es el mismo que se ha calculado para identificar la singularidad que presenta f
en el punto z = 0.
Falta únicamente determinar el coeficiente a−1 que es, por definición, el residuo de f en el polo
doble z = 0; por tanto:
µ ¶ µ ¶
d 2 d sen(πz) d cos(πz)
a−1 = Res[f, z = 0] = lı́m (z f (z)) = lı́m + lı́m
z→0 dz z→0 dz z(z − α) z→0 dz (4z − 3α)
π 4 4 + 9π
= − 2 − 2
= − ,
α 9α 9α2
donde el primer lı́mite se calcula aplicando dos veces la regla de L’Hopital y el segundo es
inmediato.
Ası́ pues, la parte principal de la serie de Laurent de f convergente en un disco perforado de
centro el punto z = 0 es:
1 + 3π 4 + 9π
− 2
− .
3α z 9α2 z
(b) El punto z = α es un polo simple ya que se cumple:
sen(πα)
lı́m (z − α) f (z) = 6= 0 (por hipótesis) ,
z→α α3
donde el lı́mite es inmediato. Por lo tanto, en este caso, la serie de Laurent de f que converge
en un disco perforado de centro el punto z = α tiene la forma:
X∞
b−1
f (z) = + bn (z − α)n .
z−α n=0

Hay que determinar el coeficiente b−1 que es, por definición, el residuo de f en el polo simple
z = α y, por tanto, coincide con el lı́mite que se acaba de calcular; es decir:
sen(πα)
b−1 = Res[f, z = α] = lı́m (z − α) f (z) = .
z→α α3
Ası́ pues, la parte principal de la serie de Laurent de f convergente en un disco perforado de
centro el punto z = α es:
sen(πα)
.
α3 (z − α)

1.2) Se trata de calcular la integral de f a lo largo de una curva cerrada ya que la intersección
de los dos dominios dados (el limitado por la elipse de focos z = 0 y z = α y el limitado por la
circunferencia de centro z = 0 y radio |α|/2) siempre tienen una intersección no vacı́a. Se puede, por
tanto, aplicar el teorema de los residuos de Cauchy, según el cual:
I X
f (z)dz = 2π i n(Γ, zk ) Res[f, z = zk ] ,
Γ k

donde la suma se extiende a los puntos singulares zk de f contenidos en el dominio limitado por
la curva Γ y n(Γ, zk ) es el ı́ndice de Γ con respecto a zk , del que se sabe que es un número entero
positivo, pues la curva se recorre en sentido positivo.
En estas condiciones, es necesario determinar todos los puntos singulares de f que son los dos
considerados anteriormente, es decir, el polo doble z = 0 y el polo simple z = α y, además, z = 3α/4
que es un cero simple de p0 (z) y puede ser una singularidad evitable de f (si cos(3απ/4) = 0) o un
polo simple de f (si cos(3απ/4) 6= 0), ya que se cumple:
3α 4 cos(3πα/4)
lı́m (z − ) f (z) = .
z→3α/4 4 9α2
Falta por decidir qué puntos singulares pertenecen al dominio acotado limitado por Γ que serán
aquellos que cumplan simultáneamente las dos desigualdades que definen los dos dominios dados. Es
sencillo comprobar que el único punto que las verifica es z = 0, pues z = α y z = 3α/4 no pertenecen
al disco de centro z = 0 y radio |α|/2. Por tanto:
I
4 + 9π
f (z)dz = 2π i n(Γ, z = 0) Res[f, z = 0] , = − 2πi n(Γ, z = 0) .
Γ 9α2
2) Dada una superficie Σ en R3 y un punto p ∈ R3 se denomina ángulo sólido de visión de Σ desde p
al área que resulta de la intersección de la esfera de centro p y radio uno y el cono sólido de vértice p
formado por todas las rectas que, trazadas desde p, intersecan la superficie Σ en al menos un punto.
En el problema propuesto el punto p es el origen y la superficie Σ es la que se obtiene por

revolución de la curva z = y (1 ≤ y ≤ 2) alrededor del eje OZ. La ecuación cartesiana de esta
qp
superficie es z = x2 + y 2 . Puesto que solamente hay que considerar la porción situada en el
primer octante, una parametrización de Σ viene dada por:
 
 x(θ, u) = u sen θ  π
R(θ, u) = (x(θ, u), y(θ, u), z(θ, u)) con: y(θ, u)
√ = u cos θ (0 ≤ θ ≤ , 1 ≤ u ≤ 2) .
  2
z = u
Por otra parte, puesto que se trata de una superficie de revolución, el cono sólido a considerar
en este caso se obtiene también por revolución alrededor del eje OZ del haz de rectas (situadas en

el plano Y Z) que, trazadas desde el origen, intersecan la curva z = y (1 ≤ y ≤ 2) al menos una

vez. Este haz√está constituido por todas las rectas cuya pendiente está comprendida entre 2/2 y 1

(z = ay con 2/2 ≤ a ≤ 1) y cada una de ellas tiene un único punto en común con la curva z = y
√ √
(como se puede comprobar resolviendo la ecuación ay = y con 2/2 ≤ a ≤ 1). Esto último significa
que desde el origen “se ve toda una cara de la superficie considerada” , lo que permite calcular el
ángulo sólido de Σ (o de la cara de Σ “que se ve desde el origen”) como el flujo del campo r/krk3
sobre Σ (en concreto, sobre la cara de Σ “que no se ve desde el origen”). Debe tenerse en cuenta
que para que el ángulo sólido se pueda calcular mediante este procedimiento el flujo debe calcularse
sobre la porción de la superficie “que se ve desde el origen” (en este caso la totalidad de la superficie).
Denotando por Σ+ esta cara, la normal a considerar es:

∂R(θ, u) ∂R(θ, u) u¡ √ ¢
× = sen θ, cos θ, u .
∂u ∂θ 2
Teniendo en cuenta que sobre la superficie considerada el campo r/krk3 tiene la expresión
R(θ, u) 1 ¡ √ ¢
= u sen θ, u cos θ, u ,
kR(θ, u)k3 (u2 + u)3/2
el ángulo sólido ΩΣ que se desea calcular es:
 
ZZ Z π/2 Z 2 √ u sen θ
r u ¡ √ ¢
ΩΣ = 3
ds = dθ 2 3/2
u sen θ, u cos θ, u  u √cos θ  du
Σ+ krk 0 1 2(u + u) u
Z π/2 Z 2 Z π/2 µ ¶¯2
1 1 1 2 ¯
¯ dθ = √ π ³√ √ ´
= dθ = − √ 3 − 2 .
2 0 1 (1 + u)
3/2 2 0 1 + u ¯1 2 6

2 (Procedimiento alternativo). En lugar de utilizar el flujo del campo r/krk3 , se trata de identi-
ficar directamente la intersección entre el cono sólido antes descrito y la esfera de centro el origen y
radio uno. Una vez identificado este casquete esférico, E, el ángulo sólido se obtendrá calculando su
área. Pero E se obtiene de la intersección √
de la esfera con el cono sólido que resulta de girar alrededor
del eje OZ el haz de rectas z = ay con 2/2 ≤ a ≤ 1 antes mencionado y que está situado en el
primer octante. Una parametrización del mismo es:

π π 2 π
E(θ, φ) = (sen φ cos θ , sen φ sen θ , cos φ) , ≤ φ ≤ − arc tg , 0≤θ≤ ,
4 2 2 2
donde θ varı́a entre 0 y π/2 por estar en el primer octante y los extremos del intervalo de variación
del azimut φ se obtienen restando de π/2 los ángulos cuyas tangentes son las pendientes de las rectas
√ √ √
z = y (que es π/4) y z = ( 2/2) y (que es arc tg 2/2), que son las que cortan a la curva z = y en
sus dos extremos.
Teniendo en cuenta que ° °
° ∂E(θ, φ) ∂E(θ, φ) °
° × ° = sen φ ,
° ∂θ ∂φ °
el ángulo sólido ΩΣ viene dado por
ZZ Z Z √ " Ã √ !#
π/2 π/2−arc tg 2/2
π π π 2
ΩΣ = ds = dθ sen φ dφ = cos − cos − arc tg
E 0 π/4 2 4 2 2
π ³√ √ ´
= √ 3− 2 ,
2 6

donde se han utilizado las relaciones cos(π/2 − a) = sen a y sen arc tg a = a/ 1 + a2 .
AMPLIACIÓN DE CÁLCULO (Curso 2004/2005) Convocatoria de febrero 15.02.05

PROBLEMA 2 (3 puntos)
1) Calcular el campo escalar u : R3 −→ R de clase C 1 para que el campo vectorial F definido por:
F (x, y, z) = (xy 2 z 2 − 2xy, u(x, y, z), x2 y 2 z + 1) y F (0, y, z) = (0, y, 1) ,
sea conservativo en R3 .
2) En tal caso, calcular la circulación de F sobre la curva Γ definida por las ecuaciones
x−z = 0
2

(xz + yz + y − 3) − 1 − x2 = 0
desde el punto A := (0, 2, 0) hasta el punto B := (1, 1, 1).
3) Calcular la circulación del campo vectorial G definido mediante
G(x, y, z) = (y 2 x2 z − 2yz, x3 yz − xz + y, x3 y 2 + 1)
sobre el mismo arco de la curva Γ considerado en el apartado anterior.

Respuesta:

1) Si F es irrotacional, como el dominio R3 es estrellado, F será conservativo. Imponemos, pues:


¯ ¯
¯ i j k ¯
¯ ¯
−→ ¯ ∂ ∂ ∂ ¯
0 = rotF = ¯¯ ¯
¯
¯ 2 2∂x ∂y ∂z ¯
¯ xy z − 2xy u(x, y, z) x2 y 2 z + 1 ¯
µ ¶
2 ∂u ∂u 2
= 2x yz − , 0, − 2xyz + 2x
∂z ∂x
∂u
=⇒ = 2xyz 2 − 2x =⇒ u = x2 yz 2 − x2 + α(y, z)
∂x
∂u ∂α ∂α
=⇒ 2x2 yz = = 2x2 yz + (y, z) =⇒ = 0,
∂z ∂z ∂z
luego α(y, z) solo depende de y. Por otra parte,
F (0, y, z) = (0, y, 1) =⇒ y = u(0, y, z) = α(y, z),
y se tiene
u = x2 yz 2 − x2 + y.
2) Llamamos L, M , N a las tres componentes de F . Fijado un punto del dominio, por ejemplo, el
−−→
origen, un potencial escalar U , es decir, un campo que verifica F = grad U , se puede obtener por la
conocida fórmula
Z x Z y Z z
U (x, y, z) = L(r, 0, 0) dr + M (x, s, 0) ds + N (x, y, t) dt
0 0 0
Z y Z z
2
= (−x + s) ds + (x2 y 2 t + 1) dt
0 0
· ¸s=y · 2 2 2 ¸t=z
2 s2 xy t 2
= −x s + + +t
2 s=0 2 t=0
y 2 x2 y 2 z 2
= −x2 y + + + z.
2 2
La circulación que se pide será
Z
F · dr = U (1, 1, 1) − U (0, 2, 0) = −1.
Γ

3) El campo G no es conservativo pero, en el plano x = z, coincide con F . Por tanto, como Γ


está contenida en ese plano, se tiene:
Z Z
G · dr = F · dr = −1.
Γ Γ
AMPLIACIÓN DE CÁLCULO (Curso 2004/05) Convocatoria de Junio 28-06-05

Problema 2.– (3 puntos) Este problema consta de dos preguntas independientes.

1. (1 punto) Se considera una superficie regular orientada S cuyo borde orientado es la curva
Γ. Sea v un vector no nulo de R3 , y sea r = (x, y, z) un punto genérico de Γ. Determı́nese
razonadamente el valor de α que hace cierta la igualdad:
ZZ Z
α v ds = v × r dr.
S Γ

2. (2 puntos) Sea C una curva simple y acotada del plano R2 . Sea f un campo escalar positivo y
de clase 1 a trozos en un dominio que contiene a C. Sea S la superficie regular definida por

S = {(x, y, z) ∈ R3 , (x, y) ∈ C, 0 ≤ z ≤ f (x, y)}.

Se pide:
Z
a) Demostrar la igualdad A(S) = f , donde A(S) representa el área de la superficie S.
C
b) Calcular explı́citamente el área A(S) cuando la curva C viene parametrizada por ϕ(t) =
(t, sen t), t ∈ [0, 2π] y la función f (x, y) = |y cos x|.

Se entregará esta hoja y, a lo sumo, una adicional.


Solución:

1. Apliquemos el Teorema de Stokes, que relaciona la integral de lı́nea de un campo F sobre la


curva Γ con el flujo de su rotacional (rot (F)) a través de la superficie S:
ZZ Z
rot (F) ds = F dr.
S Γ

En este caso el campo vectorial es


¯ ¯
¯ i v1 x ¯
¯ ¯
F (r) = v × r = ¯¯ j v2 y ¯¯ = (v2 z − v3 y) i+ (v3 x − v1 z) j+ (v1 y − v2 x) k,
¯ k v3 z ¯

y su rotacional es
µ ¶ µ ¶ µ ¶
∂F3 ∂F2 ∂F1 ∂F3 ∂F2 ∂F1
rot (F) = − i+ − j+ − k =2v
∂y ∂z ∂z ∂x ∂x ∂y

con lo cual la expresión del enunciado es cierta cuando y sólo cuando α = 2.

2. a) Si la curva C viene parametrizada como

C = {(x (t) , y (t)), t ∈ [a, b]},


entonces la integral de lı́nea del enunciado es:
Z Z b
f= f (x (t) , y (t)) k(x0 (t) , y 0 (t))k dt.
Γ a

Por otro lado, una parametrización de la superficie es

S = {Φ (t, z) = (x (t) , y (t) , z) , t ∈ [a, b], 0 ≤ z ≤ f (x (t) , y (t))} ;

calculemos su área:
ZZ Z bZ ° °
f (x(t),y(t)) ° ∂Φ ∂Φ °
A (S) = ds = ° °
° ∂t × ∂z ° dt dz.
S a 0

En este caso se tiene que


° °
° ∂Φ ∂Φ °
° ° 0 0 0 0 0 0
° ∂t × ∂z ° = k(x (t) , y (t) , 0) × (0, 0, 1)k = k(x (t) , y (t) , 0)k = k(x (t) , y (t))k ,

donde se ha utilizado que el producto vectorial de dos vectores ortogonales tiene por
norma el producto de las normas. Con todo ello, queda comprobado que el área coincide
con aquella integral de lı́nea:
Z bZ f (x(t),y(t)) Z b Z
0 0 0 0
A (S) = k(x (t) , y (t))k dt dz = f (x (t) , y (t)) k(x (t) , y (t))k dt = f.
a 0 a Γ

b) Apliquemos el resultado anterior al caso particular en que la curva es

C = {ϕ (t) = (t, sen t), t ∈ [0, 2π]}

cos x| . Entonces f (x (t) , y (t)) = |sen t cos t| , y por otro lado k(x0 (t) , y 0 (t))k =
y f (x, y) = |y √
k(1, cos t)k = 1 + cos2 t, ası́ que el área puede calcularse como
Z 2π Z 2π √
0 0
A (S) = f (x (t) , y (t)) k(x (t) , y (t))k dt = |sen t cos t| 1 + cos2 t dt =
0 0
Z · ³ ´3 ¸t=π/2 4 ³ √ ´
π/2 √ 4 √
= 4 2
sent cos t 1 + cos t dt = − 2
1 + cos t = 2 2−1 .
0 3 t=0 3
AMPLIACIÓN DE CÁLCULO (Curso 2004/05) Convocatoria de Junio 28-06-05

PROBLEMA 3 (3 puntos)
Se definen las funciones
µZ x ¶2 Z 1 2 2
−t2 e−x (t +1)
f (x) = e dt ; g(x) = dt.
0 0 t2 + 1
Obviamente, f es derivable en R. Es fácil ver que g también lo es, y que su derivada se obtiene
derivando el integrando respecto de x. Estos hechos pueden darse por válidos, y no se pide su
demostración en este ejercicio. Se pide:
(a) Calcular f 0 (x) + g 0 (x), simplificando el resultado todo lo que se pueda.
(b) Calcular f (x) + g(x), simplificando el resultado todo lo que se pueda.
(c) Calcular con todo rigor lı́m x→+∞ g(x).
(d) Utilizando (b) y (c), calcular el valor de
µ ¶
1
Γ .
2
Nota. Solo se puntuará el apartado (d) si se utilizan (b) y (c).

Respuesta: Se entregará esta hoja y, a lo sumo, una adicional.

(a) Se tiene
Z x
0 −x2 2
f (x) = 2e e−t dt ;
0
Z 1 2 2 Z 1
0 (t2 + 1)e−x (t +1) −x2 2 2
g (x) = −2x 2
dt = −2xe e−x t dt
0 t +1 0
Z x Z x
2 2 du 2 2
= −2xe−x e−u = −2e−x e−u du
0 x 0

donde se ha hecho el cambio de variable xt = u.


Por tanto, f 0 (x) + g 0 (x) ≡ 0.
(b) Integrando la identidad anterior, se tiene f (x) + g(x) ≡ K, pero f (0) = 0, y
Z 1
dt π
g(0) = 2
= [arc tg t]10 =
0 t +1 4
luego
π
= f (0) + g(0) = K,
4
de donde
π
f (x) + g(x) ≡ .
4

(c) Como x2 ≤ x2 (t2 + 1), se tiene −x2 (t2 + 1) ≤ −x2 , por tanto,
Z 1 −x2 (t2 +1) Z 1
e −x2 dt π −x2
0 ≤ g(x) = dt ≤ e = e ,
0 t2 + 1 2
0 t +1 4
de donde,
π −x2
0 ≤ lı́m g(x) ≤ lı́m e = 0,
x→+∞ x→+∞ 4
luego
lı́m g(x) = 0.
x→+∞

(d) Para p > 0, se tiene


Z ∞ Z ∞
p−1 −x 2
Γ(p) = x e dx = 2 t2p−1 e−t dt,
0 0

donde se ha hecho el cambio x = t2 . Haciendo p = 1/2,


µ ¶ Z ∞
1 2
Γ =2 e−y dy.
2 0

Tomando lı́mites en la identidad obtenida en (b) y, usando (c),


³π ´ π
lı́m f (x) = lı́m − g(x) = ,
x→+∞ x→+∞ 4 4
pero µZ ¶2

π −t2
= lı́m f (x) = e dt ,
4 x→+∞ 0

luego µ ¶
1 √
Γ = π.
2
AMPLIACIÓN DE CÁLCULO (Curso 2004/05) Convocatoria de septiembre 13.09.05

PROBLEMA 1 (4 puntos)
Considérese la curva Γ definida por las ecuaciones

x2 + y 2 + z 2 = 9 , (x − 1)2 + y 2 + z 2 = 6 ,

con una orientación tal que su proyección sobre el plano Y Z se recorre con orientación positiva. Se
pide:
1) Hallar unas ecuaciones paramétricas de Γ.
2) Sea G el campo en R3 definido por

G(x, y, z) = (sen2 (x + y) , −z 3 , y 3 ) .

Calcúlese la circulación de G a lo largo de Γ.


3) Sea a ∈ R3 un vector fijo y r = (x, y, z). Calcúlense los valores de p ∈ R para los cuales el campo
vectorial F = (a · r)p · r es solenoidal en un cierto subconjunto de R3 que se especificará.
4) Se considera la superficie Σ formada por los segmentos que unen el punto (1, 0, 0) y los puntos de
Γ, orientada de forma que los vectores normales a la misma tienen la primera componente negativa.
Calcúlese el flujo del campo H(x, y, z) = (1/x2 , y/x3 , z/x3 ) a través de Σ.

Respuesta: Se entregará esta hoja y, a lo sumo, una adicional.


1) La curva Γ es la intersección de dos esferas y por lo tanto será una circunferencia. Desarrollando
la expresión (x − 1)2 + y 2 + z 2 = 6 y utilizando que x2 + y 2 + z 2 = 9 se obtiene que Γ se puede definir
por las ecuaciones

x = 2
y + z2 = 5
2

que muestran que Γ es una circunferencia situada sobre el plano x = 2. De aquı́ se deduce inmedia-
tamente que la aplicación r(t) dada por

x = 2

y = 5 cos t

z = 5sen t, t ∈ [0, 2π)

es una posible parametrización de Γ (donde, como pide el enunciado, la orientación de la proyección


de Γ sobre el plano Y Z se recorre positivamente).
2) Γ es una curva C¡1 y G√es un campo √ C∞ 3
¢ en R luego la circulación está bien definida. Teniendo en
0
cuenta que r (t) = 0, − 5sen t, 5 cos t , la primera componente del campo G no intervendrá en
el cálculo de la circulación C pedida. Se tiene
Z Z 2π
C = G · dr = G(r(t)) · r0 (t)dt =
ZΓ2π ³ √ 0 ³ √ ´ √ ³√ ´´
= (− 5sen t)3 − 5sen t + ( 5 cos t)3 5 cos t dt =
0
Z 2π
¡ 4 ¢
= 25 cos t + sen 4 t dt
0
Utilizando las propiedades de las funciones seno y coseno
Z 2π Z 2π Z π
2
4 4
cos tdt = sen tdt = 4 cos4 tdt
0 0 0
se obtiene Z π
2
C = 200 cos4 tdt
0
integral que se puede resolver por varios métodos. Por ejemplo,
31
1 1 Γ(5/2)Γ(1/2) 1 22
Γ(1/2)2 75
C = 200 β(5/2, 1/2) = 200 = 200 = π
2 2 Γ(3) 2 2 2
3) Si a es nulo, para p ≥ 0 el campo es idénticamente nulo en R3 , y por tanto solenoidal en dicho
conjunto, y no estarı́a definido en ningún punto si p < 0. Sea entonces a ∈R3 no nulo. Si p ≥ 0 el
campo F(r) = (a · r)p r está definido y es de clase infinito en R3 y si p < 0, F sólo está definido, y es
de clase infinito, en el conjunto R3 −{r : a · r =0}, es decir, en todo R3 salvo en el plano perpendicular
a a y que pasa por el origen. Entonces, en los puntos en los que F es C 1 ,
divF(r) = grad( (a · r)p ) · r+ (a · r)p divr =
= p (a · r)p−1 grad (a · r) + (a · r)p 3 =
p (a · r)p−1 a · r+ (a · r)p 3 = (p + 3) (a · r)p
donde se ha utilizado que grad(a · r) = a. Por tanto, divF(r) = 0 en un conjunto Ω ⊂ R3 si y sólo si
(p + 3) (a · r)p = 0 en Ω, con lo que:
Para p = −3, F es solenoidal en Ω = R3 − {r : a · r =0}
Para cualquier p ≥ 0, F es solenoidal sobre el plano a · r =0
4) La superficie Σ es una porción de superficie cónica con vértice (1, 0, 0) y orientada según ”la
normal exterior”. Como x 6= 0 sobre todos los puntos de Σ, H es C 1 en un entorno de Σ y el flujo
está bien definido. El cálculo del flujo parametrizando Σ y calculando la integral doble asociada,
aunque posible, parece a priori complicado, por lo que es razonable buscar posibles alternativas. Se
observa que el campo H es un caso particular de la familia de campos definida en el apartado anterior.
Concretamente corresponde a tomar p = −3 y a = (1, 0, 0). Por ello se sabe que H está definido y es
de clase 1 en R3 − {r : x = 0} y además, divH(r) = 0 en dicho conjunto. Si la superficie Σ se cierra
con otra superficie S de forma que el conjunto A delimitado por las mismas no contenga a ningún
punto del plano x = 0, H es de clase 1 en A y por ello se puede aplicar el teorema de Gauss. Ası́
Z Z Z Z Z
H · dσ+ H · dσ = divH(r)dv = 0
Σ S A
donde S está orientada según la normal exterior. Por comodidad, se puede tomar como superficie S
la porción del plano x = 2 que está encerrada por Γ, estando su orientación definida por el vector
n = (1, 0, 0). Entonces se tiene
Z Z Z
φ : = H · dσ = − H · dσ = − H · ndσ =
Z Σ ZS S
1 1 1 5π
= − 2
dσ = − dσ = − Área(S) = −
S x 4 S 4 4
donde se ha utilizado que, sobre S, x = 2.
Baremo de corrección (sobre 4 puntos)
-Apdo. 1. 0.8 ptos. -Apdo. 2. 0.8 ptos.
-Apdo. 3. 1 pto. -Apdo. 4. 1.4 ptos.
AMPLIACIÓN DE CÁLCULO (Curso 2005/2006) Convocatoria de febrero 07.02.06

PROBLEMA 1 (3 puntos)
Sea Σ la superficie definida por la ecuación cartesiana a |x| + b |y| + z 2 = 1, donde a y b son constantes
positivas, y sea Ω el sólido encerrado por ella.

1. Se sabe que la densidad en cada punto de Ω es proporcional a la suma de las distancias del
punto a los planos XZ e Y Z. Se pide determinar su masa calculando explı́citamente una integral
triple.

2. Sobre Σ hay definida una distribución de carga de forma que la densidad superficial de carga
en cada punto P de Σ es proporcional a la distancia del origen al plano tangente a Σ en P . Se
pide calcular la carga total de Σ.

Respuesta: Se entregará esta hoja y, a lo sumo, una adicional.


De la ecuación que define a Σ se sigue que (x, y, z) ∈ Σ si y sólo si (−x, y, z) ∈ Σ. Análogamente
(x, y, z) ∈ Σ ⇔ (x, −y, z) ∈ Σ y (x, y, z) ∈ Σ ⇔ (x, y, −z) ∈ Σ por lo que Σ es simétrica respecto de
los tres planos coordenados. Además, cortando Σ por planos z = cte, se obtiene a |x| + b |y| = 1 − z 2 ,
por lo que Σ sólo está definida cuando 1 − z 2 ≥ 0, es decir, para z ∈ [−1, 1]. Para algunos de estos
valores de z, las curvas intersección son las que muestra la figura de la izquierda
-1
0-0.5
1 0.5
1

0.5

y por tanto Σ es la superficie cerrada


0
(simétrica con respecto al origen) que
-0.5
se muestra a la derecha :
-1 -0.5 0 0.5 -1
1

RRR
Se pretende calcular la integral triple M = Ω ρ(x, y, z)dxdydz donde ρ(x, y, z) = K1 (|x| + |y|)
y K1 es una constante. Puesto que ρ es una función par en x, parR en
RR
y y par en z y Ω es simétrico
respecto de los tres planos coordenados, se puede escribir M = 8 Ω̂ ρ(x, y, z)dxdydz donde Ω̂ es
la porción de Ω contenida en el primer octante. Además, puesto que en dicho octante x, y y z son
no negativas, se puede escribir
Z Z Z
M = 8K1 (x + y) dxdydz
Ω̂

Puesto que la ecuación


RRR
de Σ no se modifica
RRR
si se intercambian a por b y x por y, parece razonable
expresar M = 8K1 Ω̂ xdxdydz + 8K 1 Ω̂ ydxdydz, de forma que el valor Rde
RR
la segunda integral
será igual al de la primera intercambiando a por b. Calculemos pues I := Ω̂ xdxdydz. Por el
teorema de Fubini, fijando primero z e integrando en x e y se obtiene
Z Z Z Z 1 µZ Z ¶
I= xdxdydz = xdxdy dz
Ω̂ 0 Tz

donde Tz es el triángulo definido por ax + by ≤ 1 − z 2 , x, y ≥ 0. Para


RR
evaluar la primera integral
se puede tener en cuenta que, por definición de centro de gravedad, Tz xdxdy = xG (Tz )Area(Tz ),
donde xG (Tz ) es la coordenada x del centro de gravedad de Tz , supuesto éste homogéneo. Como
el centro de gravedad de un triángulo está situado a 1/3 de su altura, se obtiene que Area(Tz ) =
1 (1−z ) (1−z ) (1−z2 )
2 2

2 a b
y xG (Tz ) = 13 a por lo que
Z Z
1 ³ ´
2 3
xdxdy = 1 − z
Tz 6a2 b
Si no se trabaja con el centro de gravedad de Tz , se puede utilizar Fubini para, integrando primero
en y y luego en x escribir
 
Z Z Z Z
1Z
1−z 2 1−z 2 −ax 1−z 2
a b a
xdxdy = x dy  dx = x(1 − z 2 − ax)dx =
Tz 0 0 b 0
1 ³ ´
2 3
= ··· = 1 − z
6a2 b
con lo que, como debe ser, se llega al mismo resultado. En definitiva
1 Z 1³ ´
2 3 1 Z 1³ 2 4 6
´
I = 1 − z dz = 1 − 3z + 3z − z dz = · · · =
6a2 b 0 6a2 b 0
1 16 8
= 2
=
6a b 35 105a2 b
Usando la propiedad de simetrı́a a la que se aludió anteriormente,
Z Z
8
ydxdydz =
Ω̂ 105b2 a
Si uno no se da cuenta de dicha propiedad de simetrı́a, se puede proceder de la siguiente forma
   
Z Z Z 1 µZ Z ¶ Z 1 Z 1−z 2 Z 1−z 2 −by
b a
ydxdydz = ydxdy dz =  y dx dy  dz =
Ω̂ 0 Tz 0 0 0
 
Z 1
1Z 1 Z 1³
1−z 2 ´
b 2 3 8
=  y(1 − z 2 − ay)dy  dz = 2
1 − z dz = · · · =
0 a 0 6b a 0 105b2 a

Por lo tanto, la masa de Ω es


µ ¶
64K1 1 1 64K1
M= 2
+ 2 = (a + b)
105 a b ab 105a2 b2
2) En primer lugar hay que hacer constar que en las aristas de Σ el plano tangente no está definido
y por ello tampoco lo está la densidad de carga. Sin embargo como el conjunto de dichas aristas tiene
contenido de Jordan nulo en R2 , la circunstancia anterior no influye en el cálculo de la carga total
de Σ.
Para cada punto (x, y, z) de Σ la distancia al origen del plano tangente a Σ en (x, y, z) es igual a
la que se obtiene al considerar los puntos (−x, y, z), (x, −y, z) y (x, y, −z) por lo que por simetrı́a se
puede escribir que la carga total Q de Σ es

Q = 8Q̂

donde Q̂ es la carga total de la porción Σ̂ de Σ contenida en el primer octante.


Para calcular el plano Π tangente a Σ̂ en el punto (x, y, z) se puede calcular el gradiente de la
función g(x, y, z) := ax + by + z 2 − 1, ∇g(x, y, z) = (a, b, 2z) y entonces (X, Y, Z) pertenece a Π si y
sólo si verifica la ecuación
a (X − x) + b (Y − y) + 2z (Z − z) = 0
es decir,
aX + bY + 2zZ − (ax + by + 2z 2 ) = 0
que, utilizando que (x, y, z) ∈ Σ̂ y que por ello ax + by + z 2 = 1, se puede escribir
aX + bY + 2zZ − (1 + z 2 ) = 0
La distancia de Π al origen es
|a0 + b0 + 2z0 − (1 + z 2 )| 1 + z2
√ = √
a2 + b2 + 4z 2 a2 + b2 + 4z 2
y por ello la densidad superficial de carga q(x, y, z) definida en Σ es
1 + z2
q(x, y, z) = K2 √
a2 + b2 + 4z 2
Por tanto Z Z
1 + z2
Q = 8Q̂ = 8 q(x, y, z)dσ = 8K2 dσ √
Σ̂ Σ̂ a2 + b2 + 4z 2
Para parametrizar Σ̂, la mejor opción es expresar la superficie mediante una ecuación cartesiana
explı́cita. Además, y puesto que g(x, y, z) es cuadrática en z pero lineal en x e y, es conveniente
despejar o bien x o bien y como función de las otras variables. Por la simetrı́a del problema en
x e y ambas opciones son equivalentes en cuanto a dificultad. Ası́ por ejemplo expresamos x =
h(y, z)
n
= a1 (1 − by − z 2 ), (y, z) ∈ B donde
o
B es la proyección de Σ̂ sobre el plano yz, es decir,
B = (y, z) : 0 ≤ z ≤ 1, 0 ≤ y ≤ b (1 − z ) . Puesto que ∂h
1 2
∂y
= ab y ∂h
∂z
= −2z
a
se tiene
s
Z Z Z
1 + z2 1 + z2 b2 4z 2
Q = 8K2 √ dσ = 8K 2 √ 1 + + 2 dydz =
Σ̂ a2 + b2 + 4z 2 B a2 + b2 + 4z 2 a2 a
Z Z 2 √ Z Z
1+z 1 2 8
= 8K2 √ a + b2 + 4z 2 dydz = (1 + z 2 )dydz =
B a + b + 4z a
2 2 2 a B
Z 1 ÃZ 1 2)
! Z 1
8K2 b
(1−z 8K2
= (1 + z 2 ) dy dz = (1 + z 2 )(1 − z 2 )dz =
a 0 0 ab 0
8K2 Z 1 32K2
= (1 − z 4 )dz =
ab 0 5ab
Si, a pesar de que no es la√mejor opción, se parametriza Σ̂ despejando z en función de x y de
y, en la forma z = β(x, y) = 1 − ax − by, (x, y) ∈ D = {(x, y) : ax + by ≤ 1, x, y ≥ 0} se puede
proceder como sigue. Puesto que
∂β −a ∂β −b
= √ ; = √
∂x 2 1 − ax − by ∂y 2 1 − ax − by
entonces, usando que, en D, |1 − ax − by| = 1 − ax − by :
Z
1 + z2
Q = 8K2 √ dσ =
Σ̂ a2 + b2 + 4z 2
v
Z Z u
1 + (1 − ax − by) u
t1 + a2 b2
= 8K2 q + dxdy
D a2 + b2 + 4(1 − ax − by) 4(1 − ax − by) 4(1 − ax − by)
q
Z Z a2 + b2 + 4(1 − ax − by)
1 + (1 − ax − by) 1
= 8K2 q √ dxdy =
D a2 + b2 + 4(1 − ax − by) 2 1 − ax − by
Z Z
1 + (1 − ax − by)
= 4K2 √ dxdy =
D 1 − ax − by
Z Z Z Z q
dxdy
= 4K2 √ + 4K2 1 − ax − bydxdy
D 1 − ax − by D
RR
Calculemos I := D (1 − ax − by)−1/2 dxdy aplicando el teorema de Fubini, integrando primero
respecto de y y luego respecto de x:
Z Z Z 1/a ÃZ 1
(1−ax)
!
−1/2 b −1/2
I : = (1 − ax − by) dxdy = (1 − ax − by) dy dx =
D 0 0
Z 1/a µ ¶
2 2 Z 1/a
= (1 − ax − by)1/2 |y=0
y= 1b (1−ax)
dx = (1 − ax)1/2 dx =
0 b b 0
2 2 4
= (1 − ax)3/2 |x=0
x= a1 =
b 3a 3ab
Análogamente
Z Z Z 1/a ÃZ 1
(1−ax)
!
1/2 b 1/2
J : = (1 − ax − by) dxdy = (1 − ax − by) dy dx =
D 0 0
Z 1/a µ ¶
2 2 Z 1/a
= (1 − ax − by)3/2 |y=0
y= 1b (1−ax)
dx = (1 − ax)3/2 dx =
0 3b 3b 0
2 2 4
= (1 − ax)5/2 |x=0
x= a1 =
3b 5a 15ab
Por tanto µ ¶
4 4 32K2
Q = 4K2 (I + J) = 4K2 + =
3ab 15ab 5ab
AMPLIACIÓN DE CÁLCULO (Curso 2005/2006) Convocatoria de febrero 07.02.06

PROBLEMA 2 (3 puntos)

1.1 Sea f un campo escalar de clase 2 en R3 . Enúnciese y justifı́quese una condición necesaria y
suficiente sobre f para que exista un campo vectorial F en R3 de forma que ∇f = rot F.

1.2 Sea F un campo vectorial de clase 2 en R3 . Enúnciese y justifı́quese una condición necesaria y
suficiente sobre F para que exista un campo escalar f en R3 de forma que rot F = ∇f .
Nota: Los resultados teóricos que se precisen deben enunciarse con todo rigor pero sin demostración.

2.1 Determı́nese g : R → R con g(0) = 0, g(1) = −2 para que el campo escalar f (x, y, z) =
x2 + g(y) + z 2 cumpla la condición hallada en (1.1) y, en tal caso, calcúlese un potencial vector
de ∇f de la forma Q(x, z)j + R(x, y)k.

2.2 Sean Q, R los campos escalares calculados en (2.1). Determı́nense todos los campos escalares
P : R3 → R para los cuales F(x, y, z) = P (x, y, z)i + Q(x, z)j + R(x, y)k satisface la condición
hallada en (1.2).

Respuesta:

1.1 (0,5 puntos) Para que un campo vectorial de clase 1 en un abierto estrellado (en este caso todo
el espacio R3 ) sea un rotacional es necesario y suficiente que sea solenoidal; esto es, que su
divergencia sea nula. Ahora bien, div ∇f = ∆f . Ası́ pues, la condición sobre f es que sea un
campo escalar armónico. En tal caso F es un potencial vector de ∇f .

1.2 (0,5 puntos) Un campo vectorial de clase 1 en un abierto simplemente conexo es un gradiente
si, y solamente si, es irrotacional; es decir, su rotacional es nulo. Por lo tanto, la condición
sobre F es que rot rot F = 0. Se dice que f es un potencial escalar de rot F.

2.1 (1 punto) Tenemos que imponer que el campo f (x, y, z) = x2 +g(y)+z 2 sea armónico; calculemos
su laplaciano:

∂2f ∂2f ∂2f


∆f (x, y, z) = 2
+ 2
+ 2
= 2 + g 00 (y) + 2,
∂x ∂y ∂z

el laplaciano es nulo si y solo si g 00 (y) = −4, integramos: g 0 (y) = −4y + a, g(y) = −2y 2 + ay + b
e imponemos las condiciones g(0) = 0, g(1) = −2, con lo que se llega a

g(0) = a = 0, g(1) = −2 + a + b = −2 ⇒ a = b = 0, g(y) = −2y 2 .

Por tanto el campo buscado es f (x, y, z) = x2 − 2y 2 + z 2 . Ahora se pide calcular un potencial


vector del gradiente de f de la forma Q(x, z)j+R(x, y)k; calculemos el rotacional de este campo
e igualémoslo al gradiente de f :

i j k

∂ ∂ ∂
= 2x i − 4y j + 2z k ⇐⇒


∂x ∂y ∂z

0 Q(x, z) R(x, y)
∂R ∂Q ∂R ∂Q
− = 2x, − = −4y, = 2z.
∂y ∂z ∂x ∂x
Integremos ∂R/∂x y ∂Q/∂x en la variable x:
∂R
= 4y ⇒ R(x, y) = 4xy + r(y),
∂x
∂Q
= 2z ⇒ Q(x, z) = 2xz + q(z).
∂x
Y ahora impongamos la primera relación:
∂R ∂Q
− = 2x ⇒ 4x + r0 (y) − 2x − q 0 (z) = 2x ⇐⇒ r0 (y) − q 0 (z) = 0 ⇐⇒ r0 = q 0 = cte
∂y ∂z
Si el valor de la constante es a se tiene r(y) = ay + b y q(z) = az + c con a, b, c constantes
que podemos elegir arbitrariamente, en particular un potencial vector de ∇f es G(x, y, z) =
2xzj + 4xyk.
2.2 (1 punto) Si F es de clase 2, tenemos que imponer que P también sea de clase 2, esto nos
permitirá integrar las ecuaciones en derivadas parciales que surjan, asimismo nos garantiza la
igualdad de las derivadas parciales cruzadas de segundo orden.
Tenemos que imponer que rot rot F se anule en R3 ; ahora bien, como G(x, y, z) = Q(x, z)j +
R(x, y)k es un potencial vector de un gradiente (rot G = ∇f ), se cumple rot rot G = 0, además
el rotacional es un operador lineal:
rot F = rot(P i) + rot G = rot(P i) + ∇f,
por lo que basta con calcular los campos P para los cuales rot rot(P i) = 0. Calculemos, en
primer lugar, rot(P i):


i j k

∂ ∂ ∂ ∂P ∂P
rot(P i) =

=

j− k

∂x ∂y ∂z
∂z ∂y
P (x, y, z) 0 0

y ahora rot rot(P i) e igualemos a cero:



i j k

∂ ∂ ∂
∂2P ∂2P ∂2P ∂2P
! !
∂P ∂P
rot j− k = ∂x ∂y ∂z =− + i+ j+ k = 0.

∂z ∂y ∂P ∂P

∂y 2 ∂z 2 ∂x∂y ∂x∂z
0 −
∂z ∂y

De ∂ 2 P/∂x∂y = 0 se deduce que ∂P/∂x no es función de y, existirá una función H1 de dos


variables de forma que ∂P/∂x = H1 (x, z). Derivamos respecto de z:
∂2P ∂H1
= = 0 ⇒ H1 (x, z) = H2 (x).
∂x∂z ∂z
Integremos ∂P/∂x = H2 (x) en la variable x,
P (x, y, z) = h(x) + C(y, z)
donde h es una primitiva de H2 y C es arbitraria. Por último, al imponer la nulidad de la primera
componente del doble rotacional: ∂ 2 P /∂y 2 + ∂ 2 P /∂z 2 = 0 se concluye que el laplaciano de C
es nulo, ası́ pues las funciones P buscadas son
P (x, y, z) = h(x) + C(y, z), h ∈ C 2 (R), C ∈ C 2 (R2 ), ∆C = 0.

Gabriela Sansigre.
AMPLIACIÓN DE CÁLCULO (Curso 2005/2006) Convocatoria de febrero 07.02.06

NOMBRE . . . . . . . . . . . . . . . . . . . . . . . . . APELLIDOS . . . . . . . . . . . . . . . . . . . . . . . . . . . . . . . . . . . . . . . . . . .
Número de matrı́cula. . . . . . . . . . . . . . . . . . . . . . . .

PROBLEMA 3 (4 puntos)
Se considera el lóbulo Γ de la lemniscata ρ2 = a2 cos 2θ contenido en el semiplano x ≥ 0, siendo

ø
a > 0 una constante. Se pide:
(1) Dados dos puntos A, B ∈ Γ, de ángulos polares respectivos ϕ < ψ, calcular las coordenadas

Ö
(x0 , y0 ) del centro de masas G del arco AB en función de la longitud l del arco (que no es necesario
calcular en ningún momento en este problema), de ϕ y de ψ.
(2) Si O es el origen de coordenadas, demostrar que la recta OG es la bisectriz del ángulo AOB.
Para ello, se sugiere considerar el arco C de una circunferencia centrada en O, de radio arbitrario,
formado por los puntos de ángulo polar θ ∈ [ϕ, ψ], calcular las coordenadas (x∗0 , y0∗ ) del centro de
masas G∗ de C, y comparar las posiciones de G y G∗ respecto de O.
(3) Calcular las coordenadas (xs , ys ) del centro de masas del dominio plano encerrado por el
lóbulo de la lemniscata. Hallar el volumen engendrado por ese dominio al girar 360◦ en torno al eje
OY .

Respuesta: Se entregará esta hoja y, a lo sumo, una adicional.


(1) Sean
π π
≤ϕ<ψ≤ . −
4 4
Derivando respecto de θ en la ecuación de la lemniscata ρ2 = a2 cos 2θ, se tiene

a2 sen 2θ
2ρρ′ = −2a2 sen 2θ ⇒ ρ′ = − ,
ρ
de donde

2 a4 sen2 2θ
ρ2 + ρ′ = a2 cos 2θ +
a2 cos 2θ

ø
a2
= .
cos 2θ

Por tanto, a tı́tulo informativo y sin completar el cálculo, la longitud l del arco AB es
Z ψ q Z ψ dθ
l= ρ(θ)2 + ρ′ (θ)2 dθ = a √ .
ϕ ϕ cos 2θ

ö
Las coordenadas del centro de masas verifican:
Z Z ψ q
lx0 = x ds = ρ(θ) cos θ ρ(θ)2 + ρ′ (θ)2 dθ
AB ϕ
Z ψ √ a
= a cos 2θ cos θ √ dθ
ϕ cos 2θ
Z ψ
cos θ dθ = a2 [sen θ]ψϕ = a2 (sen ψ − sen ϕ).

ö
= a2
ϕ
Z Z ψ q
ly0 = y ds = ρ(θ) sen θ ρ(θ)2 + ρ′ (θ)2 dθ
AB ϕ

1
y

B C
ψ G• G∗•
x
0 ϕ a R
A
Γ

Figura 1: Centro de masas de un arco de lemniscata

Z ψ √ a
= a cos 2θ sen θ √ dθ
ϕ cos 2θ
Z ψ
=a 2
sen θ dθ = a2 [− cos θ]ψϕ = a2 (cos ϕ − cos ψ).
ϕ

Por tanto, Ã !
a2 a2
G(x0 , y0 ) = (sen ψ − sen ϕ), (cos ϕ − cos ψ) .
l l

(2) Sea C el arco de circunferencia de radio R (podemos suponer R > a, aunque no es indis-
pensable), formado por los puntos de ángulo polar θ ∈ [ϕ, ψ] (Figura 1). Una parametrización de C
es (
x(θ) = R cos θ
r(θ) ≡ θ ∈ [ϕ, ψ],
y(θ) = R sen θ
de donde |r ′ (θ)| = R para todo θ ∈ [ϕ, ψ].
La longitud del arco C es l∗ = R(ψ − ϕ). Las coordenadas del centro de masas G∗ de C son:

1 Z
1 Z ψ
x∗0 = x ds = R cos θ R dθ
R(ψ − ϕ) C R(ψ − ϕ) ϕ
R R
= [sen θ]ψϕ = (sen ψ − sen ϕ).
ψ−ϕ ψ−ϕ
1 Z
1 Z ψ
y0∗ = y ds = R sen θ R dθ
R(ψ − ϕ) C R(ψ − ϕ) ϕ
R R
= [− cos θ]ψϕ = (cos ϕ − cos ψ).
ψ−ϕ ψ−ϕ
Se tiene entonces
x∗0 Rl
=
x0 (ψ − ϕ)a2
y0∗ Rl
= ,
y0 (ψ − ϕ)a2

y ambos cocientes son iguales. Por tanto, O, G y G∗ son colineales, y los ángulos polares de G y G∗
coinciden.
Ahora bien, es obvio, por la simetrı́a del arco C, que el ángulo polar de G∗ es
ϕ+ψ
,
2
2
por tanto, este es también el ángulo polar de G, luego OG es la bisectriz del ángulo AOB. Ö
(3) Por simetrı́a, ys = 0. Sea D el dominio encerrado por el lóbulo de la lemniscata. Su área es
ZZ ZZ
Área = dxdy = ρ dρdθ
D D′
Z π/4 √
Z a cos 2 θ
=2 dθ ρ dρ
0 0
Z π/4
2
=a cos 2θ dθ
0
a2 a2
= [sen 2θ]π/4
0 = .
2 2
Para calcular xs necesitamos la integral:
ZZ ZZ
I: = x dxdy = ρ2 cos θ dρdθ
D D′
Z π/4 √
Z a cos 2 θ
= 2 cos θ dθ ρ2 dρ
0 0
2a3 Z π/4 2a3 Z π/4
= cos θ(cos 2θ)3/2 dθ = cos θ(cos2 θ − sen2 θ)3/2 dθ
3 0 3 0
(cambio u := sen θ)

2a3 Z 1/ 2
= (1 − 2u2 )3/2 du
Ã3 0
!
1
cambio u = √ sen t
2
3 Z π/2
2a 1
= (1 − sen2 t)3/2 √ cos t dt
3 0 2
√ √
2 3 Z π/2
2 3 1 5
µ ¶
= a cos4 t dt = aB ,
3 0 6 2 2
µ ¶2
1 5 31 1
µ ¶ µ ¶
√ √
2 3Γ 2 Γ 2 2 3 2 2Γ 2
= a = a
6 Γ(3) 6 2

πa3 2
= .
16
Por tanto, √
I πa 2
xs = = .
Área 8
Por el Teorema de Guldin, el volumen engendrado es:

π 2 a3 2
Vol = 2πxs Área = .
8

3
AMPLIACIÓN DE CÁLCULO (Curso 2005/2006) Convocatoria de junio 20.06.06

PROBLEMA 1 (4 puntos)
Sean a = (a1 , a2 , a3 ) un vector constante y no nulo y k una constante real estrictamente positiva.
Considérese el campo vectorial F(r) definido por:

(a1 x, a2 y, a3 z)
F(r) = ,
(24x2 + 4y 2 + 2z 2 )k

donde r = (x, y, z).

1. 1.a) Determı́nense razonadamente todos los valores posibles del vector a y de la constante k
para los que el campo F(r) es irrotacional, indicando el mayor dominio posible en el que
posee esta propiedad. (1 punto).
1.b) Sea Σ una superficie regular con borde orientado Γ que no contiene el origen. Para los
valores de a y k obtenidos en 1.a), calcúlese razonadamente la circulación del campo F(r)
sobre el borde Γ. (0,5 puntos).

2. 2.a) Sean Σ1 y Σ2 las superficies esféricas de ecuaciones cartesianas (x − 1)2 + y 2 + z 2 = 9 y


(x − 1)2 + y 2 + z 2 = 1/4, respectivamente. Para a = (2, 2, 2) y k = 3/2, calcúlese el flujo
del campo F(r) a través de la cara exterior de Σ1 y de Σ2 . (1,5 puntos)
2.b) Determı́nense razonadamente todos los valores posibles del vector a y de la constante
k para los que el campo F(r) es solenoidal, indicando el dominio en el que posee esta
propiedad. (1 punto).

Respuesta: Se entregará esta hoja y, a lo sumo, una adicional.


1.a) Para determinar los valores de a y k que se piden, calculamos el rotacional del campo:
4
µ ¶
rot F(r) = ∇ × F(r) = (a2 − 2a3 )kyz , (a1 − 12a3 )kxz , (2a1 − 12a2 )kxy
(24x + 4y 2 + 2z 2 )k+1
2

expresión que es válida en todo R3 salvo el origen puesto que k > 0.


Para que el campo tenga rotacional nulo (sea irrotacional) sus tres componentes deben anularse.
Teniendo en cuenta que k no puede ser cero, esto da lugar a un sistema lineal homogéneo de tres
ecuaciones con tres incógnitas, cuya expresión es:
     
0 1 −2 a1 0
1 0 −12 a2  = 0 .
     

2 −12 0 a3 0

Dado que el rango de la matriz de coeficientes de este sistema es 2, existen infinitas soluciones que se
obtienen expresando dos de las incógnitas en términos de la tercera. Por ejemplo, eligiendo las dos
primeras ecuaciones se tiene:
a1 = 12a3 , a2 = 2a3 .
Ası́ pues, el campo F(r) dado es irrotacional para todo valor de k > 0 y para

a = a3 (12, 2, 1) con a3 ∈ R ,

y posee esta propiedad en R3 \{(0, 0, 0)}.


1.b) De partida se tiene: ∇ × F(r) = 0 en R3 \{(0, 0, 0)}. Dado que, por hipótesis, se trata de
una superficie regular con borde orientado, podemos aplicar el teorema de Stokes, según el cual:
I ZZ
F(r) · dr = [∇ × F(r)] · dr = 0 ,
Γ Σ

puesto que la superficie no contiene el origen.


También se puede razonar del siguiente modo: Puesto que la curva Γ es el borde de una superficie
regular que no contiene el origen, es necesariamente una curva cerrada en R3 que tampoco lo contiene.
Esto permite afirmar que siempre se puede encontrar un dominio estrellado (o estelar), Ω ⊂ R3 , que
contenga la curva y no el origen. En Ω (que es estelar y contiene a Γ) el campo es irrotacional y, por
tanto, conservativo lo que implica que su circulación a lo largo de cualquier curva cerrada contenida
en él es cero.

2.a) A la hora de calcular el flujo de un campo vectorial a través de una superficie es siempre
conveniente comprobar, antes de aplicar la definición de flujo, si el campo es o no solenoidal. Ello
se justifica por la existencia del Teorema de Gauss cuya aplicación, si es posible, puede simplificar
considerablemente los cálculos.
En este caso, para los valores dados del vector a = (2, 2, 2) y la constante k = 3/2, se comprueba
fácilmente que, en efecto, se trata de un campo solenoidal, puesto que ∇ · F(r) = 0. Además, el
campo dado presenta esta propiedad en R3 \{(0, 0, 0)}.
• Consideremos, en primer lugar la superficie Σ1 de ecuación cartesiana (x − 1)2 + y 2 + z 2 = 9.
Como el dominio de R3 limitado por esta superficie esférica contiene el origen y la divergencia del
campo no está definida en ese punto, no es posible aplicar el Teorema de Gauss directamente en este
caso. La integral de volumen de la divergencia del campo extendida al dominio limitado por Σ1 no
está definida.
Sin embargo si es posible utilizarlo para sustituir la superficie dada por otra que simplifique los
cálculos. El razonamiento es el siguiente: Dada la superficie esférica Σ1 , siempre se puede encontrar
otra regular y cerrada Σ0 que no contenga el origen con la siguiente propiedad: el dominio limitado
por Σ0 en R3 contiene el origen y, además, está contenido en el dominio limitado por Σ. En estas
condiciones, la aplicación del Teorema de Gauss al dominio D ⊂ R3 comprendido entre las dos
superficies permite afirmar que el flujo a través de las caras exteriores de ambas superficies coincide,
lo que es debido a que el campo vectorial es solenoidal en ese dominio D (que no contiene el origen).
Podemos entonces elegir Σ0 como la superficie de ecuación cartesiana 24x2 + 4y 2 + 2z 2 = ²,
donde siempre se puede encontrar un ² > 0 de forma que se cumplan las condiciones sobre Σ0 antes
mencionadas. Ası́ pues:
ZZ ZZ
2 ZZ
F(r) · dσ = F(r) · dσ = r · dσ ,
Σ1 Σ0 ²3/2 Σ0

donde r = (x, y, z) y, en la última igualdad, se ha tenido en cuenta que el valor del campo sobre Σ0
2
es, precisamente: F(r)|Σ0 = 3/2 r, motivo por el cual se ha elegido Σ0 en la forma dada.
²
Falta por calcular la última integral extendida a Σ0 y para ello podemos volver a aplicar el teorema
de Gauss para escribir:
2 ZZ 2 ZZZ 6 ZZZ
r · dσ = ∇ · r dV = dV
²3/2 Σ0 ²3/2 V0 ²3/2 V0

donde se ha tenido en cuenta que ∇ · r = 3.


Finalmente, la última integral es el volumen del elipsoide 24x2 + 4y 2 + 2z 2 ≤ ², cuyos semiejes
q q q π²3/2
son 24² , 4² , 2² . Por tanto, el volumen es √ y el flujo pedido:
6 3
ZZ
π
F(r) · dσ = √ .
Σ1 3
• En el caso de la superficie esférica Σ2 , de ecuación cartesiana (x − 1)2 + y 2 + z 2 = 1/4, resulta
que el domino de R3 que limita no contiene el origen y, por tanto, el campo F(r) es solenoidal en el.
La aplicación directa de Teorema de Gauss nos permite concluir que el flujo de F(r) a través de Σ2
es cero.

2.b) Para determinar los valores del vector a y de la constante k > 0 para los que el campo
vectorial dado es solenoidal, calculamos su divergencia:
2 n
div F(r) = ∇ · F(r) = 12(a1 + a2 + a3 − 2a1 k)x2 +
(24x2 + 4y 2 + 2z 2 )k−1
´
2(a1 + a2 + a3 − 2a2 k)y 2 + (a1 + a2 + a3 − 2a3 k)z 2 .

Al igualar a cero resulta el siguiente sistema lineal homogéneo de tres ecuaciones:


     
1 − 2k 1 1 a1 0
 1 1 − 2k 1   a2  = 0 .
    

1 1 1 − 2k a3 0

Para que admita soluciones distintas de la idénticamente nula, el determinate de la matriz de coe-
ficientes debe ser cero. De ello se obtiene: 4k 2 (3 − 2k) = 0 =⇒ k = 3/2 ó k = 0, donde la última
solución se descarta pues, por hipótesis, k es estrictamente positiva.
Para k = 3/2 el rango de la matriz de coeficientes anterior es 2 y, por tanto, estamos en una
situación análoga a la que se planteó en el apartado 1.a); es decir, existen infinitas soluciones que se
obtienen expresando dos de las incógnitas en términos de la tercera. Por ejemplo:

−2a1 + a2 = −a3 

=⇒ a1 = a2 = a3 .
a1 − 2a2 = −a3

Ası́ pues, el campo F(r) dado es solenoidal para k = 3/2 y para

a = a3 (1, 1, 1) con a3 ∈ R ,

y, dado el valor de k, posee esta propiedad en R3 \{(0, 0, 0)}.


AMPLIACIÓN DE CÁLCULO (Curso 2005/2006) Convocatoria de junio 20.06.06

PROBLEMA 2 (3 puntos)

1) Sea k < 0. La integral impropia Z ∞


xk−1 cos x dx
1
¿es convergente? ¿Es absolutamente convergente?
2) Sea k < 0. La integral impropia Z ∞
xk sen x dx
1
¿es convergente?
3) Aplicar lo anterior para estudiar la convergencia de la integral impropia
Z ∞√
t sen(t2 ) dt.
1

Nota: cada apartado vale un punto.

Respuesta: Se entregará esta hoja y, a lo sumo, una adicional.

1) Acotamos el valor absoluto del integrando:


¯
¯ k−1
¯ 1
¯x cos x¯¯ ≤ xk−1 =
x1−k
y dado que 1 − k > 1 se tiene que
dx
Z ∞
<∞
1 x1−k
ası́ que por el criterio de mayoración, se tiene que la integral 1∞ xk−1 cos x dx converge absolu-
R

tamente, y, en particular, converge.


2) Integremos por partes, con el fin de relacionar esta integral con la integral del apartado anterior.
Lo hacemos sobre un intervalo finito [1, M ], y luego hacemos tender M a infinito:
Z M Z M
k k
x sen x dx = cos 1 − M cos M + k xk−1 cos x dx
Z1 ∞ Z ∞
1
k k−1
x sen x dx = cos 1 + k x cos x dx < ∞
1 1

donde, por un lado, se ha utilizado que, para k < 0,


¯ ¯
¯ k
¯M cos M ¯ ≤ M k −
−−−→ 0
¯
M →∞

xk−1 cos x dx es finita (por el apartado anterior).


R∞
y por otro lado, el hecho de que 1
xk sen x dx es convergente para k < 0.
R∞
Por todo ello, 1

3) Realizamos el cambio de variable t2 = x, que no varı́a los lı́mites de integración:


Z ∞√ Z ∞√
2 1 1 Z ∞ −1
t sen(t ) dt = 4
x sen x √ dx = x 4 sen x dx
1 1 2 x 2 1
que se trata de una integral impropia como la del apartado anterior, donde k = − 41 < 0, por lo
que concluimos que es una integral convergente.
AMPLIACIÓN DE CÁLCULO (Curso 2005/2006) Convocatoria de junio 20.06.06

PROBLEMA 3 (3 puntos)

Se considera el arco C de la lemniscata ρ2 = a2 cos 2θ contenido en el primer cuadrante (x ≥ 0,


y ≥ 0), siendo a > 0 una constante. Se pide:

1. Calcular las coordenadas (x0 , y0 ) del centroide del arco C en función de la longitud l del arco
(que no es necesario calcular en ningún momento en este problema).

2. Se considera ahora el dominio plano acotado D limitado por C en el primer cuadrante. Se pide
hallar las coordenadas (xD , yD ) del centroide de D. Si aparece una función hiperbólica inversa,
se deberá expresar el resultado en términos de la función logaritmo.

Nota: La primera pregunta vale un punto y la segunda dos.

Respuesta: Se entregará esta hoja y, a lo sumo, una adicional.

1. Es bien sabido que, en el primer cuadrante, la lemniscata existe solamente para ángulos polares
comprendidos entre 0 y π/4.
Derivando respecto de θ en la ecuación de la lemniscata ρ2 = a2 cos 2θ, se tiene

a2 sen 2θ
2ρρ0 = −2a2 sen 2θ ⇒ ρ0 = − ,
ρ

de donde

2 a4 sen2 2θ
ρ2 + ρ0 = a2 cos 2θ +
a2 cos 2θ
a2
= .
cos 2θ

Por tanto, a tı́tulo informativo y sin completar el cálculo, la longitud l del arco C es
Z π/4 q Z π/4 dθ
l= ρ(θ)2 + ρ0 (θ)2 dθ = a √ .
0 0 cos 2θ

C
D
0 a x

Figura 1: Arco de lemniscata en el primer cuadrante


Las coordenadas del centroide del arco C verifican:
Z Z π/4 q
lx0 = x ds = ρ(θ) cos θ ρ(θ)2 + ρ0 (θ)2 dθ
C 0
Z π/4 √ a
= a cos 2θ cos θ √ dθ
0 cos 2θ
Z π/4 a2
=a 2
cos θ dθ = a2 [sen θ]π/4
0 = √ .
0 2
Z Z π/4 q
ly0 = y ds = ρ(θ) sen θ ρ(θ)2 + ρ0 (θ)2 dθ
C 0
Z π/4 √ a
= a cos 2θ sen θ √ dθ
0 cos 2θ Ã !
Z π/4 1
=a 2
sen θ dθ = a 2
[− cos θ]π/4
0 =a 2
1− √ .
0 2
Por tanto, Ã Ã !!
a2 a2 1
(x0 , y0 ) = √ , 1− √ .
l 2 l 2

2. El área del dominio D es:


ZZ ZZ
Área = dxdy = ρ dρdθ
D D0

Z π/4 Z a cos 2 θ
= dθ ρ dρ
0 0
2
a Z π/4
= cos 2θ dθ
2 0
a2 a2
= [sen 2θ]π/4
0 = .
4 4
Para calcular xD necesitamos la integral:
ZZ ZZ
Ix : = x dxdy = ρ2 cos θ dρdθ
D D0

Z π/4 Z a cos 2 θ
= cos θ dθ ρ2 dρ
0 0
3
a Z π/4 a3 Z π/4
= cos θ(cos 2θ)3/2 dθ = cos θ(cos2 θ − sen2 θ)3/2 dθ
3 0 3 0
(cambio u := sen θ)

a3 Z 1/ 2
= (1 − 2u2 )3/2 du
Ã3 0
!
1
cambio u = √ sen t
2
3 Z π/2
a 1
= (1 − sen2 t)3/2 √ cos t dt
3 0 2
√ √
2 3 Z π/2
2 3 1 5
µ ¶
4
= a cos t dt = aB ,
6 0 12 2 2
µ ¶2
1 5 31 1
µ ¶ µ ¶
√ Γ Γ √ Γ
2 3 2 2 = 2 a3 2 2 2
= a
12 Γ(3) 12 2
3

πa 2
= .
32
Por tanto, √
Ix πa 2
xD = = .
Área 8

Análogamente, para calcular yD , necesitamos la integral:


ZZ ZZ
Iy : = y dxdy = ρ2 sen θ dρdθ
D D0

Z π/4 Z a cos 2 θ
= sen θ dθ ρ2 dρ
0 0
3
a Z π/4 a3 Z π/4
= sen θ(cos 2θ)3/2 dθ = sen θ(cos2 θ − sen2 θ)3/2 dθ
3 0 3 0
(cambio u := cos θ)

a3 Z 1/ 2
= −(2u2 − 1)3/2 du
³3 1
³ √ ´´
cambio t := arg ch u 2
a3 Z 0 1 4
= √ − √ sh t dt
3 arg ch 2 2

a3 Z arg ch 2 4
= √ sh t dt .
3 2 0

Ahora bien,

1 t
sh4 t = (e − e−t )4
16
1 4t
= (e − 4e2t + 6 − 4e−2t + e−4t )
16
1
= (2 ch 4t − 8 ch 2t + 6) ,
16
luego

a3 Z arg ch 2
Iy = √ (2 ch 4t − 8 ch 2t + 6) dt
48 2 0
" #arg ch √2
a3 sh 4t
= √ − 4 sh 2t + 6t
48 2 2 0
a 3 ³ √ √ √ ´
= √ 6 2 − 8 2 + 6 arg ch 2 .
48 2

Por otra parte, si w = arg ch v, se tiene:

1 w
v = ch w = (e + e−w )
2
⇒ 2vew = e2w + 1
⇒ e2w − 2vew + 1 = 0

⇒ ew = v ± v 2 − 1
³ √ ´
⇒ w = log v ± v 2 − 1 .

Como ³ √ ´ ³ √ ´
log v + v2 − 1 y log v − v2 − 1
tienen signos opuestos, y la función arg ch toma solo valores no negativos, se tiene
√ ³√ ´
arg ch 2 = log 2+1 .

Por tanto,

a3 ³ √ ³ √ ´´
Iy = √ 2 2 + 6 log 1 + 2
48 2
Ã√
√ ´
!
3 2 ³ 1
=a log 1 + 2 − ,
16 24

de donde
Iy a ³ √ ³ √ ´ ´
yD = = 3 2 log 1 + 2 − 2 .
Área 12
AMPLIACIÓN DE CÁLCULO (Curso 2005/2006) Convocatoria de septiembre 12.09.06

PROBLEMA 1 (4 puntos)
Se consideran el campo escalar u(x, y, z) = (9x2 + 4y 2 + z 2 )1/2 y los campos vectoriales:
r
F(x, y, z) = y G(x, y, z) = (P (x, y, z) , Q(x, y, z) , 0) ,
u(x, y, z)3
1 yz 1 xz
donde P (x, y, z) = , Q(x, y, z) = − y r = (x, y, z).
u(x, y, z) (9x + 4y 2 )
2 u(x, y, z) (9x + 4y 2 )
2

1) Calcúlese el rotacional del campo G(x, y, z) sabiendo que se cumple:

∂Q(x, y, z) ∂P (x, y, z) z
− = .
∂x ∂y u(x, y, z)3

¿Es G(x, y, z) un potencial vector de F(x, y, z)? En caso afirmativo, indı́quese el dominio de
R3 en el que se da esta propiedad. Además, obténgase razonadamente un potencial vector del
campo F(x, y, z) cuya tercera componente no sea nula. (1 punto.)

2) Considérese la curva Γ que resulta de la intersección de la superficie elı́ptica de ecuación carte-


siana 9x2 + 4y 2 + z 2 = 2 con el plano z = 1, orientada de forma que su proyección sobre el plano
z = 0 se recorre en sentido positivo. Sea Σ la parte de la superficie elı́ptica anterior que tiene
por borde orientado la curva Γ y está situada en el semiespacio z ≥ 1. Utilı́cese la definición
de flujo y de circulación para calcular el flujo de F(x, y, z) a través de Σ y la circulación de
G(x, y, z) a lo largo de la curva Γ.
(2 puntos.)

3) Enúnciese con toda precisión el teorema de Stokes. Decı́dase razonadamente si los resultados
obtenidos en el apartado anterior contradicen o no lo establecido en este teorema. (1 punto.)

Respuesta:
1) El rotacional del campo G(x, y, z) es, por definición:
( )
∂Q(x, y, z) ∂P (x, y, z) ∂Q(x, y, z) ∂P (x, y, z)
rot G := − , , − .
∂z ∂z ∂x ∂y

Teniendo en cuenta que:


∂u z
= z (9x2 + 4y 2 + z 2 )−1/2 = ,
∂z u
se tiene:
!
∂Q(x, y, z) x ∂ z x 1 z ∂u x
 
− = = − 2 =
∂z (9x2 + 4y 2 ) ∂z u (9x2 + 4y 2 ) u u ∂z u3
!
∂P (x, y, z) y ∂ z y 1 z ∂u y
 
= 2 2
= − 2 = .
∂z (9x + 4y ) ∂z u (9x + 4y 2 )
2 u u ∂z u3
Como se indica en el enunciado, la tercera componente de rot G es z/u3 y, por tanto, se da la
r
igualdad rot G = = F; es decir, G es un potencial vector de F en todo conjunto de R3 en
u(x, y, z)3
el que estén definidas las derivadas parciales ∂P/∂y, ∂P/∂z, ∂Q/∂x y ∂Q/∂z. Esto ocurre en todo
R3 menos el eje z (nótese que para x = y = 0 el denominador de las dos primeras componentes de
G se anula). Ası́ pues, G es un potencial vector de F en el conjunto
n o
C = R3 \ (x, y, z) ∈ R3 : x = y = 0 .

Una forma de razonar para obtener a partir de G otro potencial vector de F, consiste en tener
en cuenta que el rotacional de un gradiente (cuando está definido) es siempre nulo y, por tanto,
basta considerar un campo escalar V (x, y, z) de clase C 2 en R3 para que G + gradV sea también un
potencial vector de F. La tercera componente de este nuevo potencial vector será ∂V /∂z, por tanto
habrá que elegir el campo escalar V (x, y, z) de forma que ∂V /∂z 6= 0. Un ejemplo sencillo puede ser
V (x, y, z) = z, en cuyo caso se obtiene el potencial vector de F dado por (P (x, y, z), Q(x, y, z), 1),
cuya tercera componente no es nula.
2) Circulación de G sobre Γ: La curva Γ es una elipse situada en el plano z = 1 que tiene por
ecuación cartesiana
9x2 + 4y 2 = 1 , z = 1 ,
con lo que una de sus parametrizaciones es r(θ) = (x(θ), y(θ), z(θ)), con:
1 1
x(θ) = cos θ , y(θ) = sen θ , z(θ) = 1 ,
3 2
y θ ∈ [0, 2π] para que su proyección sobre el plano z = 0 se recorra en sentido positivo.
El valor de campo G sobre Γ es: G |Γ = √12 (y, −x, 0) con lo que, por definición, la circulación
pedida es:

− 13 sen θ
 
Z Z
1 Z 2π 
1 1

π
1
Gdr = G |Γ dr = √  2 cos θ  dθ = − √ .
sen θ , − cos θ , 0 ·  
Γ Γ 2 0 2 3 3 2
0

Flujo de F a través de Σ: Considerando como parámetro el ángulo de las coordenadas elı́pticas y


la coordenada z, una parametrización de la superficie considerada es

r(θ, v) = (x(θ, v), y(θ, v), z(θ, v)) ,

con:
√ √
2 − v2 2 − v2 √
x(θ, v) = cos θ , y(θ, v) = sen θ , z(θ, v) = v , θ ∈ [0, 2π] , v ∈ [1, 2] .
3 2
La cara, Σ+ , de la superficie orientada que hay que considerar es la que tiene por vector normal
√ √ !
∂r(θ, v) ∂r(θ, v) 2 − v2 2 − v2 v
× = cos θ , sen θ , .
∂θ ∂v 2 3 6

Por tanto, el flujo pedido es, por definición:


√ !
ZZ
1 Z 2π Z 2 ∂r(θ, v) ∂r(θ, v)
F · ds = √ r(θ, v) · × dv dθ
Σ+ 2 2 0 1 ∂θ ∂v
√ √ !
1 Z 2π Z 2 1 π √  π 2
= √ dv dθ = √ 2−1 = 1− .
2 2 0 1 3 3 2 3 2

Nota: Aunque no es lo que se pide en el enunciado, el flujo anterior puede calcularse aplicando
adecuadamente el teorema de Gauss, ya que el campo F es solenoidal en R3 \ {(0, 0, 0)}.
3) Un enunciado del teorema de Stokes puede ser el siguiente:
Sea Σ una superficie regular y orientable cuyo borde es la curva Γ. Sea H un campo vectorial de
clase C 1 en un abierto A ⊆ R3 que contiene a Σ y a su borde Γ. Entonces:
ZZ Z
rot H · dσ = H · dr ,
Σ Γ

donde la cara de Σ y la orientación de Γ que se consideran son las que resultan de aplicar la “regla
del sacacorchos”.
Para decidir si los resultados obtenidos en el apartado anterior contradicen o no este teorema hay
que comprobar, en primer lugar, que los campos F y G verifican sus hipótesis. Pero toda superficie
cuyo borde sea la curva Γ contiene al menos un punto que pertenece al eje z en el que el campo G
no es de clase C 1 . Ası́ pues, el teorema de Stokes no se puede aplicar en este caso y el hecho de que,
según muestran los cálculos anteriores,
ZZ ZZ Z
F = rotG 6= Gdr ,
Σ+ Σ+ Γ

no lo contradice.
AMPLIACIÓN DE CÁLCULO (Curso 2005/2006) Convocatoria de septiembre 12.09.06

PROBLEMA 2 (3 puntos): Este ejercicio consta de dos apartados independientes.

1) Calcúlese el volumen del sólido interior a la esfera unidad centrada en el origen, e interior al
cilindro x2 + y 2 = 1/4. (1 punto)

2) Sea el recinto D = {(x, y), 0 < x < 1, 0 < y < 1, x2 + y 2 > 1} y C su curva frontera, orientada
positivamente. Considerando el campo vectorial
à !
x
F(x, y) = ,0 ,
1 + x2 + y 2

relaciónense las integrales


ZZ
xy Z
dxdy, F · dr
D (1 + x2 + y 2 )2 C

y calcúlese sólo una de ellas. (2 puntos)

Respuesta: Se entregará esta hoja y, a lo sumo, una adicional.

1) El volumen pedido puede calcularse de muchas formas distintas, por ejemplo: como volumen de
sólido de revolución, o bien como suma de volumen del cilindro más volumen de dos casquetes
esféricos, o bien pasando a coordenadas esféricas, o cilı́ndricas, etc. Aquı́ incluimos la siguiente
solución alternativa, como volumen proyectable sobre la región plana D : x2 + y 2 ≤ 1/4 , ya
que la sólido viene definido por
½ q q ¾
3
Ω = (x, y, z) ∈ R : (x, y) ∈ D, − 1− x2 − y2 ≤z≤ 1− x2 − y2 .

Su volumen V es
 √ 
Z Z Z Z Z Z 1−x2 −y 2 Z Z q
V = dx dy dz =  √ dz  dx dy = 2 1 − x2 − y 2 dxdy
Ω D − 1−x2 −y 2 D

y, por ejemplo, pasando a polares en D,


√ ! ³ √ ´
´ 3 ¸ρ=1/2 8 − 3 3 π
Ã
2π 1/2 4π 4π 3 3
Z Z q · ³
V =2 ρ 1 − ρ2 dρ dθ = − 1 − ρ2 2
= 1− = .
0 0 3 ρ=0 3 8 6
2) El recinto D del enunciado aparece en la siguiente figura, y se observa que su borde C es una
curva de Jordan que consta claramente de tres arcos C1 , C2 , C3 :
y
C2
1
D
C3
C1

x
0 1

Podemos aplicar el Teorema de Green al campo F que es de clase C 1 en el dominio D, con lo


cual se relaciona su circulación a lo largo de C con una integral doble sobre D:
Z Z Ã !
Z Z
∂Q ∂P
F · dr = (P, Q) ·dr = − dx dy.
C C D ∂x ∂y
x
Además, en este caso Q es nula y P (x, y) = 1+x2 +y 2
ası́ que
Z Z Z
∂P Z Z
2xy
F · dr = − dx dy = dx dy
C D ∂y D (1 + x2 + y 2 )2

con lo que se deduce que la integral curvilı́nea es dos veces la integral doble del enunciado.
Ya que ambas están relacionadas como acabamos de decir, calculamos una cualquiera de ellas,
por ejemplo, la integral curvilı́nea. La curva C orientada positivamente consta de tres partes:
un segmento vertical C1 , otro horizontal C2 y un arco de circunferencia C3 ; por ello, hay que
calcular sobre cada una de esas curvas la integral de lı́nea
Z Z Z
x
F · dr = P dx + 0dy = dx
C C C 1 + x2 + y 2
R
En el segmento vertical dx = 0, luego C1 P dx = 0.
Por otro lado, el segmento horizontal está formado por los puntos (x, 1) con x ∈ [0, 1], pero hay
que cambiarlo de orientación, ası́ que queda:
Z Z 1 x 1h ³ 2 x=1
´i 1
P dx = − 2
dx = − log 2 + x = (log (2) − log (3)) .
C2 0 2+x 2 x=0 2
Por último el arco de circunferencia tiene la orientación opuesta a la de (x(t), y (t)) = (cos t, sen t)
con t ∈ [0, π/2], luego (x0 (t), y 0 (t)) = (−sen t, cos t) y entonces
¸t=π/2
π/2 x (t) 0 Z π/2
cos t 1 1
Z Z ·
P dx = − x (t) dt = sen tdt = − cos2 t = .
C3 0 2 0 2 4 t=0 4

Finalmente, la integral curvilı́nea pedida es


Z Z Z Z
1 1
F · dr = P dx + P dx + P dx = + (log (2) − log (3))
C C1 C2 C3 4 2
mientras que la integral doble es la mitad:
Z Z
xy 1 1
2 2 2
dx dy = + (log (2) − log (3)) .
D (1 + x + y ) 8 4
AMPLIACIÓN DE CÁLCULO (Curso 2005/2006) Convocatoria de septiembre 12.09.06

PROBLEMA 3 (3 puntos)
Este ejercicio consta de tres apartados independientes; cada uno de ellos vale 1 punto.

1) Estúdiese la convergencia de la integral impropia


Z ∞
e−x
√ dx.
0 x3 − x2 − x + 1

2) Determinar los valores reales de m y n para los que la integral


Z ∞ 3 m
(x − 1)
dx
1 x3n+1
converge y calcular su valor para los casos en que sean enteros.

Z ∞ −xt Z t −x2
e e
3) Calcular el valor de t para el cual la función M (t) = dx + 3 dx alcanza su
t x 0 x+1
valor mı́nimo en el intervalo [1, 10].

Solución:
1)
Z ∞
e−x
I= √ dx
0 x3 − x 2 − x + 1
En primer lugar, y debido a la presencia de la raı́z cuadrada, comprobaremos que la función su-
bradical es no negativa en [0, ∞) y que por ello la función subintegral está bien definida. Para ello
factorizaremos el polinomio x3 − x2 − x + 1. Utilizando Ruffini y tanteando la raı́z x = 1 se obtiene
de inmediato que x3 − x2 − x + 1 = (x − 1)2 (x + 1). Se observa que esta función es no negativa en
[0, ∞) por lo que la raı́z cuadrada está bien definida. Por otro lado, y puesto que x = −1 está fuera
del intervalo de integración, la función subintegral
e−x e−x
f (x) = √ =
x3 − x2 − x + 1 |x − 1| (x + 1)1/2
está definida en todo [0, ∞) salvo en el punto x = 1, en cuyo entorno la función no está acotada (pues
lı́mx→1+ = ∞ y lı́mx→1− = ∞). Por consiguiente, la integral es impropia por dos razones: la presencia
de un intervalo de integración no acotado y el hecho de que f no está acotada en el entorno de x = 1.
Obsérvese que f es siempre no negativa, con lo que la convergencia de la integral es equivalente a la
convergencia absoluta y podemos aplicar los criterios de convergencia para integrales impropias en
las que la función subintegral no cambia de signo. Por definición, la integral será convergente si y
sólo si las tres integrales
Z 1 Z a Z ∞
α= f (x)dx , β = f (x)dx , γ = f (x)dx,
0 1 a

cada una de las cuales es impropia sólo por un motivo, son convergentes, donde a es cualquier
número del intervalo (1, ∞).

1
La integral γ converge absolutamente pues (recuérdese que f ≥ 0)
e−x
|x−1|(x+1)1/2
lı́m 1 =0
x→∞
x2
R ∞ dx R ∞ dx
y 1 x2 es convergente ( 1 xp converge si y sólo si p > 1). En cuanto a α

e−x e−x
|x−1|(x+1)1/2 (1−x)(x+1)1/2 e−1
lı́m− 1 = lı́m− 1 =
x→1
1−x
x→1
1−x
21/2
R R
que es finito y no nulo, por lo que 01 f (x)dx y 01 1−xdx
tienen el mismo carácter. Como esta segunda
R 1 dx
integral es divergente ( 0 (1−x)p converge si y sólo si p < 1) se sigue que α es divergente y por
consiguiente, e independientemente del carácter de β, I es divergente. Aunque ya no es necesario
analizar el carácter de β, ésta resulta también divergente, pues
e−x e−x
|x−1|(x+1)1/2 (x−1)(x+1)1/2 e−1
lı́m+ 1 = lı́m+ 1 =
x→1
x−1
x→1
x−1
21/2
R a dx R a dx
que es finito y no nulo, y 1 x−1 es divergente ( 1 x−1 converge si y sólo si p < 1)
m
R (x3 −1)
2) Sea I(m, n) := 1∞ x3n+1 dx. Teniendo en cuenta la forma de la función subintegral y el hecho
de que debemos calcular de forma explı́cita el valor de I(m, n), parece razonable intentar relacionar
R 1 p−1
I con la función beta, β(p, q) = 0 y (1 − y)q−1 dy. Para transformar el intervalo (1, ∞) (para la x)
en el intervalo (0, 1) (para la y) parece razonable realizar el cambio x = φ(y) = 1/y (que obviamente
es un difeomorfismo al ser φ biyectivo y tanto φ como φ−1 de clase uno). Se tiene entonces
³ ´m à !
Z ∞ 3 m Z 0 1
−1 Z 1³ ´m
(x − 1) y3 1
I(m, n) := dx = ³ ´3n+1 − 2 dy = 1 − y 3 y 3n−3m−1 dy
1 x3n+1 1 1 y 0
y

que no es una β debido al término y 3 dentro del paréntesis. Para subsanar esto se puede hacer el
cambio y = η(u) = u1/3 , es decir, u = η −1 (y) = y 3 que es un difeomorfismo de (0, 1) a (0, 1). Entonces
Z 1³ ´m Z 1 µ ¶
3n−3m−1 1 −2
I(m, n) = 1 − y3 y 3n−3m−1 dy = (1 − u)m u 3 u 3 du =
0 0 3
1Z 1 1
= (1 − u)m un−m−1 du = β(m + 1, n − m)
3 0 3
Por supuesto, el procedimiento anterior es totalmente equivalente a haber realizado un único
cambio correspondiente a la composición de los dos anteriores, es decir, x = α(u) = u−1/3 , u = x−3 .
Utilizando las propiedades de la función β se obtiene que I(n, m) converge si y sólo si m > −1 y
n > m. Además, en el caso de que m y n cumplan las condiciones anteriores y sean enteros (es decir,
m = 0, 1, 2, ... y n = m + 1, m + 2, ... se puede escribir

1 1 Γ(m + 1)Γ(n − m)
I(m, n) = β(m + 1, n − m) = =
3 3 Γ(n + 1)
1 m!(n − m − 1)!
=
3 n!
3) Para calcular el mı́nimo de M derivaremos en la expresión de M . Para ello, en primer lugar
comprobaremos que la función M está definida enR [1, 10] y que Res derivable en dicho intervalo. Para
R ∞ e−xt
∞ e−xt 10 e−xt
simplificar los razonamientos podemos expresar t x dx = t x dx + 10 x dx con el fin de
separar la presencia de la variable t en los lı́mites de integración por un lado y la la presencia

2
de un intervalo de integración no acotado por otro. Obsérvese que el partir la integral Rutilizando
−xt
x = 10 es arbitrario, se podrı́an haber utilizado otros valores. Ahora denotemos α(t) := t10 e x dx,
R ∞ e−xt R −x2
β(t) := 10 x
dx y γ(t) := 0t ex+1 dx con lo que M (t) = α(t) + β(t) + 3γ(t).
Para cada t ∈ [1, 10], γ(t) es una integral en sentido propio, pues la función subintegral está aco-
−x2
tada en [0, 10]. Puesto que ex+1 es continua en [0, 10] y toda función continua es integrable, γ(t)
está definida en [1, 10]. Además, usando el teorema fundamental del cálculo integral, γ(t) es deriva-
−t2
ble con continuidad en [1, 10] y su derivada vale et+1 .
−xt
Para cada t ∈ [1, 10], α(t) es una integral en sentido propio, pues f (x, t) := e x está acotada
en Q := [1, 10] × [1, 10]. Como f es continua en Q la integral existe y por ello α está bien definida.
Además como ∂f ∂t
existe y es de clase uno en Q se sigue que α es derivable con continuidad en [1, 10]
−t2 R −xt −t2 R
y α0 (t) = − e t + t10 (−x) e x dx = − e t − t10 e−xt dx.
Por último, para cada t ∈ [1, 10], β(t) es una integral impropia al ser el intervalo de integración
no acotado. Al ser f (x, t) acotada en [10, ∞) × [1, 10] no hay más razones por las que la integral
sea impropia. Demostremos que para todo t ∈ [1, 10] la ¯integral impropia es convergente. En efecto,
¯ −xt
¯e ¯ −x R ∞ e−x
para todo t ∈ [1, 10] y todo x ∈ [10, ∞) se cumple ¯ x ¯ ≤ e x . Como 10 x
dx es convergente, por
el criterio de comparación β(t) converge absolutamente para todo t ∈ [1, 10] (nótese que de hecho
con el razonamiento anterior se ha demostrado, utilizando el criterio de Weierstrass, la convergencia
uniforme de la integral para t ∈ [1, 10]) con lo que β está bien definida. Para demostrar la derivabilidad
de β utilizamos en primer lugar que (a) f es continua en [10, ∞) × [1, 10] y ∂f ∂t
existe y es continua
R∞ ∂f R ∞ −xt
en dicho conjunto. En segundo lugar, demostraremos (b) que la integral 10 ∂t dx = − 10 e dx
converge uniformemente para t ∈ [1, 10]. Para ello emplearemos el criterio
R ∞ −x
de Weierstrass: para todo
−xt −x
t ∈ [1, 10] y todo x ∈ [10, ∞) se cumple |e | ≤ e y además 10 e dx es convergente, de donde
se sigue la convergencia uniforme de la integral.R De (a) y (b) se deduce que β es derivable con
R ∞ ∂f ∞ −xt
continuidad en [1, 10] y que β 0 (t) = 10 ∂t
dx = − 10 e dx.
Por lo tanto M es derivable con continuidad en [1, 10] y
2Z 10 Z ∞ 2
0 0 0 0 e−t −xt e−t
M (t) = α (t) + β (t) + 3γ (t) = − − e dx − e−xt dx + 3 =
t t 10 t+1
2 Z ∞ 2 2 2
e−t −xt e−t e−t e−xt x=t e−t
= − − e dx + 3 =− − | +3 =
t t t+1 t t x=∞ t+1
· ¸
2 2 3 2 t − 2
= e−t − + = e−t
t t+1 t(t + 1)

donde se ha utilizado que al ser t positivo, lı́mx→∞ e−xt = 0.


Se observa que M 0 (t) sólo tiene un cero, t = 2, en el intervalo [1, 10]. Como M 0 (t) pasa de negativa
a positiva al atravesar t = 2 podemos afirmar que t = 2 es un punto de mı́nimo relativo de M . Por
tanto, los únicos candidatos a punto de mı́nimo global en [1, 10] son t = 2 y los extremos del intervalo.
Como M 0 (t) < 0 en [1, 2) y M 0 (t) > 0 en (2, 10] podemos afirmar que t = 2 es el punto de mı́nimo
global que se pide.

3
AMPLIACIÓN DE CÁLCULO (Curso 2006/2007) Convocatoria de febrero 06.02.07

NOMBRE . . . . . . . . . . . . . . . . . . . . . . . . . APELLIDOS . . . . . . . . . . . . . . . . . . . . . . . . . . . . . . . . . . . . . . . . . . .
Número de matrı́cula. . . . . . . . . . . . . . . . . . . . . . . .

PROBLEMA 1 (4 puntos): Se consideran las integrales impropias


Z ∞ Z ∞ sen(ax)
−x
G(a) = e cos(ax) dx. F (a) = e−x dx,
0 0 x
Se pide:

1. Encontrar el conjunto de valores reales de a donde G converge uniformemente. Para dichos


valores, calcular explı́citamente G(a). (1’5 puntos)

2. Enunciar y aplicar el teorema que permite concluir que F es una función derivable en su dominio
de definición. (0’75 puntos)

3. Calcular la integral F (a) para los valores de a donde converge. (0’75 puntos)

4. Para b > 0, relacionar la integral


Z ∞ sen x
H(b) = e−xb dx
0 x
con la función F . Sabiendo que H converge uniformemente en b ≥ 0, deducir que
Z ∞ sen x π
dx = .
0 x 2
(1 punto)

Solución: (Se entregará esta hoja, y a lo sumo, otra más.)

1. El integrando de G(a) puede acotarse en valor absoluto por una función mayorante: |e−x cos ax| ≤
e−x = M (x) que es independiente de a y cuya integral impropia es convergente:
Z ∞ Z ∞
M (x) dx = e−x dx = 1 < ∞
0 0

luego, por el criterio de Weierstrass podemos afirmar que G es uniformemente convergente en


todo R.
Por tanto, G(a) existe para todo a real; podemos calcular su valor o bien integrando por partes
dos veces, o bien usando que G(a) es la parte real de la siguiente integral:
Z ∞ Z ∞ 1
 
1 1 + ia
−x iax (ia−1)x (ia−1)x
e e dx = e dx = lı́m e −1 = =
0 0 ia − 1 x→∞ 1 − ia 1 + a2

donde se ha utilizado que lı́mx→∞ e(ia−1)x = lı́mx→∞ |eiax | e−x = lı́mx→∞ e−x = 0. Por tanto,
su parte real es
Z ∞ Z ∞ 
1 + ia

1
G(a) = e−x cos (ax) dx = Re e−x eiax dx = Re = ∀a ∈ R.
0 0 1 + a2 1 + a2
2. Enunciemos el teorema pedido:
Teorema: Se considera un intervalo cerrado I de la recta real, y la integral impropia
Z ∞
F (a) = f (a, x)dx.
0

Si se verifica:

F (a0 ) converge para algún valor a0 ∈ I,


∂f
∂a
es continua en I × [0, ∞)
la integral impropia Z ∞ ∂f
(a, x)dx
0 ∂a
converge uniformemente en I,

entonces F es derivable en I, y su derivada es


Z ∞ ∂f
0
F (a) = (a, x)dx ∀a ∈ I.
0 ∂a
Veamos que se verifican las tres hipótesis del teorema, tomando I = R:

F converge en a0 = 0, pues obviamente F (0) = 0;


 

∂a
e−x senxax = e−x cos ax que es una función continua para (a, x) en R × [0, ∞)
La integral impropia de la derivada es
Z ∞ ∂f Z ∞
(a, x)dx = e−x cos(ax)dx = G(a)
0 ∂a 0

que ya se ha visto que converge uniformemente en R.

Por todo ello, el teorema garantiza que F es derivable en R, y además concluye que

F 0 (a) = G(a) ∀a ∈ R.

3. De la última igualdad, recuperamos F (a) = arc tg(a) + C y observando por la definición que
F (0) = 0 = C finalmente se llega a la expresión

F (a) = arc tg(a) ∀a ∈ R.

4. Para b > 0, mediante un simple cambio de variable y = xb, se tiene que


∞ ∞
−xb sen x −y sen(y/b) 1 1
Z Z    
H(b) = e dx = e dy = F = arc tg .
0 x 0 y b b

Por otro lado, la convergencia uniforme de H en [0, ∞) garantiza que H es continua en dicho
intervalo, y en particular en b = 0. Por ello, podemos concluir:
Z ∞ sen x 1 π
 
dx = H(0) = lı́m+ H(b) = lı́m+ arc tg = .
0 x b→0 b→0 b 2
AMPLIACIÓN DE CÁLCULO (Curso 2006/2007) Convocatoria de febrero 6.02.07

NOMBRE . . . . . . . . . . . . . . . . . . . . . . . . . APELLIDOS . . . . . . . . . . . . . . . . . . . . . . . . . . . . . . . . . . . . . . . . . . .
Número de matrı́cula. . . . . . . . . . . . . . . . . . . . . . . .

PROBLEMA 2 (3 puntos)
Dado a > 0, sea Γ la curva llamada cardioide, de ecuación polar ρ = a(1+cos θ). Sea D el dominio
limitado por Γ. Se pide:
(1) Hallar la longitud de Γ y el área de D. (1 punto)
(2) Hallar el centroide del arco Γ. (1 punto)
(3) Hallar el centroide del dominio D. (1 punto)

Respuesta: Se entregará esta hoja y, a lo sumo, una adicional.


Puede verse la curva y el dominio en la Figura 1.
y
a Γ

D
2a
0 x

Figura 1: Cardioide

(1) Derivando en ρ(θ) = a(1 + cos θ), se obtiene

ρ′ (θ) = −a sen θ .

Por tanto,

ρ(θ)2 + ρ′ (θ)2 = a2 (1 + cos θ)2 + a2 sen2 θ


= 2a2 (1 + cos θ)
θ
= 4a2 cos2 ,
2
y la longitud del arco es
Z 2π q
l(Γ) = ρ(θ)2 + ρ′ (θ)2 dθ
0
2π θ
Z
= 2a cos dθ

0 2
Z π θ
= 4a cos dθ
"0
2
# π
θ
= 8a sen
2 0

1
= 8a .

Calculamos a continuación el área de D:


ZZ
Área(D) = dxdy
ZZD
= ρ dρdθ
D′
Z 2π Z a(1+cos θ)
= dθ ρ dρ
0 0
2
a 2π
Z
= (1 + cos θ)2 dθ
2 0
a2 2π
Z
= (1 + 2 cos θ + cos2 θ) dθ
2 0
3πa2
= .
2

(2) Calculamos las coordenadas (xΓ , yΓ ) del centroide del arco Γ:

1
Z
xΓ = x ds
l(Γ) Γ
1 2π
Z q
= ρ(θ) cos θ ρ(θ)2 + ρ′ (θ)2 dθ
8a 0
1 Z 2π θ
= a(1 + cos θ) cos θ 2a cos dθ

8a 0 2
Z π
θ
=a cos3 cos θ dθ
0 2 !
Z π
3 θ 2 θ 2 θ
=a cos cos − sen dθ
0 2 2 2
Z π ! !
5 θ 3 θ 2 θ θ
=a cos − cos sen dθ t=
0 2 2 2 2
Z π/2  
= 2a cos5 t − cos3 t sen2 t dt
0
1 3
    
=a B ,3 − B ,2
2 2
  
1 3
  
 Γ 2 Γ(3) Γ Γ(2)
2  
= a − 
7 7
   
Γ Γ
2 2

√ 1 √ 
 2 π π 
= a − 2 
5 3 1√ 5 3 1√ 
π π
222 222
4a
= .
5
Por simetrı́a, yΓ = 0, luego el centroide es
4a
 
(xΓ , yΓ) = ,0 .
5

2
(3) Para calcular las coordenadas (xD , yD ) del centroide de D, necesitamos la integral:
ZZ
Ix = x dxdy
ZZD
= ρ2 cos θ dρdθ
D′
Z 2π Z a(1+cos θ)
= cos θ dθ ρ2 dρ
0 0
3
a 2π
Z
= cos θ(1 + cos θ)3 dθ
3 0
a3 2π
Z
= cos θ(1 + 3 cos θ + 3 cos2 θ + cos3 θ) dθ
3 0
a3 2π
Z
= (cos θ + 3 cos2 θ + 3 cos3 θ + cos4 θ) dθ
3 0
a3 2π
Z
= (3 cos2 θ + cos4 θ) dθ (los demás términos tienen integral nula)
3 0
4a3 π/2
Z
= (3 cos2 θ + cos4 θ) dθ
3 0
2a3 1 3 1 5
    
= 3B , +B ,
3 2 2 2 2
1 1 1 1 5
        
3 
2a  Γ Γ Γ Γ
= 3 2 2 2 + 2 2 
3  Γ(2) Γ(3) 

5πa3
= .
4
Las coordenadas (xD , yD ) del centroide son:

Ix 2 5πa3 5a
xD = = =
Área(D) 3πa2 4 6
e
yD = 0 , por simetrı́a.

3
AMPLIACIÓN DE CÁLCULO (Curso 2006/2007) Convocatoria de febrero 6.02.07

NOMBRE . . . . . . . . . . . . . . . . . . . . . . . . . APELLIDOS . . . . . . . . . . . . . . . . . . . . . . . . . . . . . . . . . . . . . . . . . . .
Número de matrı́cula. . . . . . . . . . . . . . . . . . . . . . . .

PROBLEMA 3 (3 puntos)
Se consideran el casquete del elipsoide:
x2 y 2 z 2
+ + = 1,
4 4 16
situado en el semiespacio z ≥ 0 y la superficie cónica formada por los segmentos que unen el punto
(0, 0, −8) con el borde del casquete anterior. Sea F(x, y, z) el campo vectorial definido por
z
 
F(x, y, z) := y U (x, y, z) , − x U (x, y, z) , z(x + y + )
2
!3/2
(z + 8)2
donde U (x, y, z) := − x2 − y 2 . Se pide:
16
1) Calcular el flujo del campo F(x, y, z) a través del casquete elipsoidal orientado de forma que la
tercera componente de su vector normal sea positiva. (1 punto)
2) Calcular el flujo del campo F(x, y, z) a través de la superficie cónica orientada de forma que la
tercera componente de su vector normal sea negativa. (1 punto)
3) Sean Ω ∈ R3 el domino limitado por las dos superficies definidas anteriormente, G := (xg , yg , zg )
el centroide de Ω y Σ+ la cara exterior de su frontera. Relacionar el flujo del campo F(x, y, z)
a través de Σ+ con las coordenadas de G y calcularlas. (1 punto)

Respuesta: Se entregará esta hoja y, a lo sumo, una adicional.


En primer lugar, dado que a lo largo del problema hay que calcular flujos del campo vectorial
F(x, y, z), conviene calcular su divergencia para saber qué posibilidades hay a la hora de aplicar el
teorema de Gauss. Su expresión es:
h  i
z
∂ [y U (x, y, z)] ∂ [x U (x, y, z)] ∂ z x+y+ 2
∇ · F(x, y, z) = − + = x+y+z.
∂x ∂y ∂z
Se trata pues de un campo vectorial que n (salvo en el origen) no es solenoidal.
o Nótese que el campo
3 3 (z+8)2 2 2
está definido en R menos el conjunto (x, y, z) ∈ R : 16 − x − y < 0 .
1) En vista de ello, optamos por calcular el flujo pedido parametrizando la
superficie, que se muestra en la figura, y aplicando la definición (pese a que,
como se demuestra en la solución alternativa que se expone más abajo, esta
no es la opción más sencilla).
Sea E + la cara del casquete considerada. Debido a que su proyección sobre
el plano XY es el disco de ecuación cartesiana: x2 + y 2 ≤ 4, utilizamos como
parámetros las coordenadas polares (r, θ) del plano. Entonces, una parametri-
zación, Σ(r, θ) = (x(r, θ), y(r, θ), z(r, θ)), de E + viene dada por las siguientes
ecuaciones paramétricas:
s
r2
x(r, θ) = r cos θ , y(r, θ) = r cos θ , z(r, θ) = 4 1 − , 0 ≤ r ≤ 2, 0 ≤ θ ≤ 2π ,
4
con lo que el vector normal, que debe tener su tercera componente positiva, es:

2 r2 cos θ 2 r2 sin θ
!
Σ(r, θ) Σ(r, θ)
n(r, θ) := × = √ , √ ,r .
∂r ∂θ 4 − r2 4 − r2
Por definición, la expresión del flujo pedido es:
ZZ ZZ
F(x, y, z) · dσ = F(x(r, θ), y(r, θ), z(r, θ)) · n(r, θ) dr dθ ,
E+ [0,2]×[0,2π]

donde h √ i
F(x(r, θ), y(r, θ), z(r, θ)) · n(r, θ) = 2 r 4 − r2 + r 4 − r2 (cos θ + sin θ)
y, por tanto:
ZZ Z 2 Z 2 √  Z 2π 
F(x, y, z) · dσ = 2π 2 r (4 − r2 ) + 2 r2 4 − r2 dr (cos θ + sin θ) dθ .
E+ 0 0 0

Finalmente, la integral en la variable θ es cero, con lo que resulta:


ZZ Z 2
F(x, y, z) · dσ = 4π r (4 − r2 ) = 16π .
E+ 0

1) Alternativa. El casquete elipsoidal no es una superficie cerrada. Sin embargo, es posible


cerrarla considerando otra superficie regular cualquiera que tenga el mismo borde que el casquete.
Como vamos a ver a continuación, la elección del disco de ecuación cartesiana: x2 + y 2 ≤ 4 situado
en el plano XY , resulta especialmente apropiada en este caso. La aplicación del teorema de Gauss a
la superficie cerrada formada por el disco y el casquete elipsoidal da lugar a la expresión:
ZZ ZZ ZZZ
F(x, y, z) · dσ + F(x, y, z) · dσ = (x + y + z) dV ,
E+ C+ D

donde E + es la cara del casquete orientada de forma que la tercera componente de su vector normal
es positiva, C + representa la cara del disco tal que la tercera componente de su vector normal es
negativa y D es el recinto acotado de R3 limitado por la superficie cerrada, que viene dado por:
 s 
 x2 y 2 
D = (x, y, z) ∈ R3 : x2 + y 2 ≤ 4 , 0 ≤ z ≤ 4 1 − − .
 4 4

Como ya se ha mencionado, la elección del disco para obtener una superficie cerrada resulta muy
conveniente. Nótese que la tercera componente del campo sobre el disco es nula (en el disco es z = 0),
mientras que el vector normal al disco tiene sus dos primeras componentes nulas; es decir, el campo
sobre el disco y la normal al disco son perpendiculares, con lo que el flujo es nulo. Ası́ pues:
ZZ ZZ ZZZ
F(x, y, z) · dσ = 0 =⇒ F(x, y, z) · dσ = (x + y + z) dV .
C+ E+ D

Por otra parte, la integral de volumen que aparece en el segundo miembro es, por definición, el
resultado de multiplicar el volumen de D por la suma de las componentes de su centroide. Pero este
dominio es simétrico con respecto a los planos XZ e Y Z y, por tanto, las dos primeras componentes
de su centroide son nulas; es decir:
ZZZ ZZZ
x dV = y dV = 0 ,
D D

con lo cual se obtiene la igualdad:


ZZ ZZZ
F(x, y, z) · dσ = z dV ,
E+ D
según la cual el flujo pedido es igual al producto del volumen de D por la tercera componente de
su centroide. La integral de volumen se puede calcular de forma sencilla utilizando coordenadas
cilı́ndricas: ZZZ ZZZ
z dV = z r drdθ dz ,
D D0
 s 
 r2 
donde r es el jacobiano y D0 = (r, θ, z) ∈ R3 : 0 ≤ r ≤ 2, 0 ≤ θ ≤ 2π , 0 ≤ z ≤ 4 1 − . Por
4
tanto, el flujo pedido es:
q
2
r2
!
ZZZ ZZZ Z 2π Z 2 Z 4 1− r4 Z 2
z dV = z r drdθ dz = dθ dr z r dz = 16π 1− r dr = 16π .
D D0 0 0 0 0 4

Además, aunque no se pide, el centroide de D tiene por coordenadas (0, 0, 23 ), ya que su volumen es
la mitad del volumen del elipsoide de semiejes 2, 2 y 4, que es 32π3
.
2) Unas ecuaciones paramétricas para la superficie cónica que se muestra
en la figura se obtienen construyendo las ecuaciones de todas los segmentos
que unen el punto (0, 0, −8) con el borde del casquete elipsoidal. El borde es la
circunferencia x2 +y 2 = 4 situada en el plano z = 0, con lo que unas ecuaciones
paramétricas de esta curva son:

x(θ) = 2 cos θ , y(θ) = 2 sen θ , z(θ) = 0 , 0 ≤ θ ≤ 2π .

La ecuación del haz de segmentos mencionado es, entonces:


x y z+8
= = = λ, 0 ≤ λ ≤ 1,
x(θ) y(θ) z(θ) + 8

de donde se obtiene la parametrización Σ(θ, λ) = (x(θ, λ), y(θ, λ), z(θ, λ)) para la superficie cónica,
cuyas ecuaciones son:

x(θ, λ) = 2λ cos θ y(θ, λ) = 2λ sen θ z(θ, λ) = −8 + 8λ , 0 ≤ θ ≤ 2π , 0 ≤ λ ≤ 1,

(z + 8)2
y de ellas se deduce fácilmente la ecuación cartesiana de esta superficie, que es: x2 + y 2 = .
16
Sea S + la cara de la superficie que se debe considerar, es decir aquella cuyo vector normal tiene
su tercera componente negativa y que, por tanto, es:
Σ(θ, λ) Σ(θ, λ)
n(θ, λ) := × = (16λ cos θ, 16λ sen θ, −4λ) .
∂θ ∂λ
El flujo se calcula ahora aplicando su definición para lo que, en general, resulta conveniente ver el
valor que el campo toma sobre la superficie. Esto es especialmente adecuado en este caso, puesto que
la función U (x, y, z) definida en el enunciado se anula sobre la superficie cónica. Teniendo esto en
cuenta, el valor del campo sobre ella es:

F(x(θ, λ), y(θ, λ), z(θ, λ)) := (0 , 0 , 16(λ − 1)[λ(cos θ + sen θ) + 2(λ − 1)]) ,

con lo cual

F(x(θ, λ), y(θ, λ), z(θ, λ)) · n(θ, λ) = −64λ2 (λ − 1)[cos θ + sen θ] − 128λ(λ − 1)2

y el flujo pedido es:


1 2π 1 64π
ZZ Z  Z  Z
2
F(x, y, z)·dσ = − 64λ (λ − 1)dλ [cos θ + sen θ]dθ −256π λ(λ−1)2 dλ = − .
S+ 0 0 0 3
2) Alternativas. El teorema de Gauss se puede utilizar aquı́ de forma análoga a como se
describe en la solución alternativa del primer apartado. A la hora de construir la superficie cerrada,
hay dos posibilidades razonables que se pueden utilizar en este caso.
Una de ellas consiste en cerrar la superficie cónica por el disco x2 + y 2 ≤ 4, z = 0, en cuyo caso el
flujo pedido es igual al resultado de multiplicar el volumen del cono limitado por ambas superficies
por la tercera componente de su centroide. Dado que el volumen del cono es un tercio del área de su
base por su altura, es decir: 32π
3
, resulta que el centroide del cono es (0, 0, −2) (pero este resultado
no se pide).
La otra es cerrar la superficie cónica utilizando el casquete elipsoidal. Esto último constituye el
objetivo del tercer apartado de este problema.

3) Dado que el centroide de un sólido de R3 se expresa en términos de una


integral de volumen, en este apartado se nos pide relacionar un flujo con una
integral triple. Un resultado que, en caso de que se cumplan las hipótesis,
permite relacionar estos dos tipos de integrales es el teorema de Gauss. Estas
hipótesis se cumplen claramente en las condiciones establecidas en el enun-
ciado, puesto que el campo vectorial es de clase C 1 en Ω y la superficie que
encierra el sólido (ver la figura) es regular a trozos. Su aplicación da lugar a la
relación:
ZZ ZZZ ZZZ
F(x, y, z) · dσ = ∇ · F(x, y, z) = (x + y + z)dV .
Σ+ Ω Ω

Puesto que Ω es simétrico con respecto a los planos XZ e Y Z, el mismo


argumento utilizado en la solución alternativa al primer apartado nos permite
concluir que las dos primeras componentes de su centroide son nulas; es decir:
ZZZ ZZZ
xdV = ydV = 0 .
Ω Ω

Por tanto, ZZ ZZZ


F(x, y, z) · dσ = z dV = zg VΩ ,
Σ+ Ω
donde VΩ es el volumen de Ω que se obtiene como la suma del volumen de la parte correspondiente
al elipsoide más el volumen del cono, es decir: VΩ = 32π
3
+ 32π
3
= 64π
3
. Ahora bien, obviamente:
ZZ ZZ ZZ
F(x, y, z) · dσ = F(x, y, z) · dσ + F(x, y, z) · dσ ,
Σ+ E+ S+

donde E + es la cara del casquete elipsoidal considerado en el primer apartado, S + es la cara de la


superficie cónica considerada en el segundo y los flujos de F(x, y, z) a través de ellas se han calculado
en esos apartados. Ası́ pues:
1 3 64π 1
Z Z ZZ   
zg = F(x, y, z) · dσ + F(x, y, z) · dσ = 16π − =−
VΩ E+ S+ 64π 3 4
1
y el centroide de Ω es G = (0, 0, − ).
4
NOTA: Caso de no recordar la fórmula del volumen de un cono, este se puede calcular conside-
z+8
rando que las secciones por planos z = cte del mismo, son discos de radio . El área de estos
2 4
z+8

discos es π y el volumen del cono se puede calcular mediante la expresión:
4
2
0 z+8 32π
Z 
π dz = .
−8 4 3
AMPLIACIÓN DE CÁLCULO (Curso 2006/2007) Convocatoria de junio 12.06.07

PROBLEMA 1 (4 puntos) Este problema consta de dos ejercicios independientes.

1. (2 puntos) Determinar los valores reales de a para los cuales converge


Z ∞ e−x − eax
dx.
0 x

Solución:
En primer lugar calculemos el lı́mite del integrando en el origen, usando la regla de L’Hôpital:

e−x − eax
lı́m = −(1 + a)
x→0 x
Como el lı́mite existe para todo valor de a, el origen es una singularidad evitable y la integral
solo es impropia en infinito.
Calculemos también el lı́mite del integrando en el infinito:

e−x − eax
(
0 a≤0
lı́m =
x→∞ x −∞ a > 0

Es condición necesaria de convergencia que el lı́mite del integrando en el infinito sea nulo u
oscile, por tanto a ≤ 0. Dado que el origen es una singularidad evitable, vamos a descomponer
la integral:
Z ∞ e−x − eax Z 1 −x
e − eax Z ∞ −x
e − eax
dx = dx + dx = I0 + I1 (a)
0 x 0 x 1 x
I0 es propia, nos ocupamos de la convergencia de I1 (a). Utilicemos el siguiente criterio de
comparación:
Sean f y gZ dos funciones continuas en [ a, ∞), con g no negativa; si ∀x ≥ a |f (x)| ≤ g(x)
∞ Z ∞
entonces: ( g converge ⇒ f converge absolutamente).
a a
Z ∞
Sabemos que ebx dx es convergente para b < 0; entonces
0

e−x − eax

≤ e−x + eax , x ≥ 1


x

Luego para a < 0 la integral I1 (a) es convergente. Por último, para a = 0 descomponemos la
integral como sigue:
Z ∞ e−x Z ∞
1
I1 (0) = dx − dx,
1 x 1 x
el primer sumando es convergente y el segundo divergente, luego la integral es divergente.
En conclusión, la integral del enunciado converge solo si a < 0.
2. (2 puntos) Determinar los valores reales de a y b para los cuales converge
Z ∞ ta
I(a, b) = dt
0 (4 + t2 )b

y calcular explı́citamente I(2, 3).


Indicación: Relacionar la integral con una beta de Euler.

Solución:
Una de las expresiones de la función beta de Euler es
Z ∞ xp−1
β(p, q) = dx,
0 (1 + x)p+q

convergente para p, q > 0. Hagamos los cambios adecuados para√ expresar I(a, b) como una beta;
2 2
dado que 4 + t = 4(1 + (t/2) ) el cambio natural es t = 2 x que transforma biyectivamente
la semirrecta (0, ∞) en sı́ misma y no cambia los lı́mites. Se tiene entonces:
a−1
2a Z ∞ x 2 2a a+1 a+1
 
I(a, b) = b dx = b β ,b − .
4 0 (1 + x)b 4 2 2

Para que sea convergente tiene que ser


a+1 a+1 a+1
 
p= >0 ∧ q =b− >0 ⇐⇒ b > > 0.
2 2 2
Por último,
 2
3
2 2
3 3 1 Γ 2
 
I(2, 3) = 3 β , = ,
4 2 2 16 Γ(3)


3 1 1 1 π
    
Γ =Γ 1+ = Γ = ⇒
2 2 2 2 2

π
I(2, 3) = .
128

Gabriela Sansigre.
AMPLIACIÓN DE CÁLCULO (Curso 2006/2007) Convocatoria de junio 12.06.07

NOMBRE . . . . . . . . . . . . . . . . . . . . . . . . . APELLIDOS . . . . . . . . . . . . . . . . . . . . . . . . . . . . . . . . . . . . . . . . . . .
Número de matrı́cula. . . . . . . . . . . . . . . . . . . . . . . .

PROBLEMA 2 (3 puntos)
En R2 se consideran el campo vectorial definido por:

F(x, y) := (x2 + y 2 )2 (2x − 3, 2y + 1)

y las circunferencias de ecuaciones cartesianas x2 + y 2 = 2 y (x − 1)2 + y 2 = 1, ambas recorridas en


sentido positivo.
Sean, Γ1 el arco de la circunferencia orientada centrada en el origen contenido en el semiplano
x ≥ 1 y Γ2 el arco de la otra circunferencia orientada contenido en el semiplano x ≤ 1. Se pide:
1) Calcular la circulación del campo F(x, y) a lo largo del arco orientado Γ1 . (0,75 puntos)
2) Sea E ⊂ R2 el dominio acotado limitado por el arco Γ2 y por la recta x = 1. Aplicando
adecuadamente el teorema de Green en E, obtener la circulación del campo F(x, y) sobre el
arco orientado Γ2 . (1 punto)
3) Sea D ⊂ R2 el domino acotado limitado por las curvas Γ1 y Γ2 , sobre el que se considera
definida una densidad que en cada punto es igual al cuadrado de la distancia del punto al
origen. Relacionar la circulación del campo F(x, y) a lo largo de la frontera de D recorrida en
sentido positivo con las coordenadas del centro de gravedad de D (con la densidad dada) y
calcularlas.
1
(Dato: La masa total de D es MD = (5π − 8)). (1,25 puntos)
4

Respuesta: Se entregará esta hoja y, a lo sumo, una adicional.

1) Puesto que la curva Γ1 es el arco de la circunferencia de ecuación cartesiana x2 +y2 = 2 (orientada


positivamente) situado en el semiplano x ≥ 1, unas ecuaciones paramétricas de la misma son:
√ √ π π
x(t) = 2 cos t , y(t) = 2 sin t , t ∈ [− , ] ,
4 4
donde el rango de variación del parámetro t (que es el ángulo polar) se determina sin más que
obtener los ángulos polares de los dos puntos en los que la circunferencia corta a la recta x = 1. Estos
puntos son (1, −1), (1, 1) y sun ángulos polares − π4 , π4 , respectivamente. El sentido de recorrido se
corresponde con la orientación positiva de la circunferencia, por tanto Γ1 se recorre desde (1, −1)
hasta (1, 1), por lo que t ∈ [− π4 , π4 ].
Para calcular la circulación utilizamos su definición:
Z Z π/4 Ã !
x0 (t)
F · ds = F(x(t), y(t)) · dt ,
Γ1 −π/4 y 0 (t)

donde:
à !
³ √
à √ !
x0 (t) √ ´ −√ 2 sin t √
F(x(t), y(t))· = 4 2 2 cos t − 3 , 2 2 sin t + 1 · = 4 2 (cos(t) + 3 sin(t)) .
y 0 (t) 2 cos t
Por tanto: Z Z π/4 √ √ Z π/4
F · ds = 4 2 (cos t + 3 sin t) dt = 4 2 cos t dt = 8 .
Γ1 −π/4 −π/4
2) Si representamos el campo F en la forma F(x, y) = (L(x, y), M (x, y)), sus
dos componentes L y M (que son polinomios en dos variables) son funciones
1

de clase C ∞ . Por otra parte, el dominio E, que se representa en la figura, es 0.5

simplemente conexo. Por tanto, se puede aplicar el teorema de Green al campo E


y dominio dados, lo que establece la igualdad: 0

Z ZZ Ã !
∂M ∂L -0.5

F · ds = − dxdy ,
∂E E ∂x ∂y -1

-0.5 0 0.5 1 1.5


donde ∂E representa la frontera de E recorrida en sentido positivo. La curva
∂E está formada por el arco orientado Γ2 y por el segmento orientado de la recta x = 1 comprendido
entres los puntos (−1, 1) y (1, 1), recorrido desde el primero hasta el segundo. Sea S este segmento
orientado. De la igualdad anterior se deduce entonces:
Z ZZ Ã ! Z
∂M ∂L
F · ds = − dxdy − F · ds
Γ2 E ∂x ∂y S

lo que permite obtner la circulación de F sobre Γ2 calculando las dos integrales del segundo miembro.

• Circulación sobre el segmento S. Unas ecuaciones paramétricas de S son:

x(t) = 1 , y(t) = t , t ∈ [−1, 1] .

Por tanto:
Z Z 1 Ã ! Z 1
x0 (t) ³ ´2
F · ds = F(x(t), y(t)) · dt = (1 + 2 t) 1 + t2 dt
S −1 y 0 (t) −1
Z 1³ Ã !¯1
´ t5 2t3 ¯ 56
2 2 ¯
= 2 1+t dt = 2 t + + ¯ = .
0 5 3 ¯ 15
0

• Integral doble. Puesto que:

∂M ∂L ³ ´
− = 4 x2 + y 2 (x + 3 y) ,
∂x ∂y

la integral que hay que calcular es:


ZZ Ã ! ZZ ³ ´ ZZ ³ ´
∂M ∂L 2 2
− dxdy = 4x x + y dxdy + 12 y x2 + y 2 dxdy .
E ∂x ∂y E E

Ahora bien, dado que E es simétrico con respecto al eje de abscisas, la segunda integral del
segundo miembro es cero, puesto que el integrando es impar en y. Es decir:
ZZ Ã ! ZZ ³ ´
∂M ∂L
− dxdy = 4x x2 + y 2 dxdy .
E ∂x ∂y E

Para calcular esta integral, tenemos en cuenta que el dominio E es la mitad de la circunferencia
de ecuación cartesiana (x − 1)2 + y 2 = 1 situada a la izquierda de la recta x = 1, con lo que,
en coordenadas polares trasladadas: x = 1 + r cos θ, y = r sin θ, el correspondiente dominio
transformado se puede describir por las ecuaciones:
½ ¾
0 π 3π 2
E = (r, θ) ∈ R , ≤ θ ≤ ,0≤r≤1 .
2 2
Entonces, utilizando las mencionadas coordenadas y teniendo en cuenta que el jacobiano de la
transformación es r:
ZZ Ã ! ZZ ³ ´
∂M ∂L
− dxdy = 4 (1 + r cos θ) 1 + r2 + 2 r cos θ r drdθ
E ∂x ∂y E0
Z 1 Z 3π/2 n o
= dr 4r(1 + r2 ) + 4r2 (3 + r2 ) cos θ + 8r3 cos2 θ dθ
0 π/2
Z 1 Z 1 µZ 1 ¶ ÃZ 3π/2 !
2 2 2 3 2
= 4π r(1 + r )dr − 8 r (3 + r )dr + 8 r dr cos θ dθ
0 0 0 π/2
48
= 4π − ,
5
donde: Z 3π/2 Z 3π/2
2 1 π
cos θdθ = (1 + cos 2θ)dθ = .
π/2 π/2 2 2
Finalmente, la circulación sobre el arco orientado Γ2 es:
Z ZZ Ã ! Z · ¸
∂M ∂L 48 56 40
F · ds = − dxdy − F · ds = 4π − − = 4π − ,
Γ2 E ∂x ∂y S 5 15 3

3) Dado que el centro de gravedad de un recinto de R2 se expresa en términos de integrales dobles,


en este apartado se nos pide relacionar una circulación con dos integrales do-
bles. Un resultado que, en caso de que se cumplan las hipótesis, permite re-
lacionar estos dos tipos de integrales es el teorema de Green en el plano, ya 1

utilizado en el apartado anterior. Las hipótesis se cumplen claramente en las


condiciones establecidas en el enunciado, puesto que el campo vectorial es de
0.5

clase C ∞ en D y el dominio D, que se muestra en la figura, es simplemente 0


D
conexo. Su aplicación da lugar a la relación:
ZZ Ã ! -0.5
Z
∂M ∂L
F · ds = − dxdy .
∂x ∂y
-1
∂D D
-0.5 0 0.5 1 1.5

En esta igualdad ∂D es la frontera de D recorrida en sentido positivo que está formada por los arcos
orientados Γ1 y Γ2 , de forma que se tiene:
Z Z Z
40 16
F · ds = F · ds + F · ds = 8 + 4π − = 4π − ,
∂D Γ1 Γ2 3 3
donde las circulaciones del campo F sobre los arcos orientados Γ1 y Γ2 se han calculado en los dos
apartados anteriores.
En lo que se refiere al segundo miembro, su expresión es:
ZZ Ã ! ZZ ZZ
∂M ∂L 2 2
− dxdy , = 4 x(x + y ) dxdy + 12 y(x2 + y 2 ) dxdy ,
D ∂x ∂y D D

con lo que, teniendo en cuenta la definición de las coordenadas del centro de gravedad, (Xg (D), Yg (D)),
de D con la densidad dada:
1 ZZ 1 ZZ
Xg (D) := x(x2 + y 2 ) dxdy , Yg (D) := y(x2 + y 2 ) dxdy ,
MD D MD D
resulta: ZZ Ã !
∂M ∂L
− dxdy = 4MD Xg (D) + 12MD Yg (D) ,
D ∂x ∂y
y, por tanto: Z
16
F · ds = 4MD Xg (D) + 12MD Yg (D) = 4π − ,
∂D 3
que es la relación pedida.
Para calcular las dos coordenadas basta tener en cuenta la simetrı́a de D con respecto al eje de
abscisas, con lo que Yg (D) = 0. Como la masa total, MD , de D es un dato que se proporciona en el
enunciado, de la anterior expresión se deduce el valor de Xg (D), que es:
µ ¶
1 16 4 (3π − 4)
Xg (D) = 4π − = .
4MD 3 3 (5π − 8)
AMPLIACIÓN DE CÁLCULO (Curso 2006/2007) Convocatoria de junio 12.06.07

NOMBRE . . . . . . . . . . . . . . . . . . . . . . . APELLIDOS . . . . . . . . . . . . . . . . . . . . . . . . . . . . . . . . . . . . . . . .
Número de matrı́cula . . . . . . . . . . . . . . . . . . . . .

PROBLEMA 3 (3 puntos)

1. Sea la región B del plano Y Z definida por 0 ≤ z ≤ y 2 , y ∈ [1, 2]. Se considera el sólido Ω
engendrado cuando B gira alrededor del eje y. En cada punto de Ω la densidad es proporcional
al producto de las distancias de dicho punto a los planos coordenados. Se pide el momento de
inercia de Ω respecto del eje y. (1 pto)

2. Sea Γ la curva sobre el plano Y Z definida por z = y 2 , y ∈ [1, 2]. Sea Σ la superficie de
revolución engendrada cuando Γ gira alrededor del eje y. Sobre Σ hay definida una distribución
superficial de masa de forma que la densidad en cada punto P de la superficie es proporcional
a la distancia del origen al plano tangente a Σ en el punto P . Se pide calcular la masa de Σ.
(2 ptos.)

Respuesta: Se entregará esta hoja y, a lo sumo, una adicional.


Solución: 1. La densidad es
ρ(x, y, z) = K |x| |z| |y|
por lo que
Z Z Z Z Z Z
2 2
I = ρ(x, y, z)(x + z )dxdydz = K |x| |z| |y| (x2 + z 2 )dxdydz =
Z ZΩ Z Ω

= 4K xzy(x2 + z 2 )dxdydz
Ω0

donde Ω0 es la porción de Ω contenida en el primer octante. Se ha usado que Ω es simétrico


respecto de los planos x = 0 y z = 0 y que la función subintegral es par en x y par en z.
Para simplificar el cálculo de la integral, efectuamos el siguiente cambio a coordenadas cilı́ndri-
cas (x, y, z) = φ(ρ, θ, y)
x = ρsenθ , y = y , z = ρ cos θ
y entonces la imagen inversa de Ω0 es
n π √ o
B := φ−1 (Ω0 ) = (ρ, θ, y) : 0 ≤ ρ ≤ y 2 , θ ∈ [0, ], y ∈ [1, 2]
2
Por tanto
Z Z Z
I = 4K (ρsenθ) (ρ cos θ) yρ2 ρdθdρdy =
B
Z π Z √ ÃZ 2 !
2 y
2
= 4K cos θsenθdθ y ρ5 dρ dy =
0 1 0
Z √
2
11 1 1 1 ¡ 7 ¢ 127K
= 4K y 13 dy = 4K 2 −1 =
26 1 2 6 14 42
2. Parametrizamos la superficie Σ en la forma (x, y, z) = α(θ, y)

x = α1 (θ, y) = y 2 senθ

y = α2 (θ, y) = y y ∈ [1, 2], θ ∈ [0, 2π]
z = α3 (θ, y) = y 2 cos θ

y operando se obtiene ° °
° ∂α ∂α ° p
° (θ, y) × (θ, y)° = y 2 1 + 4y 2
° ∂θ ∂y °
Para hallar la ecuación del plano tangente a Σ en (x, y, z) obtendremos primero un vector
perpendicular a la superficie en dicho punto. Para ello es conveniente trabajar con la ecuación
cartesiana de Σ, que es x2 + z 2 = y 4 o bien

F (x, y, z) = x2 − y 4 + z 2 = 0

Se sabe que el gradiente de F en (x, y, z)


¡ ¢
∇F (x, y, z) = 2x, −4y 3 , 2z

o bien, el vector paralelo (x, −2y 3 , z) , es perpendicular a Σ en (x, y, z). Por ello, el plano
tangente a Σ en el punto (x, y, z) tiene ecuación

x (X − x) − 2y 3 (Y − y) + z(Z − z) = 0

o bien ¡ ¢
xX − 2y 3 Y + zZ − x2 − 2y 4 + z 2 = 0
que, puesto que x2 + z 2 = y 4 sobre la superficie, se puede simplificar a

xX − 2y 3 Y + zZ + y 4 = 0

Por lo tanto, la densidad sobre la superficie tiene la forma

|y 4 | y4 y2
ρΣ (x, y, z) = K p =K p = Kp
x2 + 4y 6 + z 2 y 2 1 + 4y 2 1 + 4y 2

y por lo tanto
Z Z
y2
MΣ = ρΣ (x, y, z)dσ = K p dσ =
Σ Σ 1 + 4y 2
Z 2π Z √2 Z √2
y2 2
p 2Kπ ³ √ ´
= K p y 1 + 4y 2 dydθ = K2π y 4 dy = 4 2−1
0 1 1 + 4y 2 1 5
AMPLIACIÓN DE CÁLCULO (Curso 2006/2007) Convocatoria de septiembre 18.09.07

PROBLEMA 1 (3 puntos)

Dada la integral impropia Z ∞ sen ax


F (a) = dx
0 x
se pide:

1. Determinar el conjunto de valores reales de a para los cuales la integral F (a) converge.
2. Enunciar el teorema de derivabilidad de integrales impropias dependientes de un parámetro. Estudiar
si dicho teorema puede aplicarse a la función F .
3. Sabiendo que F (1) = π/2, calcular F (a) allá donde exista.
4. Basándose en los apartados anteriores,

a) ¿Es F uniformemente convergente en todo su dominio de definición?


b) ¿Es F derivable en algún intervalo?

Respuesta: (Se entregará esta hoja y, a lo sumo, una adicional.)

1. En primer lugar, observamos que la integral no es impropia en el origen: en ese punto el integrando
presenta una singularidad evitable ya que lı́m x→0 sen(ax)
x = a y este lı́mite existe para cada valor de a.
Ası́ pues, la integral sólo es impropia en infinito, y allı́ estudiamos su convergencia.
R∞
Para a = 0 la integral converge, pues obviamente F (0) = 0 0 dx = 0.
Para a 6= 0, aplicaremos el Criterio de Abel-Dirichlet:
“Si f, g : [b, ∞) → R son funciones de clase C 1 tales que bx f es acotada en [b, ∞), g es decreciente,
R
R∞
y lı́mx→∞ g (x) = 0, entonces la integral impropia b f (x) g (x) dx es convergente”.
En este caso, tomando, por ejemplo, b = 1, g (x) = 1/x cumple las hipótesis del teorema, y una
primitiva de f (x) = sen (ax) está acotada en [1, ∞) para cada valor a 6= 0 :
x x cos (a) − cos (ax)
Z Z
≤ 2

f (t) dt = sen (at) dt =


1 1 a |a|
luego F (a) converge para todo a ∈ R.
Alternativamente: También puede estudiarse la convergencia en infinito integrando por partes
B cos ax x=B 1 B B

sen ax cos ax 1 cos aB 1 cos ax
Z Z   Z
dx = − − 2
dx = cos a − − dx
1 x ax x=1 a 1 x a B a 1 x2
y haciendo tender B a infinito, el lı́mite existe:
Z ∞ sen ax cos a 1
Z ∞ cos ax
dx = − dx
1 x a a 1 x2
R∞ |cos ax| R∞ 1
pues la última integral impropia converge (absolutamente): 1 x2
dx ≤ 1 x2
dx < ∞.
2. Enunciemos el teorema pedido:
Teorema: Se considera un intervalo abierto I ⊆ R, y la integral impropia
Z ∞
F (a) = f (a, x)dx.
b

Si se verifica:
F (a0 ) converge para algún valor a0 ∈ I,
∂f
f (a, x) y ∂a (a, x) son continuas en I × [b, ∞)
y la integral impropia Z ∞ ∂f
(a, x)dx
b ∂a
converge uniformemente en I,

entonces F es convergente y derivable en I, y su derivada es


Z ∞ ∂f
F 0 (a) = (a, x)dx ∀a ∈ I.
b ∂a
Veamos si se verifican las tres hipótesis del teorema, tomando b = 0, para algún intervalo I ⊆ R:

F converge en a0 = 0, pues sabemos que F (0) = 0 (y el apartado 3 afirma que F (1) converge);
f (a, x) = sen(ax)
x es continua en I ×(0, ∞), y puede extenderse por continuidad en x = 0 definiendo
f (a, 0) = lı́mx→0 sen(ax)
x = a.
∂ sen ax

Por otro lado, ∂a x = cos ax que es una función continua para (a, x) en I × [0, ∞)
La integral impropia de la derivada es
Z ∞ ∂f
Z ∞
(a, x)dx = cos(ax)dx que DIVERGE PARA TODO a.
0 ∂a 0
R∞
En efecto, para a = 0 se tiene 0 dx = ∞; para a 6= 0, aplicamos la definición de convergencia
de integral impropia:
Z B 1
lı́m cos(ax)dx = lı́m sen (aB) NO EXISTE
B→∞ 0 a B→∞
luego dicha integral impropia no converge para ningún valor de a. Entonces, no tiene sentido
estudiar su convergencia uniforme.

Por ello, se deduce que no puede aplicarse dicho teorema de derivación bajo el signo de la
integral impropia F.
3. Es el ejercicio 1.7, apartado 1, de la colección de ejercicios de la asignatura: Realizamos,
para a > 0, el cambio de variable ax = y que no varı́a los lı́mites de integración:
Z ∞ sen ax
Z ∞ sen y 1
Z ∞ sen y π
F (a) = dx = dy = dy = F (1) = .
0 x 0 y/a a 0 y 2
Además, F es una función impar: F (−a) = −F (a) , ası́ que para a < 0, F (a) = − π2 .
Por tanto, se trata de una función constante a trozos, definida en R de la siguiente forma:
(
0 si a = 0
F (a) = π .
2 signo (a) si a 6= 0

4. a) Utilizaremos el teorema de continuidad de funciones definidas como integrales impropias depen-


dientes de un parámetro. El integrando de F es una función continua en R × [0, ∞) (o que se puede
extender por continuidad); si además F convergiera uniformemente en su dominio de definición
R, entonces F serı́a también continua en R. Pero no lo es, pues obviamente F es discontinua en
0; ası́ pues, F no converge uniformemente en R.
b) De la expresión explı́cita de F , se deduce que es derivable en cualquier intervalo que no
contenga al origen (es decir, allá donde es constante). Por ejemplo, en (0, ∞) y en (−∞, 0) .
OBSERVACIÓN: Nótese que, aunque F sea derivable en un intervalo, no siempre puede
derivarse bajo el signo integral. Aún más, el hecho de no poder aplicar ese teorema de
derivación no indica nada acerca de la derivabilidad o no derivabilidad de F .
AMPLIACIÓN DE CÁLCULO (Curso 2006/2007) Convocatoria de septiembre 18.09.07

NOMBRE . . . . . . . . . . . . . . . . . . . . . . . . . APELLIDOS . . . . . . . . . . . . . . . . . . . . . . . . . . . . . . . . . . . . . . . . . . .
Número de matrı́cula. . . . . . . . . . . . . . . . . . . . . . . .

PROBLEMA 2 (3 puntos)
En R3 se considera la superficie cónica Σ formada por los segmentos de recta que pasan por el
punto P ≡ (0, 0, 1) y se apoyan en la curva de ecuaciones
( )
(x − 1)2 + y 2 = 1
z=0

Se pide:
1) (1 pto) Calcular la integral
Z q
I= x2 + (z − 1)2 dσ
Σ

2) (2 ptos) Sea Ω el sólido homogéneo limitado por el plano z = 0 y la superficie Σ. Calcular el


centro de gravedad de Ω.

Respuesta: Se entregará esta hoja y, a lo sumo, una adicional.


Solución:
1) En primer lugar calcularemos unas ecuaciones paramétricas (x, y, z) = φ(t, λ) de Σ, parametrizan-
do los segmentos que unen P con la curva. Esta última tiene ecuaciones

x = 1 + cos t
y = sent ; t ∈ [0, 2π]
z = 0

por lo que Σ se puede parametrizar en la forma

(x, y, z) = φ(t, λ) = (0, 0, 1) + λ [(1 + cos t, sent, 0) − (0, 0, 1)] ; t ∈ [0, 2π], λ ∈ [0, 1]

es decir,

x = λ(1 + cos t)
y = λsent ; t ∈ [0, 2π], λ ∈ [0, 1]
z = 1−λ

Ahora, operando


∂φ ∂φ
× = k−λ (cos t, sent, cos t + 1)k = λ 2 + 2 cos t + cos2 t


∂t ∂λ
Por ello
Z q
I = x2 + (z − 1)2 dσ =
Σ
Z Z q √
= (λ(1 + cos t))2 + λ2 λ 2 + 2 cos t + cos2 tdtdλ =
[0,2π]×[0,1]
Z Z   Z 1 Z 2π  
2 2 2
= λ 2 + 2 cos t + cos t dtdλ = λ dλ 2 + 2 cos t + cos2 t dt =
[0,2π]×[0,1] 0 0
Z 2π
1 5π
 
= 2 ∗ 2π + 0 + cos2 tdt =
3 0 3
2) Por definición
R R R
Ω xdxdydz ydxdydz
Ω Ωzdxdydz
xG = ; yG = ; zG =
V ol(Ω) V ol(Ω) V ol(Ω)
Por simetrı́a yG = 0, lo cual también se puede deducir sin más que darse cuenta
R
que Ω es simétrico
respecto del plano y = 0 y la función f (x, y, z) = y es impar en y, con lo que Ω ydxdydz = 0. Como
Ω es un cono su volumen es un tercio del área de la base por la altura. Por ello
π
V ol(Ω) =
3
Calculemos el volumen también integrando. Ω es tal que su intersección con cada plano z = h es un
cı́rculo que denotamos Ch . Por ello una buena elección para integrar es integrar primero en x e y y
luego en z. Por tanto
Z Z 1 Z Z  Z 1
V ol(Ω) = dxdydz = dxdy dz = Área(Cz )dz
Ω 0 Cz 0

Para calcular Área(Cz ) basta con determinar el radio rz de Cz . Para ello se puede proceder geométri-
camente por semejanza de triángulos, que implica
2rz 1−z
=
2 1
de donde rz = 1 − z. Calculemos rz de otra forma. Para ello calcularemos la ecuación cartesiana de
Σ, lo cual se puede hacer eliminando t y λ de las ecuaciones paramétricas. Se obtiene

(x − 1 + z)2 + y 2 = (1 − z)2

con lo que, de nuevo, se obtiene que rz = 1 − z. Por tanto


Z 1 Z 1 π
V ol(Ω) = Área(Cz )dz = π(1 − z)2 dz =
0 0 3
Análogamente,
Z Z 1 Z Z  Z 1 Z 1
zdxdydz = z dxdy dz = z Área(Cz )dz = π z(1 − z)2 dz =
Ω 0 Ch 0 0
Z 1 π
= π (z − 2z 2 + z 3 )dz =
0 12
con lo que
π
12 1
zG = π =
3
4
Este resultado también se podı́a haber obtenido directamente recordando que en un cualquier cono
homogéneo el centro de gravedad está situado a una distancia de la base igual a un cuarto de la
altura del cono.
Una vez determinados yG y zG la posición de xG se puede determinar directamente. En efecto,
para z = 1/4 se tiene rz = 1 − 1/4 = 3/4, con lo que xG = 3/4.
Calculemos xG integrando. Si Ω∗ denota la porción del cono correspondiente a y ≥ 0, y Ch∗ la
porción de Ch correspondiente a y ≥ 0 tenemos, integrando primero en x luego en y y finalmente en
z
!
Z Z Z 1 Z Z
H : = xdxdydz = 2 xdxdydz = 2 xdxdy dz =
Ω Ω∗ 0 Cz∗
  √  
Z 1 Z 1−z Z 1−z+ (1−z)2 −y 2
= 2   √ xdx dy  dz =
0 0 1−z− (1−z)2 −y 2
" 2 2 # !
1 Z 1 Z 1−z q  q
2 2 2 2
= 2 1 − z + (1 − z) − y − 1 − z − (1 − z) − y dy =
2 0 0
Z 1 Z 1−z q  Z 1 Z 1−z q 
2 2
= 4 (1 − z) (1 − z) − y 2 dy dz = 4 (1 − z) (1 − z) − y 2 dy dz
0 0 0 0

y efectuando el cambio y = (1 − z)u


Z 1 Z 1 q 
2 2
H = 4 (1 − z) (1 − z) − (1 − z) u2 (1 − z) du dz =
0 0
Z 1 Z 1 √ 
= 4 (1 − z) |1 − z| 1 − u2 du dz
0 0
Z 1 Z 1 √ 
1
Z 1 √ 
= 4 (1 − z)3 dz 1 − u2 du = 4 1 − u2 du
0 0 4 0

donde se ha usado que 1 − z ≥ 0 para z ∈ [0, 1]. Tras efectuar el cambio u =senw, se obtiene
π
H=
4
con lo que
π
4 3
xG = π =
3
4
AMPLIACIÓN DE CÁLCULO (Curso 2006/2007) Convocatoria de septiembre 18.09.07

PROBLEMA 3 (4 puntos)
Este problema tiene por objeto mostrar un campo solenoidal en un dominio simplemente conexo de R3
que no es un rotacional.

1. En una esfera centrada en el origen y de radio R, calcúlese el área de un casquete esférico Qh , de


altura h (0 < h < 2R) y borde circular Ch .

Ch

Figura 1: Casquete Qh

2. Calcúlese el valor del parámetro α para que el campo F(r) = krkα r sea solenoidal en

Ω = {r ∈ R3 , 1 ≤ krk ≤ 2}.

3. Supóngase Qh ⊂ Ω (el casquete del primer


Z apartado con 1 < R < 2) y orientado según el vector
normal exterior a la esfera. Calcúlese F con el valor de α que hace F solenoidal. Calcúlese
Z Qh

asimismo lı́m F.
h→2R Qh
Z
4. Sea G un campo cualquiera de clase 1 en Ω. Calcúlese lı́m G. Conclúyase que F no admite
h→2R Ch
potencial vector en Ω.
Indicación: Úsese el teorema de acotación de la integral de lı́nea. Supóngase que G es un potencial vector de F,
aplı́quese el teorema de Stokes y lléguese a contradicción.

Respuesta:

1. Calculemos el área del casquete por dos procedimientos:

a) Mediante la parametrización natural en función de los ángulos polar y acimutal:

Φ(ϕ, θ) = (R sen ϕ cos θ, R sen ϕ sen θ, R cos ϕ), (ϕ, θ) ∈ [0, ϕ0 ] × [0, 2π]

con ϕ0 tal que cos ϕ0 = (R − h)/R. El vector normal asociado es

∂Φ ∂Φ
× = R sen ϕΦ(ϕ, θ)
∂ϕ ∂θ

y su norma k∂Φ/∂ϕ × ∂Φ/∂θk = R2 sen ϕ, por lo que el área del casquete será:
ZZ Z ϕ0
A(Qh ) = 2
R sen ϕ dϕ dθ = 2πR 2
sen ϕ dϕ = 2πR2 (− cos ϕ|ϕ0 0 ) = 2πRh.
[0,ϕ0 ]×[0,2π] 0
b) Considerando el casquete como superficie de revolución de la función no negativa
f : [R − h, R] → R

f (x) = R2 − x2
alrededor del eje OX. Sabemos que en este caso el área viene dada por
Z R q
A(Qh ) = 2π f (x) 1 + f 0 (x)2 dx
R−h
q
Cuenta habida que f (x) 1 + f 0 (x)2 = R se concluye que el área es A(Qh ) = 2πRh.
Nota: El cálculo del área de un casquete esférico está propuesto en el ejercicio 7.5.(a).

2. El campo F será solenoidal en Ω si su divergencia es nula. Recordemos la fórmula para calcular la


divergencia de un campo que sea producto de un campo escalar g por un campo vectorial G:
div(gG) = ∇g · G + g div G.
En nuestro caso, g(r) = krkα y G(r) = r. Calculemos la derivada parcial de g respecto de x:
∂(x2 + y 2 + z 2 )α/2 α
= (x2 + y 2 + z 2 )(α/2)−1 2x = αkrkα−2 x
∂x 2
Análogamente, ∂g/∂y = αkrk y, ∂g/∂z = αkrkα−2 z, ası́ pues ∇g(r) = αkrkα−2 r y div G(r) = 3,
α−2

luego:
div F(r) = αkrkα−2 r · r + 3krkα = 0 ⇐⇒ (α + 3)krkα = 0 ⇐⇒ α = −3.
r
Observemos que el campo F(r) = está bien definido en Ω, ya que la única singularidad de F es
krk3
el origen que no pertenece a Ω. (Para más detalles sobre campos centrales consúltese el ejercicio 6.7.)
3. El vector normal unitario exterior a la esfera es r/krk por lo tanto:
Z Z Z
r r 1 1 2πh
F= 3
· dσ = 2
dσ = 2 A(Qh ) = .
Qh Qh krk krk Qh krk R R
Z
2πh
Por último, lı́m F = lı́m = 4π.
h→2R Qh h→2R R

4. Sea G un campo cualquiera de clase 1 en Ω; sea M una cota de G en Ω: su existencia está asegurada
por la continuidad de G en el compacto Ω, y Lh la longitud de la circunferencia Ch ; según el teorema
de acotación de la integral de lı́nea se tiene:
¯Z ¯
¯ ¯
¯ G¯¯ ≤ M Lh ,
¯
Ch

cuando h tiende a 2R el casquete tiende a la esfera y su borde a un punto, ası́


Z que la longitud Lh
tiende a cero, además la constante M es independiente de R, por lo que lı́m G = 0.
h→2R Ch

Supongamos ahora que F admite un potencial vector G de clase 1 en Ω (esto es, F = rot G); estamos
en condiciones de aplicar el teorema de Stokes al campo G sobre la superficie Qh , suponiendo el
casquete y su borde Ch orientados coherentemente:
Z Z Z
F= rot G = G
Qh Qh Ch

Si tomamos lı́mite cuando h tiende a 2R en los extremos de la igualdad anterior tenemos


Z Z
4π = lı́m F = lı́m G=0
h→2R Qh h→2R Ch

contradicción que viene de suponer que F admite un potencial vector.


Gabriela Sansigre.
AMPLIACIÓN DE CÁLCULO (Curso 2007/2008) Convocatoria de febrero 12.02.08

PROBLEMA 1 (4 puntos)
Nota: Los apartados de este problema son independientes.
Sea Γ el arco de la rama de la hipérbola y 2 − z 2 = 1, x = 0, limitado por los planos y = 1 e y = 2.

1) Calcúlese la masa total del arco Γ cuando sobre él se considera definida una densidad lineal de
masa que, en cada punto, es inversamente proporcional a la distancia del punto al origen.
(1 punto)

2) Sea Σ la superficie generada cuando el arco Γ gira alrededor del eje y. Sobre Σ se considera
definida una densidad de masa que en cada punto de la superficie es igual a la distancia al
origen del plano tangente a la superficie en ese punto. Calcúlense las coordenadas del centro
de gravedad de Σ.
(1 punto)

3) Sea Ω ∈ R3 el sólido acotado limitado por los planos z = 1, z = −1 y por la superficie de


revolución que se obtiene cuando el arco Γ gira alrededor del eje z. Sobre Ω se considera
definida una densidad de masa que en cada punto es igual a la distancia del punto al plano
XY . Calcúlese el momento de inercia de Ω con respecto a la recta que pasa por el punto (1, 1, 0)
y es paralela al eje z. (2 puntos)

Respuesta:

1) La densidad lineal de masa definida sobre el arco Γ es:


k
d(x, y, z) = √ ,
x2 + y2 + z2

donde k > 0 es la constante de proporcionalidad. Por tanto, la masa total del arco vendrá dado por
la integral curvilı́nea:
Z
k
MΓ = √ 2 ds .
Γ x + y2 + z2
Ahora bien, dado que la densidad es simétrica con respecto al origen y el arco es simétrico con
respecto al eje OY la integral anterior es igual al doble de la integral extendida al semi-espacio
z ≥ 0; es decir: Z
MΓ = 2 d(x, y, z)ds
Γz≥0

Puesto que el arco está situado en el plano Y Z, una parametrización del segmento situado en el
semiespacio z ≥ 0 es: r(u) = (x(u), y(u), z(u)), dónde

x(u) = 0 , y(u) = u , z(u) = + u2 − 1 , u ∈ [1, 2] ,

con lo que el vector tangente viene dado por


à ! s
u 2u2 − 1
r0 (u) = 0, 1, √ 2 =⇒ kr0 (u)k = .
u −1 u2 − 1
Por lo tanto,
s
Z 2 Z 2
k 1 2u2 − 1
MΓ = 2 q kr0 (u)k du = 2k √ du
1 x(u)2 + y(u)2 + z(u)2 1 2u2 − 1 u2 − 1
Z 2 √
1
= 2k √ du = 2k ArgCh(2) = 2k log(2 + 3) ,
1 u2 − 1

dónde la última integral se resuelve mediante el cambio de variable t = Ch(u):


Z 2 Z ArgCh(2) Z ArgCh(2)
1 Sh(t)
√ du = dt = dt = ArgCh(2) .
1 u2 − 1 0 |Sh(t)| 0

Parametrización alternativa:
El arco Γ admite la parametrización s(v) = (x(v), y(v), z(v)), dónde
√ √ √
x(v) = 0 , y(v) = v 2 + 1 , z(u) = v , v ∈ [− 3, 3] ,

con vector tangente dado por


à ! s
0 v 0 2v 2 + 1
s (v) = 0, √ , 1 =⇒ ks (v)k = .
v2 + 1 v2 + 1

Por lo tanto,
s
Z √3 Z √3
k 0 1 2v 2 + 1
MΓ = √ q ks (v)k dv = k √ √ dv
− 3 x(v)2 + y(v)2 + z(v)2 − 3 2v 2 + 1 v2 + 1
Z √3 √ √
1
= k √ √ dv = 2k ArgSh( 3) = 2k log(2 + 3) ,
− 3 v2 + 1

dónde, en este caso, la última integral se resuelve de forma análoga al caso anterior pero mediante el
cambio de variable t = Sh(v).
2) Para obtener la expresión de la densidad definida sobre la superficie Σ es necesario determinar
la ecuación del plano tangente a Σ en cada uno de sus puntos. Si n(x, y, z) denota un vector normal
cualquiera a la superficie en el punto (x, y, z) de la misma, la ecuación cartesiana del plano tangente
es:
n(x, y, z) · (X − x, Y − y, Z − z) = 0 ,
donde (X, Y, Z) representan las coordenadas cartesianas de los puntos del plano. La distancia de este
plano al origen y, por tanto, la densidad ρ(x, y, z) definida sobre Σ, se puede representar entonces
por
|n(x, y, z) · (x, y, z)|
ρ(x, y, z) = .
kn(x, y, z)k
Denotando por MΣ la masa total de Σ, que viene dada por la integral de superficie:
ZZ
MΣ = ρ(x, y, z) dσ ,
Σ

las coordenadas de su centro de gravedad,

Cgσ = (Xg, Y g, Zg) ,


son, por definición, las siguientes integrales de superficie:
1 ZZ
Xg = x ρ(x, y, z) dσ = 0 ,
MΣ Σ
1 ZZ
Yg = y ρ(x, y, z) dσ ,
MΣ Σ
1 ZZ
Zg = z ρ(x, y, z) dσ = 0 ,
MΣ Σ

donde la coordenadas Xg y Zg son nulas debido a que la superficie es simétrica con respecto al eje
OY y, por tanto, la densidad es una función par con respecto a las coordenadas x, z.
Para calcular las dos integrales de superficie que faltan, tenemos en cuenta que Σ es una superficie
de revolución obtenida al girar el arco Γ alrededor del eje OY . Por tanto, tiene por ecuación cartesiana:

f (x, y, z) := x2 − y 2 + z 2 − 1 = 0 , 1 ≤ y ≤ 2.

Un vector normal al punto (x, y, z) es entonces:

n(x, y, z) = grad(f (x, y, z)) = 2(x, −y, z) ,

lo que nos permite obtener la expresión de la densidad que resulta ser:


|n(x, y, z) · (x, y, z)| 2(x, −y, z) · (x, y, z) 1
ρ(x, y, z) = = √ 2 = √ 2 ,
kn(x, y, z)k 2 x +y +z 2 2 x + y2 + z2

donde se ha tenido en cuenta que, sobre la superficie, x2 − y 2 + z 2 = 1.


Por otra parte, una de las posibles parametrizaciones Σ(u, θ) = (x(u, θ), y(u, θ), z(u, θ)) tiene por
ecuaciones paramétricas:
√ √
x(u, θ) = u2 − 1 sen θ , y(u, θ) = u , z(u, θ) = u2 − 1 cos θ , u ∈ [1, 2] , θ ∈ [0, 2π] ,

con lo que un vector normal es:


³ √ √ ´ √
n(u, θ) = − u2 − 1 sen θ , u , u2 − 1 cos θ =⇒ kn(u, θ)k = 2u2 − 1 .

Ası́ pues, el valor de la densidad sobre la superficie es:


|n(u, θ) · Σ(u, θ)| 1
ρ(x(u, θ), y(u, θ), z(u, θ)) = = ,
kn(u, θ)k kn(u, θ)k
con lo que se tiene:
ZZ ZZ
MΣ = ρ(x, y, z) dσ = ρ(x(u, θ), y(u, θ), z(u, θ)) kn(u, θ)k dudθ
Σ (u,θ)∈[1,2]×[0,2π]
ZZ
= dudθ = 2π .
(u,θ)∈[1,2]×[0,2π]

Finalmente,
1 ZZ 1 ZZ
Yg = yρ(x, y, z) dσ = u dudθ
2π Σ 2π (u,θ)∈[1,2]×[0,2π]
¯2
1 Z 2π Z 2
u2 ¯¯ 3
= dθ udu = ¯ = ,
2π 0 1 2 ¯1 2

y el centro de gravedad es
3
Cgσ = (0 , , 0) .
2
3) El sólido Ω es simétrico con respecto al eje OZ y la densidad, que viene dada por Λ(x, y, z) = |z|,
es par en la variable z. Esto nos permite concluir que el centro de gravedad de Ω se encuentra situado
en el eje OZ.
Se puede entonces aplicar el teorema de Steiner según el cual el momento de inercia con respecto
al eje pedido, I(0,1,1) , se puede calcular a partir de la expresión

I(0,1,1) = IOZ + MΩ D2

dónde MΩ es la masa de Ω, IOZ es el momento de inercia √ con respecto al eje OZ (que es paralelo al
eje dado y pasa por el centro de gravedad de Ω) y D = 2 es la distancia entre los dos ejes.
La masa de Ω y su momento de inercia con respecto al eje OZ vienen dados por las integrales
triples: ZZZ ZZZ
MΩ = |z| dxdydz , IOZ = dz (x, y, z)2 |z| dxdydz
Ω Ω
√ 2 2
dónde dz (x, y, z) = x + y es la distancia al eje OZ.
Ahora bien, la simetrı́a de Ω con respecto al origen y el hecho de que las funciones a integrar sean
pares en las tres variables x, y y z, permite calcular las dos integrales como el doble de las mismas
integrales pero extendidas a la parte de Ω situada en el semi-espacio z ≥ 0. Esta parte de Ω queda
descrita adecuadamente en coordenadas cilı́ndricas por las ecuaciones

x = r sen θ y = r cos θ , z = z, θ ∈ [0, 2π] , r ∈ [0, z 2 + 1] , z ∈ [0, 1] .

Al utilizarlas hay que tener en cuenta que el jacobinao del cambio a cilı́ndricas es |J| = r.
Se tiene entonces:
ZZZ ZZZ Z 2π Z 1 Z √z 2 +1
MΩ = 2 z dxdydz = 2 zr dθdzdr = 2 dθ z dz r dr
Ω(z≥0) Ω0 0 0 0
Z 2π Z 1 Z 1
z2 + 1 3π
= 2 dθ z dz = 2π z (z 2 + 1) dz = .
0 0 2 0 2
Y:
ZZZ ZZZ Z 2π Z 1 Z √z 2 +1
2 2 3
IOZ = 2 z(x + y ) dxdydz = 2 zr dθdzdr = 2 dθ z dz r3 dr
Ω(z≥0) Ω0 0 0 0
Z 2π Z 1 Z 1
(z 2 + 1)2 7π
= 2 dθ z dz = π z (z 2 + 1)2 dz = .
0 0 4 0 6
Finalmente, utilizando al teorema de Steiner ya mencionado, el momento de inercia de Ω con respecto
al eje paralelo al eje OZ y que pasa por el punto (0, 1, 1) es:
7π 25π
I(0,1,1) = IOZ + 2 MΩ = + 3π = .
6 6
AMPLIACIÓN DE CÁLCULO (Curso 2007/2008) Convocatoria de febrero 12.02.08

PROBLEMA 2 (3 puntos)

1. (0, 75 puntos) Sea f un campo escalar de clase 2 en R2 . Se dice que una función g es conjugada
de f si se verifican las relaciones:

∂g ∂f ∂g ∂f
=− , = .
∂x ∂y ∂y ∂x

Dese una condición necesaria y suficiente sobre f para que admita una conjugada de clase 2.

2. (1 punto) Calcúlese una función h con h(0) = h0 (0) = 1 y para la cual f (x, y) = cos x h(y)
cumple la condición hallada en el primer apartado. En tal caso, determı́nese una conjugada g
de f .

3. (1, 25 puntos) Se considera la curva Γ parametrizada por ϕ(t) = (sec2 t, 2 + tg t), t ∈ [0, π/4]
y el campo vectorial
Z V = (g, f ) siendo f y g las funciones del segundo apartado. Calcúlese la
integral de lı́nea V.
Γ

Respuesta:

1. Si se busca una función g de clase dos, sus derivadas parciales de segundo orden cruzadas
coinciden:
∂2g ∂2g
=
∂x∂y ∂y∂x

lo cual se traduce en
∂2f ∂2f
= − ⇐⇒ ∆f = 0;
∂x2 ∂y 2

es decir, la función f tiene que ser armónica. La condición es también suficiente; basta con
integrar una cualquiera de las dos ecuaciones (operación que puede realizarse gracias a la
continuidad de ∂f /∂y),
Z x ∂f
g(x, y) − g(0, y) = − (t, y)dt
0 ∂y

e imponer que se satisfaga la otra:

∂g ∂g Z x
∂2f Z x 2
∂ f ∂f ∂f
(x, y) − (0, y) = − 2 (t, y)dt = 2
(t, y)dt = (x, y) − (0, y)
∂y ∂y 0 ∂y 0 ∂x ∂x ∂x

(tengamos presente que la derivación paramétrica es posible ya que tanto ∂f /∂y como ∂ 2 f /∂y 2
son continuas) con lo que la función g buscada queda:
Z x ∂f Z y
∂f
g(x, y) = − (t, y)dt + (0, t)dt + c,
0 ∂y 0 ∂x

siendo c una constante arbitraria.


2. Calculemos el laplaciano de f (x, y) = cos x h(y),

∂2f ∂2f
+ = − cos x h(y) + cos x h00 (y) = cos x(−h(y) + h00 (y)) = 0 ⇐⇒
∂x2 ∂y 2
h00 (y) − h(y) = 0 ⇐⇒ h(y) = aey + be−y , a, b constantes

Al imponer las condiciones iniciales h(0) = h0 (0) = 1 se tiene a+b = 1, a−b = 1 ⇒ a = 1, b = 0.


Entonces h(y) = ey y f (x, y) = cos x ey .
Para calcular g aplicamos la fórmula deducida en el apartado anterior; como nos piden una
cualquiera, tomamos la constante c nula:
Z x ∂f Z x
g(x, y) = − (t, y)dt = −ey cos t dt = − sen x ey .
0 ∂y 0

3. Observemos que el campo V = (g, f ) es conservativo ya que se cumple la condición ∂g/∂y =


∂f /∂x que, en un simplemente conexo (todo R2 en este caso) además de necesaria es también
suficiente. La integral de lı́nea depende exclusivamente de los extremos inicial y final de la
curva que son A = ϕ(0) = (1, 2), B = ϕ(π/4) = (2, 3). Podemos proceder de dos formas, o
bien calculando un potencial escalar de V o bien integrando a lo largo del segmento de extremos
A y B.

3 · 1 Cálculo de la integral de lı́nea a partir de un campo h, potencial escalar de V.


∂h
(x, y) = f (x, y) = cos x ey podemos elegir h(x, y) = cos x ey ,
∂y
Z Z
V= ∇h = h(B) − h(A) = e3 cos 2 − e2 cos 1.
Γ Γ

3 · 2 Cálculo de la integral de lı́nea a lo largo del segmento [A, B].


Sea ψ(t) = (1, 2) + t(1, 1), t ∈ [0, 1] una parametrización del segmento. Entonces:
Z Z 1 Z 1
0
V= V(ψ(t)) · ψ (t)dt = e2+t (cos(t + 1) − sen(t + 1))dt.
Γ 0 0

Una primitiva de et (cos t − sen t) es et cos t, ası́ pues:


Z
V = e2+t cos(t + 1)|10 = e3 cos 2 − e2 cos 1.
Γ

Gabriela Sansigre.
AMPLIACIÓN DE CÁLCULO (Curso 2007/2008) Convocatoria de febrero 12.02.08

NOMBRE . . . . . . . . . . . . . . . . . . . . . . . . . APELLIDOS . . . . . . . . . . . . . . . . . . . . . . . . . . . . . . . . . . . . . . . . . . .
Número de matrı́cula. . . . . . . . . . . . . . . . . . . . . . . .

PROBLEMA 3: (3 puntos)

1. En R3 se considera el campo vectorial F(r) = v × r, donde v es un vector fijo. Determinar


todas las funciones f : R → R tales que el campo f (v · r) r es un potencial vector de F.
(0’5 puntos)

2. Sea S la porción del cilindro 2x2 + (y + z − 1)2 = 1 limitada entre los planos z = 0 y z = 3,
orientada según la normal exterior al cilindro. Relacionar el flujo del campo F a través de S
con dos integrales curvilı́neas, justificando la respuesta. (1 punto)

3. Calcular dicho flujo cuando v = (1, 0, 1), utilizando el resultado del apartado anterior.
(1’5 puntos)

Respuesta: (Se entregará esta hoja y, a lo sumo, una adicional)

1. Impongamos que el rotacional del campo G (r) = f (v · r) r sea F (r) = v × r : aplicando las
propiedades del rotacional, y el hecho de que rot(r) = 0, se tiene

rot (f (v · r) r) = f 0 (v · r) v × r+f (v · r) rot (r) = f 0 (v · r) v × r.

Este rotacional es igual a v × r si y sólo si f 0 (v · r) = 1 para todo vector de posición r, es


decir, si y sólo si f 0 (t) = 1 para todo t real. Por tanto, las funciones pedidas son todas las de
la forma
f (t) = t + c, c ∈ R.
Ası́ pues, los potenciales vectores de F son los campos de la forma

G (r) = (v · r+c) r = (v · r) r+cr

donde c es una constante arbitraria.

2. La superficie cilı́ndrica S : 2x2 + (y + z − 1)2 = 1, limitada por los planos z = 0 y z = 3 no es


cerrada; la cerraremos con las tapas superior e inferior:

S1 : 2x2 + (y + 2)2 ≤ 1 en el plano z = 3


S0 : 2x2 + (y − 1)2 ≤ 1 en el plano z = 0.

Denotaremos C1 , C0 a sus respectivas curvas borde;√


obsérvese que se trata de elipses con distinto
centro, pero con iguales longitudes de semiejes: 1/ 2 y 1.
Sabemos por el apartado anterior que F es solenoidal, pues admite un potencial vector. Por
tanto, aplicando el Teorema de la divergencia de Gauss a la superficie cerrada, se tiene que el
flujo de F a través de la cara exterior de S es
Z Z Z
F ds = F ds+ F ds
S S1 S0
donde tanto S1 como S0 deben estar orientadas según la normal que apunta al interior del
cilindro, es decir, S1 con vector normal unitario (0, 0, −1) y S0 con vector normal unitario
(0, 0, 1).
Para transformar dichos flujos en integrales curvilı́neas, aplicamos el Teorema de Stokes en
cada una de ellas; puede aplicarse puesto que F =rot(G) y G ∈ C 1 (R3 ) :
Z Z
F ds = G dl donde la curva C1 está orientada negativamente en el plano z = 3 y
S1 C1
Z Z
F ds = G dl donde la curva C2 está orientada positivamente en el plano z = 0.
S0 C0

En efecto, el Teorema de Stokes indica que las orientaciones de la superficie y su curva borde
deben ser coherentes; en el caso de la superficie S1 , que está orientada según el vector normal
unitario (0, 0, −1) , la orientación coherente de su curva borde C1 es la orientación negativa.
Por fin, la relación buscada es
Z Z Z Z Z
F ds = G dl+ G dl = (v · r) r dl+ (v · r) r dl
S C1 C0 C1 C0

donde se ha elegido elR potencial vector G con c = 0. (Si fuera c =6 0, se tendrı́a la expresión
idéntica, puesto que C cr dl =0 al tratarse de la circulación de un campo conservativo a lo
largo de una curva cerrada).

3. Para v = (1, 0, 1) , el potencial vector es (v · r) r = (x + z) r. Aplicando lo anterior,


Z Z Z
(1, 0, 1) × r ds = (x + z) r dl+ (x + z) r dl
S C1 C0

donde ambas curvas deben estar orientadas convenientemente. La curva C0 , de ecuaciones


2x2 + (y − 1)2 = 1, z = 0 debe estar parametrizada como
!
cos t
r (t) = (x (t) , y (t) , z (t)) = √ , 1 + sen t, 0 t ∈ [0, 2π]
2
para que su orientación sea positiva. Su vector derivada es
!
0 0 0 sen t0
r (t) = (x (t) , y (t) , z (t)) = − √ , cos t, 0 .
2
Por ello, la circulación a lo largo de C0 es
Z Z 2π
(x + z) r dl = x (t) (r (t) · r0 (t)) dt=
C0 0
Z 2π
cos t 1
 
= √ sen t cos t + cos t dt.
0 2 2
R 2π R 2π
Dado que las integrales del tipo 0 senp t cos t dt o 0 sen t cosp t dt son nulas (para p ≥ 0),
queda solamente √
Z
1 Z 2π 2 π 2
(x + z) r dl = √ cos t = √ = π.
C0 2 0 2 2
Calculemos ahora la integral a lo largo de C1 , de ecuaciones 2x2 + (y + 2)2 = 1, z = 3. Si la
parametrizamos como
!
cos t
r (t) = (x (t) , y (t) , z (t)) = √ , −2 + sen t, 3 t ∈ [0, 2π]
2
entonces !
0 0 0 sen t
0
r (t) = (x (t) , y (t) , z (t)) = − √ , cos t, 0
2
pero queda orientada positivamente, y deseamos la orientación contraria; por tanto, cambiamos
de signo la circulación obtenida:
Z Z 2π
(x + z) r dl = − (x (t) + 3) (r (t) · r0 (t)) dt=
C1 0
Z 2π !
cos t 1

= − √ +3 sen t cos t − 2 cos t dt =
0 2 2
2 Z 2π √
= √ cos2 t = 2π.
2 0
Finalmente, se obtiene que el flujo pedido es:
√ √
Z
2 √ 3 2
F ds = π + 2π = π.
S 2 2

ALTERNATIVA PARA EL APARTADO 3: calcular las integrales de superficie a través de las


superficies planos S1 y S0 , orientadas convenientemente. Por ejemplo, denotando el dominio plano
D0 = {(x, y) ∈ R2 , 2x2 + (y − 1)2 ≤ 1} se tiene que
Z Z Z Z
F ds = (1, 0, 1) × (x, y, 0) dr = ((1, 0, 1) × (x, y, 0)) · (0, 0, 1) dxdy =
S0 S0 D0


1 0 1
Z Z Z Z
π 2
=
x y 0 dxdy = ydxdy = 1 · Área (D0 ) = √ = π


D0 D0 2 2
0 0 1

D0 ydxdy = yG ·Área(D0 ) y el centro geométrico del interior de una


RR
donde se ha usado que
elipse coincide con √
el centro de la elipse. En este
√ caso la elipse tiene centro (0, 1) y las longitudes de
sus semiejes son 1/ 2 y 1, luego el área es π/ 2.
De forma análoga, para S1 definimos el dominio plano D1 = {(x, y) ∈ R2 , 2x2 + (y + 2)2 ≤ 1} :
Z Z Z Z
F ds = (1, 0, 1) × (x, y, 3) dr = ((1, 0, 1) × (x, y, 3)) · (0, 0, −1) dxdy =
S1 S1 D1

1 0 1 √
Z Z Z Z

= −
x y 3 dxdy = − ydxdy = − (−2) Área (D1 ) = √ = 2π


D1
0 0 1
D1 2

puesto que el centro de la elipse ahora es (0, −2) , aunque su área es la misma. Finalmente, llegamos
al mismo resultado: √ √
Z
2 √ 3 2
F ds= π + 2π = π.
S1 2 2
AMPLIACIÓN DE CÁLCULO (Curso 2007/2008) Convocatoria de junio 17.06.08

NOMBRE . . . . . . . . . . . . . . . . . . . . . . . . . . . . . . . . APELLIDOS . . . . . . . . . . . . . . . . . . . . . . . . . . . . . . . . . . . . . . . . . . . . . . . . . . . . .
Número de matrı́cula . . . . . . . . . . . . . . . . . . . . . . . . . . . . . . . .

PROBLEMA 1 (4 puntos)

1) (1.5 ptos.) Determinar razonadamente los valores de a para los cuales está definida la función
Z ∞
arctan(ax) − arctan x
I(a) = dx
0 x

2) (1 pto.) Estudiar si I(a) es derivable para los valores de a hallados en el apartado anterior.
3) (0.5 ptos.) Calcular I(a)
4) (1 pto.) Calcular razonadamente la integral
Z ∞
(ax2 − 1) log x
dx
0 (1 + a2 x2 )(1 + x2 )
para a > 0, a 6= 1.

Respuesta: Se entregará esta hoja y, a lo sumo, una adicional.

1)
Obsérvese en primer lugar que si se separa la integral en suma de integrales en la forma
Z ∞ Z ∞ Z ∞
arctan(ax) − arctan(x) arctan(ax) arctan x
dx = dx − dx
0 x 0 x 0 x
se obtiene (es fácil comprobarlo comparando con 1/x) que la primera diverge para todo a 6= 0 y la segunda es divergente.
Por ello procediendo de esta forma la única consecuencia que se puede extraer sobre la convergencia de I(a) es que
para a = 0 la integral es divergente (pues es suma de una integral convergente y una divergente). Por ello para estudiar
la convergencia de la integral impropia para el resto de los valores de a se debe proceder de otra forma.
Sea
arctan ax − arctan x
f (x, a) := dx
x
Para todo a ∈ R, la función f (∗, a) (es decir, como función de x) es de clase C ∞ (0, ∞), luego está acotada en todos
los compactos de dicho conjunto. Además,
a 1
arctan ax − arctan x L0 Hospital 1+a2 x2 − 1+x2
lim+ f (x, a) = lim+ = lim+ =a−1
x→0 x→0 x x→0 1
con lo que para todo a ∈ R, la función f (∗, a) está acotada en el entorno de x = 0. Por ello, la integral sólo es impropia
porque el intervalo de integración es no acotado.

a>1
π/2

a=1

0<a<1

arctg(ax)

a<0

−π/2

0 1 2 3 4 5 6 7 8 9 10
Cuando a = 1 la función subintegral se anula luego la integral es convergente. Puesto que arctan(ax) es una función
creciente si a > 0 y decreciente si a < 0 (ver figura) se tiene que para todo a 6= 1, la función f (∗, a) conserva el signo
en (0, ∞) (es o siempre negativa o siempre positiva). En concreto, para x > 0, arctan ax − arctan x ≥ 0 si a > 1 y
arctan ax − arctan x ≤ 0 si a < 1. Por ello, para estudiar la convergencia de I podemos tomar valor absoluto (pues
al no cambiar de signo la función subintegral, la convergencia es equivalente a la convergencia absoluta) y utilizar los
criterios de convergencia para funciones no negativas.
Cuando a < 0, el numerador de f (∗, a) tiende a −π cuando x → ∞, lo que parece que f (∗, a) se comporta cuando
x → ∞ ”como −1/x” cuando x → ∞, con lo que la integral parece ser divergente. En efecto
¯ arctan ax−arctan x ¯
¯ ¯
x
lim = π 6= 0
x→∞ 1/x
R∞ R∞
y como 1 dx x es divergente, también lo es 1 f (x, a)dx y por ello I(a) (recuérdese que este criterio se ha podido
aplicar porque la función subintegral es no negativa). Cuando a = 0
¯ arctan ax−arctan x ¯
¯ ¯
x
lim = π/2 6= 0
x→∞ 1/x
con lo que, razonando de forma análoga, la integral diverge.
Cuando a > 0 el numerador de f tiende a cero cuando x → ∞, con lo que intuı́mos que quizás f (x, a) se comporte
”como 1/x2 ” cuando x → ∞. Veamos,
¯ arctan ax−arctan x ¯ ¯ ¯ ¯ ¯
¯ ¯ ¯ arctan ax − arctan x ¯ ¯ a 1 ¯
2 x2 − 1+x2 ¯
L 0
Hôpital ¯
lim x
= ¯¯ lim ¯
¯ = ¯ lim
1+a
¯=
x→∞ 1/x2 x→∞ 1/x ¯x→∞ − x12 ¯
|a − 1|
= finito
a
R∞
y como 1 dx x2 es convergente, I(a) es convergente.
Resumiendo, I(a) converge (y además lo hace absolutamente) si y sólo si a > 0.
Obsérvese que, salvo para el caso trivial a = 1, no es posible aplicar el criterio de Dirichlet para asegurar la
convergencia de la integral. En efecto, aunque 1/x es C 1 (0, ∞), es estrictamente decreciente y tiende a cero, la función
arctan ax − arctan x no cumple que la primitiva
Z x
F (x) := (arctan at − arctan t)dt
0

esté acotada. En efecto, ya hemos razonado que para a 6= 1 la función subintegral


R∞ conserva el signo (positivo si a > 1
y negativo si a < 1). Por ello F (x) está acotada si y sólo si la integral 0 (arctan at − arctan t)dt es convergente.
Puesto
R∞ que en este caso la convergencia es equivalente a la convergencia absoluta estudiamos la convergencia de
0
|arctan at − arctan t| dt que es divergente pues si a < 0,

|arctan ax − arctan x|
lim = π 6= 0
x→∞ 1
R∞
y 0
1dx es divergente. Si a > 0, con a 6= 1,
¯ ¯ ¯ £ ¤ ¯¯
|arctan ax − arctan x| ¯ arctan ax − arctan x ¯ L0 Hôpital ¯¯ x2
(a − a2 2
)x + a − 1 ¯
lim = ¯¯ lim ¯
¯ = ¯ lim ¯=
x→∞ 1/x x→∞ 1/x ¯x→∞ (1 + a2 x2 ) (1 + x2 ) ¯
¯ ¯ ¯ ¯
¯ a − a2 ¯ ¯ 1 − a ¯
= ¯¯ ¯=¯ ¯ finito y no nulo
a2 ¯ ¯ a ¯
R∞ R∞
Como 0 x1 dx es divergente, la integral 0 (arctan at − arctan t)dt también es divergente. Como conclusión, salvo en
el caso a = 1, la función F (x) no está acotada. R∞
2) Sea [c, d] un intervalo compacto contenido en (0, ∞). Sabemos que para poder garantizar que 0 f (x, a)dx
sea derivable para a ∈ [c, d], se debe cumplir que f (x, a) y ∂f ∂a (x, a) sean continuas en [c, d] × (0, ∞) y que la integral
R ∞ ∂f
0 ∂a
(x, a)dx converja uniformemente en [c, d]. Claramente lo primero se cumple. Estudiemos entonces la convergencia
uniforme de Z ∞ Z ∞
∂f 1
(x, a)dx = dx
0 ∂a 0 1 + a2 x 2
Claramente, si a ∈ [c, d] ⊂ (0, ∞) ¯ ¯
¯ 1 ¯ 1 1
¯ ¯
¯ 1 + a2 x2 ¯ = 1 + a2 x2 ≤ 1 + c2 x2
R∞ R∞
y, como c 6= 0, 0 1+c12 x2 dx es convergente, por lo que por el criterio de Weierstrass, la integral 0 ∂f
∂a (x, a)dx converge
uniformemente en todo intervalo [c, d] con c > 0. Por consiguiente, I(a) es derivable en (0, ∞) y además, su derivada
vale Z ∞
0 1
I (a) = dx , a > 0
0 1 + a2 x2
3) Integrando en la expresión de I 0 (a) se tiene
Z ∞
1 1 a>0 π
I 0 (a) = 2 x2
dx = arctan(ax) |x=∞
x=0 = , a>0
0 1 + a a 2a

y por ello
π π
I(a) =log |a| + C = log a + C, a > 0
2 2
Imponiendo que I(1) = 0 se obtiene C = 0 luego
π
I(a) = log a, a > 0
2
A continuación haremos algunas observaciones (que no son necesariasR para la resolución del problema) sobre la

convergencia puntual y la convergencia uniforme de la integral H(a) := 0 1+a12 x2 dx. Hemos visto en el apartado
anterior que H(a) converge uniformemente en todo intervalo de la forma [c, d] con c > 0, de donde que se deduce que
H(a) converge puntualmente en (0, ∞) (en efecto, para todo a ∈ (0, ∞) existen 0 < c < d tales que a ∈ [c, d] y como
H converge uniformemente en [c, d] también lo hará puntualmente y por ello H(a) será convergente). Sin embargo,
ahora veremos que H(a) no converge uniformemente en (0, ∞) y de hecho ni siquiera lo hace en (0, d] para ningún
d > 0. En efecto, de la definición de convergencia uniforme se tiene que H(a) converge uniformemente en (0, d] si
¯Z ¯
¯ T 1 ¯
¯ ¯
sup ¯ dx − H(a)¯ → 0
a∈(0,d] ¯ 0 1 + a 2 x2 ¯ T →∞

Veamos
¯Z ¯ ¯ ¯
¯ T 1 ¯ ¯1 π ¯¯
¯ ¯
sup ¯ dx − H(a)¯ = sup ¯¯ arctan(ax) |x=T − =
a∈(0,d] ¯ 0 1 + a x
2 2 ¯ a∈(0,d] a
x=0
2a ¯
¯ ¯
¯1
¯ π ¯¯ 1 ¯¯ π ¯¯
= sup ¯ arctan(aT ) − ¯ = sup ¯arctan(aT ) − ¯ = ∞
a∈(0,d] a 2a a∈(0,d] a 2

que, obviamente, no tiende a cero cuando T → ∞. Obsérvese que si en vez de trabajar en (0, d) se trabaja en [c, d],
con c > 0, se tiene
¯Z ¯
¯ T 1 ¯ 1 ¯¯ π ¯¯ 1 ¯¯ π ¯¯
¯ ¯
sup ¯ dx − H(a)¯ = sup ¯ arctan(aT ) − ¯ = ¯ arctan(cT ) − ¯ → 0
a∈[c,d] ¯ 0 1 + a x ¯ a∈[c,d] a
2 2 2 c 2 T →∞

lo que muestra, utilizando la definición, que hay convergencia uniforme en todo intervalo [c, d], con c > 0 (esto ya lo
sabı́amos del apartado 2)
4) Sea J(a) la integral del enunciado. Parece razonable intentar relacionar J(a) con I(a). En vista de que el
intervalo de integración es el mismo y de la expresión de las funciones subintegrales, podemos intentar integrar por
partes en la expresión de I tomando u(x) = arctan(ax) − arctan x y v 0 (x) = x1 . Entonces

a 1 a − 1 + (a − a2 )x2 ax2 − 1
u0 (x) = − = = (1 − a)
1 + a2 x2 1 + x2 (1 + a2 x2 ) (1 + x2 ) (1 + a2 x2 ) (1 + x2 )
v(x) = log x

con lo que
Z ∞
¡ 2 ¢
ax − 1 log x
I(a) = (arctan ax − arctan x) log x |∞
0 −(1 − a) dx = (a − 1)J(a)
0 (1 + a2 x2 ) (1 + x2 )
para a > 0, donde se ha usado que
arctan ax − arctan x L0 Hôpital
lim+ (arctan ax − arctan x) log x = lim+ 1 =
x→0 x→0
log x
2 ¡ ¢
(1 − a) (1+aax −1
2 x2 )(1+x2 ) (1 − a) ax2 − 1 x log2 x
= lim −1 1 = lim+ =
x→0+
log2 x x
x→0 (1 + a2 x2 ) (1 + x2 )
2
= (a − 1) lim+ x log x = 0
x→0
arctan ax − arctan x L0 Hôpital
lim (arctan ax − arctan x) log x = lim 1 =
x→∞ x→∞
log x
¡ ¢
(1 − a) ax2 − 1 x log2 x
= lim =0
x→∞ (1 + a2 x2 ) (1 + x2 )

Por tanto, para a > 0, a 6= 1


1 π log a
J(a) = I(a) =
a−1 2 a−1
AMPLIACIÓN DE CÁLCULO (Curso 2007/2008) Convocatoria de junio 17.06.08

NOMBRE . . . . . . . . . . . . . . . . . . . . . . . . . APELLIDOS . . . . . . . . . . . . . . . . . . . . . . . . . . . . . . . . . . . . . . . . . . .
Número de matrı́cula. . . . . . . . . . . . . . . . . . . . . . . .

PROBLEMA 2 (3 puntos)
Dada una constante a > 0, se considera el arco Γ de la curva llamada astroide contenido en el
primer cuadrante. Las ecuaciones paramétricas de esa curva son:
x = a cos3 t
(

y = a sen3 t .
Se pide:
(1) Hallar el área del dominio D limitado por el arco Γ y los ejes coordenados. Hallar también la
longitud del arco Γ. (1 punto)
(2) Hallar el centroide (xD , yD ) del dominio D. Obviamente, por simetrı́a, se verifica xD =
yD . (1 punto)
(3) Hallar el centroide (x0 , y0 ) del arco Γ. Obviamente, por simetrı́a, se verifica x0 = y0 .(1 punto)

Respuesta: Se entregará esta hoja y, a lo sumo, una adicional.


Elevando ambas ecuaciones a potencia 2/3 y sumando miembro a miembro, se obtiene la ecuación
de la astroide en coordenadas cartesianas:
x2/3 + y 2/3 = a2/3 .
La gráfica de Γ puede verse en la Figura 1.
y
a

D Γ
x
0 a

Figura 1: Arco de astroide

(1) Se tiene
3/2
ZZ Z a Z (a2/3 −x2/3 )
Área(D) = dxdy = dx dy
D 0 0
  1/3 
x
Z a  3/2 t = arc sen a
a2/3 − x2/3
 
= dx 
 x = a sen3 t


0
dx = 3a sen2 t cos t dt
Z π/2  3/2
= a2/3 − a2/3 sen2 t 3a sen2 t cos t dt
0
π/2 3a2 3 5
Z  
2 2 4
= 3a sen t cos t dt = B ,
0 2 2 2

1
3 5 1 1 31 1
       
3a2 Γ 2 Γ 2 3a2 2 Γ 2 22
Γ
2 3πa2
= = = .
2 Γ(4) 2 6 32

Por otra parte, la longitud es


Z π/2 q
l(Γ) = x′ (t)2 + y ′ (t)2 dt .
0

Como

x′ (t) = −3a cos2 t sen t


y ′(t) = 3a sen2 t cos t ,

se tiene

x′ (t)2 + y ′(t)2 = 9a2 (cos4 t sen2 t + sen4 t cos2 t)


= 9a2 sen2 t cos2 t(cos2 t + sen2 t) ,

de donde " #π/2


π/2 sen2 t 3a
Z
l(Γ) = 3a sen t cos t dt = 3a = .
0 2 0
2

(2) Se tiene
ZZ
Área(D) · xD = x dxdy
D
Z a Z (a2/3 −x2/3 )3/2
= x dx dy
0 0
x = a sen3 t
!
Z a
2/3 2/3 3/2
= x(a −x ) dx
0 dx = 3a sen2 t cos t dt
Z π/2
= a sen3 t(a2/3 − a2/3 sen2 t)3/2 3a sen2 t cos t dt
0
Z π/2
3
= 3a sen5 t cos4 t dt
0
3a3 5
 
= B 3,
2 2
5
 
3a3 Γ(3)Γ 2
=
11
 
2 Γ
2
8a3
= ,
105
de donde
8a3 256a
xD = = .
105 Área(D) 315π

(3) Se tiene
Z
l(Γ)x0 = x ds
Γ

2
Z π/2 q
= a cos3 t x′ (t)2 + y ′ (t)2 dt
0
Z π/2
=a cos3 t 3a sen t cos t dt
0
Z π/2
2
= 3a cos4 t sen t dt
0
" #π/2
cos5 t
= 3a2 −
5 0
3a2
= ,
5
de donde
3a2 2a
x0 = = .
5l(Γ) 5

3
AMPLIACIÓN DE CÁLCULO (Curso 2007/2008) Convocatoria de junio 17.06.08

NOMBRE . . . . . . . . . . . . . . . . . . . . . . . . . APELLIDOS . . . . . . . . . . . . . . . . . . . . . . . . . . . . . . . . . . . . . . . . . . .
Número de matrı́cula. . . . . . . . . . . . . . . . . . . . . . . .

PROBLEMA 3 (3 puntos)
Se considera el campo vectorial definido por:

(z , x z , f (x, y))
F(x, y, z) =
(x2 + 8y 2 + z 2 )3/2

donde f (x, y) es una función de clase C 1 en R2 .

1) Determı́nese la función f (x, y) para que el campo F sea solenoidal. Para ese valor de f (x, y)
obténgase un potencial vector de F cuya tercera componente sea nula, indicando razonadamente
la mayor región de R3 en la que F lo admite.
(1 punto)

2) Sea Γ la porción de curva de ecuación cartesiana x2 + 4y 2 = 1, z = x que está situada en el


primer octante. Sea Σ1 la superficie generada por los segmentos que parten del punto (1, 0, 1)
y se apoyan en Γ. Calcúlese el flujo del campo F, determinado en el primer apartado, a través
de la cara de Σ1 cuya normal tiene tercera componente positiva.
(1 punto)

3) Sea Σ2 una superficie regular y cerrada cualquiera que no pasa por el origen. Calcúlense razon-
adamente todos los valores posibles que puede tomar el flujo del campo F, determinado en el
primer apartado, a través de la cara exterior de la mencionada superficie. (1 punto)

Respuesta:

1) El campo F(x, y, z) = (L(x, y, z), M (x, y, z), N (x, y, z)) está definido en R3\{(0, 0, 0)}. En ese
conjunto su divergencia es:

∂L ∂M ∂N 3z(x + 8xy + f (x, y))


∇·F= + + =− .
∂x ∂y ∂z (x2 + 4y 2 + z 2 )5/2

Por definición F es solenoidal si su divergencia es nula, lo que ocurre si y solamente si

f (x, y) = −x(1 + 8y) ,

que es la función f (x, y) que se pide. Para esta expresión de f se tiene un campo solenoidal en el
conjunto R3 \{(0, 0, 0)} que no es estrellado con respecto a ningún punto. En esta situación no se
puede asegurar la existencia de un potencial vector para F en este conjunto. Lo que sı́ podemos
afirmar es que F admite un potencial vector en cualquier conjunto estrellado que esté contenido en
R3 \{(0, 0, 0)}.
Para calcularlo suponemos que el potencial vector es un campo de la forma

G(x, y, z) = (P (x, y, z), Q(x, y, z), 0) ,


que debe cumplir 


∂Q z

 − = 2 ,

 ∂z (x + 8y 2 + z 2 )3/2


 ∂P xz
∇ × G = F ⇐⇒  = 2 ,

 ∂z (x + 8y 2 + z 2 )3/2





∂Q ∂P −x(1 + 8y)
 − = 2 .
∂x ∂y (x + 8y 2 + z 2 )3/2
Integrando en la primera de las igualdades con respecto a z, se obtiene:
1
Q(x, y, z) = + C1 (x, y) ,
(x2 + 8y 2 + z 2 )1/2
donde C1 (x, y) es una función arbitraria que solamente puede depender de las variables x e y.
Análogamente, integrando en la segunda de las igualdades con respecto a z, se obtiene:
x
P (x, y, z) = − 2 + C2 (x, y) ,
(x + 8y 2 + z 2 )1/2
donde C2 (x, y) es otra función arbitraria que solamente puede depender de las variables x e y.
Sustituyendo estas expresiones de P y Q en la tercera igualdad, resulta la siguiente condición que
deben cumplir las funciones C1 y C2 :
∂C2 (x, y) ∂C1 (x, y)
− = 0.
∂x ∂y
Una elección posible (la más sencilla) es C1 (x, y) = C2 (x, y) = 0 con lo que un potencial vector del
campo F es: Ã !
x 1
G(x, y, z) = − 2 , , 0
(x + 8y 2 + z 2 )1/2 (x2 + 8y 2 + z 2 )1/2
y cualquier otro potencial vector de F se diferenciará del dado en un campo de gradientes.
Ahora bien, el campo G está definido y es de clase C 1 en todo el conjunto R3 \{(0, 0, 0)} y, además,
en todo ese conjunto también cumple ∇ × G = F. Esto significa (por definición de potencial vector)
que el campo G es un potencial vector de F en R3 \{(0, 0, 0)}. (1 punto)

2) Teniendo en cuenta que la superficie Σ1 no es cerrada y puesto que el campo F que se


considera es solenoidal y admite un potencial vector, se dispone de varias posibilidades a la hora de
calcular su flujo.
Una de ellas consiste en utilizar la propia definición de flujo, para lo que es necesario obtener una
parametrización de la superficie Σ1 que está formada por la el haz de segmentos que unen el punto
(1, 0, 1) con el arco Γ descrito en el enunciado. Unas ecuaciones paramétricas de esta superficie son:
sen θ
x(u, θ) = 1 + u(cos θ − 1) , y(u, θ) = u , z(u, θ) = 1 + u(cos θ − 1) ,
2
donde u ∈ [0, 1] y θ ∈ [0, π/2], dado que la curva Γ está contenida en el primer octante y se ha
parametrizado mediante las ecuaciones
sen θ
x(θ) = cos θ , y(θ) = , z(θ) = cos θ , θ ∈ [0, π/2] .
2
Ahora bien, el valor que toma el campo sobre esta superficie es
1
F(x(u, θ), y(u, θ), z(u, θ)) = ×
(2 + 4(cos θ − 1)u + 4(1 − cos θ)u2 )3/2
´
(1 + u (cos θ − 1) , (1 + u (cos θ − 1))2 , − (1 + u (cos θ − 1)) (1 + 4 u sen θ) .
lo que desaconseja utilizar este procedimiento dada la complejidad de las integrales que serı́a necesario
calcular.
Otra posibilidad pasa por hacer uso del teorema de Stokes. Dado que Σ1 y su curva borde
cumplen las condiciones de regularidad a trozos adecuadas y ambas están contenidas en un abierto
de R3 \{(0, 0, 0)} (que siempre existe) en el que se cumple ∇ × G = F (con G de clase C ∞ ), el
teorema de Stokes permite escribir:
ZZ ZZ I
F · dσ = ∇ × G · dσ = G · ds ,
Σ1 Σ1 Λ

dónde Λ representa el borde de Σ1 orientado de tal forma que su proyección sobre le plano z = 0
se recorra en sentido positivo (lo que se corresponde con el sentido que dicta “la regla del sacacor-
chos” aplicada teniendo en cuenta que es la normal con tercera componente positiva la que se debe
considerar para Σ1 ).
La curva cerrada Λ está formada por el arco Γ (una de cuyas parametrizaciones se ha dado
anteriormente) y los segmentos que unen el punto (1, 0, 1) con los extremos de Γ que son el mismo
(1, 0, 1) y el punto (0, 1/2, 0). Ası́ pues, Λ está formada por la unión yuxtapuesta del arco Γ, recorrido
desde el punto (1, 0, 1) al (0, 1/2, 0), y el segmento S que une (0, 1/2, 0) con (1, 0, 1) recorrido en ese
mismo sentido. Una parametrización de S es:
1−v
x(v) = v , y(v) = , z(v) = v , v ∈ [0, 1] .
2
Por tanto: ZZ ZZ I Z Z
F · dσ = ∇ × G · dσ = G · ds = G · ds + G · ds .
Σ1 Σ1 Λ Γ S

Calculamos ahora las dos circulaciones del potencial vector G.


Z
• G · ds.
Γ
El valor que el potencial vector G toma sobre el arco Γ es:
à !
cos θ 1
G(x(θ), y(θ), z(θ)) = − √ , √ , 0
2 2
y la tangente al arco correspondiente a la parametrización considerada es:
à !
cos θ
T(θ) = − sen θ , , − sen θ .
2

Se tiene entonces,
cos θ
G(x(θ), y(θ), z(θ)) · T(θ) = √ (1 + 2 sen θ),
2 2
luego:
Z Z π/2 ÃZ Z π/2 !
1 π/2
G · ds = G(x(θ), y(θ), z(θ)) · T(θ) dθ = √ cos θ dθ + 2 sen θ cos θ dθ
Γ 0 2 2 0 0
µ ¯ ¶
1 ¯ π/2 1
= √ sen θ|π/2
0 + sen2 θ¯ =√ .
2 2 0 2
Z
• G · ds.
S
El valor que el potencial vector G toma sobre el segmento S es:
1 1
G(x(v), y(v), z(v)) = 2
(−v , 1 , 0) = q (−v , 1 , 0)
2 − 4 v + 4v (2v − 1)2 + 1
y la tangente al segmento que se corresponde con la parametrización considerada es:
µ ¶
1
T(v) = 1 , − , 1 .
2
Se tiene entonces,
µ ¶
2v + 1 1
G(x(v), y(v), z(v)) · T(v) = − q
(2v − 1)2 + 1 2

luego:
Z Z 1
1 Z1 2v + 1
G · ds = G(x(v), y(v), z(v)) · T(v) dv = − q dv
S 0 2 0 (2v − 1)2 + 1
1 Z1 4v − 2 Z 1
1
=− q dv − q dv
4 0 (2v − 1)2 + 1 0 (2v − 1)2 + 1
¯1
1q ¯ 1 Z ArgSh(1) ³ √ ´
=− (2v − 1)2 + 1¯¯ − dt = − ArgSh(1) = − log 1 + 2 .
4 0 2 −ArgSh(1)
donde la primera de las integrales que aparecen en la segunda lı́nea es inmediata y la segunda se
calcula mediante el cambio de variable: 2v − 1 = Sh(t).
Finalmente, el flujo pedido es:
ZZ ZZ I Z Z ³ √ ´
1 1
F·dσ = ∇×G·dσ = G·ds = G·ds + G·ds = √ −ArgSh(1) = √ −log 1 + 2 .
Σ1 Σ1 Λ Γ S 2 2
(1 punto)

3) Supuesto que la superficie Σ2 no pasa por el origen, solamente existen dos situaciones posibles:
A) El recinto acotado V ⊂ R3 limitado por Σ2 no contiene el origen.
En tal caso, el campo F es solenoidal en V con lo que el teorema de Gauss, según el cual
ZZ ZZZ
F · dσ = ∇ · F dxdydz
Σ2 + V

nos permite concluir que el flujo es nulo, siendo Σ+ 2 la cara exterior de Σ2 .


3
B) El recinto acotado V ⊂ R limitado por Σ2 sı́ contiene el origen.
En tal caso y cualquiera que sea la superficie regular Σ2 , siempre se puede encontrar una superficie
elipsoidal, S, de ecuación cartesiana
x2 + 8y 2 + z 2 = ε2
tal que el elipsoide acotado, E, que limita contiene el origen y, tanto S como E están contenidos en
V . Es decir, se tiene (0, 0, 0) ∈ E ⊂ V ⊂ R3 .
Ahora bien, el conjunto V \E tiene por frontera las superficies Σ2 y S y, además, V \E no contiene
el origen, con lo que el campo F es solenoidal en él. Aplicando de nuevo el teorema de Gauss se obtiene:
ZZ ZZ ZZZ ZZ ZZ
F · dσ + F · dσ = ∇ · F dxdydz = 0 =⇒ F · dσ = F · dσ ,
Σ+
2 S− V \E Σ+
2 S+

donde, al igual que antes, Σ+ 2 es la cara exterior de Σ2 , S



es la cara interior de S y S + es la cara
exterior de S.
Ası́ pues, cualquiera que la sea la superficie regular Σ2 , calcular el flujo pedido consiste en calcu-
larlo a través de la cara exterior de la superficie elipsoidal mencionada.
A continuación se detallan tres posibles formas de proceder:
a) Sabemos que
Z Z Z Z
(z, xz, −x(1 + 8y)) (z, xz, −x(1 + 8y)) 1 Z Z
dσ = dσ = (z, xz, −x(1 + 8y)) dσ
Σ+
2 (x2 + 8y 2 + z 2 )3/2 S+ (x2 + 8y 2 + z 2 )3/2 ε3 S+

pues x2 + 8y 2 + z 2 = ε2 sobre S. Ahora, como el campo (z, xz, −x(1 + 8y)) es de clase 1 en R3 y
además es solenoidal, se puede aplicar Gauss y obtener que el flujo pedido es cero. Obsérvese que,
como era de esperar, el flujo no depende del parámetro ε.
b) Otra forma de razonar, sin aplicar una segunda vez el teorema de Gauss, es utilizar que un
vector normal a la superficie cerrada S es el gradiente de la ecuación h(x, y, z) = 0 que la describe.
Para S tiene la expresión:
N(x, y, z) = (2x , 16y , 2z) ,
y, en este caso, tiene la dirección y sentido de la normal exterior.
Resulta entonces que el campo F es tangente a la superficie elipsoidal en cada punto, ya que:

(z , x z , −x(1 + 8y))
F(x, y, z) · N(x, y, z) = · (2x , 16y , 2z)
(x2 + 8y 2 + z 2 )3/2
1
= 2 (2xz + 16xyz − 2xz(1 + 8y)) = 0 ,
(x + 8y 2 + z 2 )3/2

con lo que también en este caso B) el flujo pedido es cero.


c) Otra posibilidad es proceder utilizando una parametrización P(u, v) = {x(u, v), y(u, v), z(u, v)}
de S; por ejemplo, la dada por las ecuaciones:
ε
x(u, v) = ε cos u sen v , y(u, v) = √ sen u sen v , z(u, v) = ε cos v , u ∈ [0, 2π] , v ∈ [0, π] .
2 2
La normal exterior es:
à !
∂P ∂P ε2 ε2
× = √ cos u sen2 v , ε2 sin u sin2 v , √ cos v sen v
∂v ∂u 2 2 2 2

y tras algunas operaciones se comprueba nuevamente que:


à !
∂P ∂P
F(x(u, v), y(u, v), z(u, v)) · × = 0,
∂v ∂u
lo que de nuevo conduce a la conclusión de que el flujo en este caso B) también es cero y, como era
de esperar, no depende del parámetro ε.
(1 punto)
AMPLIACIÓN DE CÁLCULO (Curso 2007/2008) Convocatoria de septiembre 02.09.08

NOMBRE . . . . . . . . . . . . . . . . . . . . . . . . . APELLIDOS . . . . . . . . . . . . . . . . . . . . . . . . . . . . . . . . . . . . . . . . . . .
Número de matrı́cula. . . . . . . . . . . . . . . . . . . . . . . .

PROBLEMA 1 (4 puntos)

• 1. (1.25 ptos.) Sea p un número real. Estudiar con todo rigor la convergencia y la convergencia
absoluta de Z ∞
senx
Hp := dx
0 xp
Para resolver este apartado se pueden utilizar los siguientes resultados:
R∞
– (a) 1 senx xp
dx diverge si p ≤ 0
R ∞ |senx|
– (b) 1 xp dx converge si p > 1
R 1 |sen 1−x
x
|
– (c) 0 x(1−x) dx diverge

• 2. (1.5 ptos.) Sea n un entero positivo, sea Mn el recinto acotado del primer cuadrante de R2
limitado por las curvas de ecuaciones

xy = 1
xy = 2
y = nx
1
y = x
n
y considérese la integral Z Z
xβ+2 y
In := sen( )dxdy
Mn yβ x
donde β es un número real.
Se pide expresar In en términos de una integral simple y estudiar para qué valores de β el lı́mite
limn→∞ In existe y es finito.

• 3. (1.25 ptos.) Estudiar razonadamente la convergencia de la integral


Z Z
xβ+2 y
J= β
sen( )dxdy
M y x
donde
M = {(x, y) : 1 ≤ xy ≤ 2 ; x, y ≥ 0}

Respuesta: Se entregará esta hoja y, a lo sumo, una adicional.


1. La integral Hp es impropia porque el intervalo de integración es no acotado y, para ciertos
valores de p, porque la función subintegral no está acotada en el entorno de x = 0. Por ello Hp
converge (absolutamente) si y sólo si convergen (absolutamente) las dos integrales
Z ∞ Z 1
senx senx
Ap := dx y B p := dx
1 xp 0 xp
Por (a) Ap diverge si p ≤ 0. Además Ap converge para p > 0 por el criterio de Abel-Dirichlet. En
efecto, para p > 0 la función 1/xp es C 1 [1, ∞) estrictamente decreciente y tiende a 0 cuando x → ∞,
mientras que la función sen(x) tiene primitiva acotada (es decir, − cos x es una función acotada).
En cuanto a la convergencia absoluta de Ap , por (b) sabemos que Ap converge absolutamente si
x t
p > 1. Por otro lado de (c) se tiene, haciendo el cambio de variable t = 1−x , es decir, x = t+1 (que
es inyectivo en (0, 1))
Z 1 ¯¯ x ¯
¯ Z ∞
sen 1−x |sent|
dx = · · · = dt
0 x(1 − x) 0 t
donde la última integral sólo es impropia por ser no acotado el intervalo de integración. Como la
primera integral diverge deducimos que Ap no converge absolutamente para p = 1. Para p ≤ 0, Ap
no converge por lo que no puede converger absolutamente. Queda por estudiar el caso p ∈ (0, 1). Si
p ∈ (0, 1) podemos acotar
|senx| |senx|
0≤ ≤ , ∀x > 1
x xp
R∞ R ∞ |senx|
y como 1 |senx|
x
dx diverge también lo hace 1 xp
dx con p ∈ (0, 1). En definitiva Ap converge
⇔ p > 0, y Ap converge absolutamente ⇔ p > 1.
Por otro lado, puesto que senx/xp es no negativa en (0, 1), Bp converge si y sólo si converge
absolutamente. Además podemos aplicar el criterio del cociente
senx
xp senx
lim+ 1 = lim+ =1
x→0
xp−1
x→0 x

y por tanto Bp converge (y además lo hace absolutamente) si y sólo si p − 1 < 1, es decir, si y sólo
si p < 2.
En definitiva, Hp converge si y sólo si p ∈ (0, 2) y Hp converge absolutamente si y sólo si p ∈ (1, 2)
2. In es una integral doble en sentido propio. Haciendo el cambio

xy = u
y
= v
x
(que es inyectivo en Mn ) o bien

x = u1/2 v −1/2
y = u1/2 v 1/2

para el que el jacobiano queda


1
|Jφ(u, v)| =
2 |v|
y teniendo en cuenta que φ−1 (Mn ) es el rectángulo [1, 2] × [ n1 , n] se obtiene
Z Z µZ 2 ¶ ÃZ n
! Z n
1 u 1 senv 3 senv
In = β+2
senvdudv = udu dv = dv
2 1
[1,2]×[ n ,n] v 2 1 1
n
v β+2 4 1
n
v β+2

Ahora Z ∞
3 senv
lim In = dv
n→∞ 4 0 v β+2
y por ello el lı́mite existe y es finito cuando la integral
Z ∞
senv
dv
0 v β+2
converge, es decir, usando el apartado 1, cuando

0<β+2<2

o bien
−2 < β < 0
3. J es una integral doble impropia tanto porque el dominio de integración M está no acotado
como porque la función subintegral puede no estar acotada en el dominio. Por ello J será convergente
si y sólo si el lı́mite Z Z
xβ+2 y
lim β
sen( )dxdy
n→∞ Bn y x
existe, es finito y es independiente de la sucesión básica de integración Bn que se considere para J.
La función subintegral no conserva el signo, por lo que el lı́mite puede, en principio, depender de la
sucesión básica utilizada. Puesto que en Rn con n ≥ 2 la convergencia de una integral es equivalente
a la convergencia absoluta de la misma, se tiene que J converge si y sólo si lo hace la integral
Z Z
xβ+2 ¯¯ y ¯¯
K := β
¯sen( )¯ dxdy
M y x
Como ahora la función subintegral es no negativa y claramente Mn es una sucesión básica para la
integración en M , se tiene que K es convergente si y sólo si existe y es finito
Z Z
xβ+2 ¯¯ y ¯¯
lim β
¯sen( )¯ dxdy
n→∞ Mn y x

Haciendo el mismo cambio del apartado anterior se obtiene


Z Z Z
xβ+2 ¯¯ y ¯¯ 3 ∞ |senv|
lim β
¯sen( )¯ dxdy = dv
n→∞ Mn y x 8 0 v β+2

y por ello, usando el apartado 1, K converge si y sólo si

1<β+2<2

o bien
−1 < β < 0
En definitiva, J converge si y sólo si −1 < β < 0.
AMPLIACIÓN DE CÁLCULO (Curso 2007-08) Convocatoria de septiembre 02.09.08

PROBLEMA 2 (3 puntos)
En el plano R2 se considera la región D del primer cuadrante encerrada por las parábolas de ecuaciones
x2 = y, x = y 2 ; sea Σ la superficie de revolución que se obtiene al girar la curva frontera de D alrededor
del eje OX.
1. Calcular el centroide de D. (1, 25 puntos)
2. Sea F(x, y, z) = (2xy − x2 , x − y 2 , 2xz + z). Calcular el flujo del campo F a través de la
superficie Σ orientada según el vector normal saliente. (1, 75 puntos)

Respuesta:
1. Los puntos de intersección de ambas parábolas son (0, 0) y (1, 1), ası́ que la región puede
describirse como:

D = {(x, y) ∈ R2 / 0 ≤ x ≤ 1, x2 ≤ y ≤ x}
Dado que D es simétrica respecto de la bisectriz del primer cuadrante su centroide cg (D) =
(xg , yg ) está situado sobre la bisectriz –es decir, xg = yg – y su área es el doble de la parte D∗
situada debajo de la bisectriz, esto es:
D∗ = {(x, y) ∈ R2 / 0 ≤ x ≤ 1, x2 ≤ y ≤ x}
Por tanto,
ZZ Z 1 Z x  Z 1 1
A(D) = 2A(D∗ ) = 2 dx dy = dy dx = 2 (x − x2 ) dx = .
D∗ 0 x2 0 3
Calculemos la componente xg del centroide:
√ !
ZZ Z 1 Z x Z 1   2 1 3
A(D)xg = x dx dy = x dy dx = x3/2 − x3 dx = − = .
D 0 x2 0 5 4 20
9
Se concluye que el centroide de D es cg (D) = (1, 1).
20
2. Dado que la superficie es cerrada no está de más calcular la divergencia del campo:
div F(x, y, z) = 2y − 2x − 2y + 2x + 1 = 1.
Como la divergencia es constante, el teorema de Gauss –que puede aplicarse ya que F es
un campo C ∞ y Σ es unión de dos superficies regulares– nos permite calcular la integral de
superficie (con orientación saliente) como el volumen del sólido Ω de frontera Σ:
Z ZZZ ZZZ
F= div F(x, y, z) dxdydz = dxdydz = V (Ω).
Σ Ω Ω
Ahora bien, al ser el sólido de revolución, su volumen puede calcularse aplicando el teorema de
Guldin:

El volumen engendrado por un recinto plano que gira alrededor de un eje no secante y situado
en su mismo plano es igual al producto del área del recinto por la longitud de la circunferencia
descrita por su centro de gravedad.

En nuestro caso, si giramos D alrededor de OX el radio de la circunferencia descrita por cg es


la componente yg , luego:
Z
9 1 3π
F = V (Ω) = 2πyg A(D) = 2π = .
Σ 20 3 10
Si no se recuerda el teorema de Guldin, para el cálculo de un volumen de revolución de un arco
y = f (x), x ∈ [a, b] alrededor del eje OX podemos también usar la conocida fórmula
Z b
V =π f 2.
a

En nuestro caso, al volumen engendrado por el arco y = x hay que restarle el correspondiente a
y = x2 :
Z 1 3π
V (Ω) = π (x − x4 ) dx = .
0 10

Como alternativa al segundo apartado, si no se aplica el teorema de Gauss puede hacerse directamente
la integral de superficie; dado que la curva que gira está formada por dos arcos regulares, la superficie
habrá de parametrizarse
√ a través de dos funciones. Llamemos Σ1 a la superficie generada al girar
el arco y = x, x ∈ [0, 1] alrededor del eje OX y Σ2 a la relativa al arco y = x2√. Entonces una
parametrización de Σ1 se obtiene premultiplicando un punto genérico del arco (t, t, 0), t ∈ [0, 1]
por la correspondiente matriz de giro de ángulo θ, θ ∈ [0, 2π]:
      
x(θ, t) 1 0 0 t t
 √   √
σ1 (θ, t) =  y(θ, t)  =  0 cos θ − sen θ   t  =  √t cos θ  .
   

z(θ, t) 0 sen θ cos θ 0 t sen θ

El vector normal asociado a esta parametrización es

∂σ1 (θ, t) ∂σ1 (θ, t) 1 √ √


N1 (θ, t) = × = (− , t cos θ, t sen θ)
∂θ ∂t 2
Si tomamos, por ejemplo, θ = 0, t = 1/2 se tiene N1 (0, 1/2) = (−1/2, 1/2, 0) que apunta al exterior
de Σ por lo que orienta adecuadamente la superficie. Análogamente, para Σ2 se tiene
      
x(t, θ) 1 0 0 t t
 2   2
σ2 (t, θ) =  y(t, θ)  =  0 cos θ − sen θ   t  =  t cos θ 
  

z(t, θ) 0 sen θ cos θ 0 t2 sen θ

Obsérvese que hemos cambiado el orden de los parámetros para obtener la orientación adecuada;
ahora el vector normal es

N2 (t, θ) = (2t3 , −t2 cos θ, −t2 sen θ)

La integral de superficie pedida es:


Z Z Z
F= F+ F.
Σ Σ1 Σ2

Hagamos con cierto detalle la integral del campo a través de Σ1 :


Z ZZ
F= F(σ1 (θ, t)) · N1 (θ, t) dθ dt.
Σ1 [0,2π]×[0,1]

Al sustituir y hacer el producto escalar se obtiene


√ 1 √ √
F(σ1 (θ, t)) · N1 (θ, t) = (2t t cos θ − t2 , t − t cos2 θ, (2t + 1) sen θ) · (− , t cos θ, t sen θ)
2
t2 √
= − t t cos3 θ + (2t + 1)t sen2 θ,
2
e integrando:

t2
!
Z ZZ
3
F= + (2t + 1)t sen2 θ dθ dt = π.
Σ1 [0,2π]×[0,1] 2 2

De forma totalmente análoga (no nos detenemos en los detalles),


Z ZZ
6
F= F(σ2 (t, θ)) · N2 (t, θ) dt dθ = − π.
Σ2 [0,1]×[0,2π] 5
Y, para terminar:
Z Z Z
3 6 3
F= F+ F = π − π = π.
Σ Σ1 Σ2 2 5 10

Gabriela Sansigre.
AMPLIACIÓN DE CÁLCULO (Curso 2007/2008) Convocatoria de septiembre 02.09.08

NOMBRE . . . . . . . . . . . . . . . . . . . . . . . . . APELLIDOS . . . . . . . . . . . . . . . . . . . . . . . . . . . . . . . . . . . . . . . . . . .
Número de matrı́cula. . . . . . . . . . . . . . . . . . . . . . . .

PROBLEMA 3 (3 puntos)
Se considera el campo vectorial
!
x2 y xy 2
F = yf (r) − 3 , xf (r) − 3
r r

definido en R2 \{0}, siendo r := x2 + y 2. Se pide:
(1) Determinar una función f : R\{0} → R que haga al campo F conservativo en algún
dominio. (1,5 puntos)
(2) Para la función f hallada en el apartado anterior, calcular un potencial escalar de F .
(1,5 puntos)

Respuesta: Se entregará esta hoja y, a lo sumo, una adicional.


(1) Sea
x2 y xy 2
P := yf (r) − , Q := xf (r) − .
r3 r3
Derivando,
∂P f ′ (r) x2 r 3 − 3rx2 y 2
= f (r) + y 2 −
∂y r r6
∂Q f ′ (r) y 2 r 3 − 3rx2 y 2
= f (r) + x2 − .
∂x r r6
Igualando las derivadas:
f ′ (r) x2 r 2 − 3x2 y 2 2 f (r)

y 2 r 2 − 3x2 y 2
y2 − = x −
r r5 r r5
2 2 ′ 2 2 2 ′ 2
⇒ y r f (r) − x = x r f (r) − y
⇒ r 2 (y 2 − x2 )f ′ (r) = x2 − y 2
1
⇒ f ′ (r) = − 2
r
1
⇒ f (r) = + k .
r
Podemos elegir k = 0, y tenemos
1
f (r) = .
r

(2) En todo punto de la forma (1, y), el campo F está bien definido. Un potencial escalar de F
será:
Z Z x
U(x, y) = Q(1, y) dy + P (s, y) ds
1
Z " # " #
1 y2 x y s2 y
Z
= √ − dy + √ 2 − ds .
1 + y 2 (1 + y 2)3/2 1 s + y 2 (s2 + y 2 )3/2
| {z } | {z }
=I =J

1
Se tiene
1 y2 1 + y2 − y2 1
√ − = = ,
1+y 2 (1 + y )
2 3/2 (1 + y )
2 3/2 (1 + y 2 )3/2
de donde
dy
Z
I= (cambio y = sh η)
(1 + y 2 )3/2
ch η dη dη y
Z Z
= 2 = 2 = th η = √
(1 + sh η)3/2 ch η 1 + y2
ya que
sh2 η sh2 η y2
th2 η = = = .
ch2 η 1 + sh2 η 1 + y2
Por otra parte,
y s2 y s2 + y 2 − s2 y3
√ − = y = ,
s2 + y 2 (s2 + y 2 )3/2 (s2 + y 2)3/2 (s2 + y 2 )3/2
de donde
!
Z x y 3 ds Z x
ds s
J= = !2 3/2 cambio = z
(s2 + y 2)3/2 y

1 1
1 +
s 
y
" #z=x/y
x 1
x/y y dz z y y
Z
= = y √ =ys −y s
1/y (1 + z 2 )3/2 1 + z2 z=1/y x 2 1
1+ 2 1+ 2
y y
xy y
=√ −√ .
x2 +y 2 1 + y2
Por tanto,
xy xy
U(x, y) = I + J = √ = .
x2+y 2 r

2
AMPLIACIÓN DE CÁLCULO (Curso 2008/2009) Convocatoria de Febrero 03.02.09

NOMBRE . . . . . . . . . . . . . . . . . . . . . . . . . APELLIDOS . . . . . . . . . . . . . . . . . . . . . . . . . . . . . . . . . . . . . . . . . . .
Número de matrı́cula. . . . . . . . . . . . . . . . . . . . . . . .

PROBLEMA 1 (3 puntos)
Siendo a > 0 una constante, se considera el arco Γ de la curva de ecuación
x4 − 2ax3 + 4a2 y 2 = 0

contenido en el primer cuadrante (Figura 1).


y
Γ

D
0 V x

Figura 1: Curva x4 − 2ax3 + 4a2 y 2 = 0 en el primer cuadrante

Se pide:
(1) Hallar el área del dominio D limitado por la curva Γ y el eje OX. (1 punto)
(2) Si (x0 , y0) designa el centroide del domino D, hallar la ordenada y0 (no se pide x0 por razón
de brevedad). (1 punto)
(3) Dado el campo vectorial
!
ax3 y x
3
F = x log(1 + y ) + e sen y + x , 2
+ ex cos y + 5xy ,
1+y 2

calcular la circulación de F a lo largo del arco Γ recorrido desde el punto V hasta el origen O. Para
responder a esta pregunta se sugiere utilizar, de manera adecuada, el teorema de Green. (1 punto)

Respuesta: Se entregará esta hoja y, a lo sumo, una adicional.


(1) Los puntos de corte de Γ con el eje OX se obtienen haciendo y = 0 en la ecuación de la curva,
y son:
O(0, 0) y V (2a, 0) .
Despejando y en la ecuación de la curva, resulta
1q
y= (2a − x)x3 .
2a
El área de D será
Z 2a 1q 1
Z 2a √
Área(D) = (2a − x)x3 dx = x 2ax − x2 dx .
0 2a 2a 0

Como
2ax − x2 = a2 − (x − a)2 ,
haremos en la integral el cambio de variable
x−a
t = arc sen ⇒ x − a = a sen t ⇒ dx = a cos t dt
a
1
π π
x=0 ⇒ t=− , x = 2a ⇒ t= ,
2 2
de modo que
1 π/2
Z √
Área(D) = a(1 + sen t) a2 − a2 sen2 t a cos t dt
2a −π/2
a2 Z π/2
= (1 + sen t) cos2 t dt
2 −π/2
a2 π/2
Z
= (cos2 t + sen t cos2 t) dt
2 −π/2
Z π/2
= a2 cos2 t dt
0
2
πa
= .
4

(2) La ordenada del centroide de D es


1 ZZ
y0 = y dxdy .
Área(D) D
Calculamos, pues, la integral
ZZ
J:= y dxdy
D
1

Z 2a Z
2a
(2a−x)x3
= dx y dy
0 0
1 2a Z
= (2a − x)x3 dx
8a2 0
1
Z 2a
= 2 (2ax3 − x4 ) dx
8a 0
" #x=2a
1 ax4 x5
= 2 −
8a 2 5 x=0
!
1 24 a5 25 a5
= 2 −
8a 2 5
1 2
 
= 2a3 −
2 5
a3
= .
5
Por tanto,
J 4a
y0 = = .
Área(D) 5π

(3) Sea C := Γ ∪ OV la curva obtenida cerrando el arco Γ con el segmento OV . Aplicaremos la


fórmula de Green-Riemann al campo F en el dominio D.
Si hacemos F = (P, Q), se tiene
∂Q ∂P 3ax2 y x 2x3 y
− = + e cos y + 5y − − ex cos y
∂x ∂y 1+y 2 1+y 2
2
x y(3a − 2x)
= + 5y .
1 + y2

2
Por la fórmula de Green-Riemann,
!
Z
∂Q ∂P
ZZ
x F · dr = − dxdy
C D ∂x ∂y
x2 y(3a − 2x)
ZZ ZZ
= dxdy +5 y dxdy ,
D 1 + y2 | D {z }
| {z }
:=I =J

donde la segunda integral coincide con la integral J del apartado (2).


Calculamos la primera:
Z 1 √(2a−x)x3
y
Z 2a
2 2a
I= x (3a − 2x) dx dy
0 0 1 + y2

y= 1 (2a−x)x3
Z 2a
1

2a
= x2 (3a − 2x) log(1 + y 2) dx
0 2 y=0
1 2a 1
Z  
= (3ax2 − 2x3 ) log 1 + 2 (2a − x)x3 dx .
2 0 4a
Sea
(2a − x)x3 2ax3 − x4
f (x) := 1 + = 1 + .
4a2 4a2
Entonces,
6ax2 − 4x3 1
f ′ (x) = = (3ax2 − 2x3 ) ,
4a2 2a2
de donde
1 Z 2a 2 ′
I= 2a f (x) log f (x) dx
2 0
= a2 [f (x) log f (x) − f (x)]x=2a
x=0
=0

ya que f (0) = f (2a) = 1.


Por tanto, Z
x F · dr = I + 5J = a3 .
C
Por otra parte, Z Z Z
x F · dr = x F · dr + −
−→ F · dr .
C Γ OV
−−→
Parametrizamos el segmento OV ,
( (
x(t) ≡ t x′ (t) ≡ 1

y(t) ≡ 0 y ′ (t) ≡ 0 ,

de donde
Z Z 2a

−→ F · dr = [P (t, 0)x′(t) + Q(t, 0)y ′(t)] dt
OV 0
" #2a
Z 2a t2
= t dt = = 2a2 ,
0 2 0

luego Z Z Z
x F · dr = x F · dr − −
−→ F · dr = a3 − 2a2 = a2 (a − 2) .
Γ C OV

3
Nota 1. El cálculo de x0 , que no se pide en el problema, es también sencillo utilizando el mismo
cambio de variable x − a = a sen t del apartado (1). Sabemos que
1
ZZ
x0 = x dxdy .
Área(D) D

Calculamos la integral doble


ZZ
H:= x dxdy
D
1

Z 2a Z
2a
(2a−x)x3
= x dx dy
0 0
1 2a 2 √
Z
= x 2ax − x2 dx
2a 0
1 π/2
Z √
= (a + a sen t)2 a2 − a2 sen2 t a cos t dt
2a −π/2
a3 π/2
Z
= (1 + sen t)2 cos2 t dt
2 −π/2
a3 Z π/2
= (cos2 t + 2 sen t cos2 t + sen2 t cos2 t) dt
2 −π/2
!
a3 π
Z π/2
= +2 sen2 t cos2 t dt
2 2 0

a3 π 3 3
  
= +B ,
2 2 2 2
3 
a π π

= +
2 2 8
5πa3
= ,
16
de donde
5πa3 πa2 5a
x0 =
: = .
16 4 4
Nota 2. Claramente, la curva completa es simétrica respecto del eje OX, y su gráfica puede verse
en la Figura 2.
y

0 x

Figura 2: Curva x4 − 2ax3 + 4a2 y 2 = 0 completa

4
AMPLIACIÓN DE CÁLCULO (Curso 2008/2009) Convocatoria de febrero 03.02.09

NOMBRE . . . . . . . . . . . . . . . . . . . . . . . . . APELLIDOS . . . . . . . . . . . . . . . . . . . . . . . . . . . . . . . . . . . . . . . . . . .
Número de matrı́cula. . . . . . . . . . . . . . . . . . . . . . . .

PROBLEMA 2 (4 puntos)
Nota: Los apartados de este problema son independientes.
En R3 se considera la circunferencia definida por la intersección del plano x = 0 con la superficie
cilı́ndrica y 2 + (z + 1)2 = 4.
1. Sea Γ1 el arco de esta circunferencia limitado por los planos z = 1 y z = −1, situado en el
semiespacio y ≥ 0. Sobre Γ1 se considera definida una densidad lineal de masa que en cada
punto es proporcional al cuadrado de la distancia del punto al eje OY . Calcúlese el momento
de inercia de Γ1 con respecto al eje OZ. (1 punto.)
2. Sea Γ2 el arco de la mencionada circunferencia situado en el semiespacio y ≥ 0 y limitado por
los planos z = 0 y z = 1. Sea Σ la superficie de revolución que se obtiene al girar Γ2 alrededor
del eje OZ. Calcúlese la masa de Σ si sobre ella se considera definida una densidad superficial
de masa que en cada punto es proporcional a la distancia al origen del plano tangente a la
superficie en ese punto. (2 puntos.)
3. Calcular la masa del dominio acotado Ω ⊂ R3 limitado por la superficie Σ definida en el
apartado anterior y por el plano z = 0, sabiendo que la densidad de masa por unidad de
volumen en cada punto de Ω es proporcional a la distancia de ese punto al plano XY .
(1 punto.)

Respuesta: Se entregará esta hoja y, a lo sumo, una adicional.


1. La curva Γ1 está contenida en el plano x = 0, por tanto las distancias de uno de sus puntos
(x, y, z) a los ejes OY y OZ son, respectivamente, dOY (x, y, z) = |z|, dOZ (x, y, z) = |y| . La densidad
lineal de masa, DΓ1 , definida sobre Γ1 tiene entonces la expresión
DΓ1 (x, y, z) := [dOY (x, y, z)]2 = k |z|2 ,
donde k > 0 es la constante de proporcionalidad.
Teniendo estas expresiones en cuenta, resulta que (por definición) el momento de inercia de Γ1
con respecto al eje OZ viene dado por la siguiente integral curvilı́nea:
Z Z Z
IOZ (Γ1 ) = d2OZ DΓ1 ds = k |y|2 |z|2 ds = k y 2 z 2 ds .
Γ1 Γ1 Γ1

NOTA: No es conveniente aplicar en este problema el teorema de Steiner, dada la ausencia de


simetrı́as en el arco.
Para calcular la integral en cuestión, sea r(t) = (x(t), y(t), z(t)), t ∈ [a, b], una parametrización
de Γ1 , en la que siempre es x(t) = 0 puesto que el arco está contenido en el plano Y Z. Entonces:
Z Z b Z b q
2 2 2 2 0
IOZ (Γ1 ) = k y z ds = k y(t) z(t) kr (t)k dt = k y(t)2 z(t)2 [y 0 (t)]2 + [z 0 (t)]2 dt .
Γ1 a a

Resta entonces parametrizar Γ1 que es un arco de la circunferencia de ecuación cartesiana:


x = 0, y 2 + (z + 1)2 = 4 ,
definido por las condiciones: y ≥ 0, −1 ≤ z ≤ 1.

1
Para ello existen varias alternativas. Dos de las más razonables se exponen a continuación.
A) Coordenadas polares en el plano Y Z.
Haciendo una traslación, que lleve el punto (0, 0, −1) al origen, y utilizado entonces coordenadas
polares en el plano Y Z, unas ecuaciones paramétricas para el arco Γ1 son:
x(θ) = 0 , y(θ) = 2 cos θ , z(θ) = −1 + 2 sen θ , θ ∈ [0, π/2] ,
donde los valores θ = 0 y θ = π/2 corresponden a los puntos de corte de la circunferencia con los
planos z = −1 y z = 1, respectivamente, y se ha tenido en cuenta que el arco se encuentra en el
semiespacio y ≥ 0.
Se tiene entonces:
r0 (θ) = (x0 (θ), y 0 (θ), z 0 (θ)) = (0 , −2 sen θ , 2 cos θ) =⇒ kr0 (θ)k = 2 ,
y, por tanto:
Z Z π/2 Z π/2
IOZ (Γ1 ) = k y 2 z 2 ds = k y(θ)2 z(θ)2 kr0 (θ)k dθ = 8k cos2 θ (2 sen θ − 1)2 dθ
Γ1 0 0
Z π/2 Z π/2 Z π/2
2 2 2
= 32k cos θ sen θ dθ − 32k cos θ sen θ dθ + 8k cos2 θ dθ
0 0 0
µ µ ¶ µ ¶ µ ¶¶ µ ¶
3 3 3 3 1 32
= k 16 B , − 16 B ,1 +4B , = k 2π − + 2π
2 2 2 2 2 3
µ ¶
32
= k 4π − ,
3
R π/2
donde B(p, q) = 2 0 cos2p−1 θ sen2q−1 θ dθ es la función Beta de Euler. Para calcular sus valores se
ha utilizado su relación con la función Gamma de Euler:
√ B(p, q) = Γ(p)Γ(q)/Γ(p + q) junto con los
valores Γ(n) = (n − 1)Γ(n − 1), Γ(1) = 1 y Γ(1/2) = π.
B) Parametrización cartesiana.
Si se utiliza como parámetro la coordenada y, unas ecuaciones paramétricas para el arco Γ1 son:

x(t) = 0 , y(t) = t , z(t) = −1 + 4 − t2 , t ∈ [0, 2] ,
donde, los valores t = 0 y t = 2 corresponden a los valores de la coordenada, y, en los puntos de
corte del arco con los planos z = 1 y z = −1, respectivamente.
Se tiene entonces:
à !
0 0 0 0 t 2
r (t) = (x (t), y (t), z (t)) = 0 , 1 , − √ =⇒ kr0 (θ)k = √ ,
4 − t2 4 − t2
y, por tanto:
Z Z 2 Z 2 ³√ ´2
2 2 2 2 0 1
IOZ (Γ1 ) = k y z ds = k y(t) z(t) kr (t)k dt = 2k t2 4 − t2 − 1 √ dt .
Γ1 0 0 4 − t2
Para calcular esta integral, efectuamos el cambio de variable t = 2 sen θ, dt = 2 cos θdθ, t = 0 ⇒ θ = 0,
t = 2 ⇒ θ = π/2. De ello resulta:
Z 2 ³√ Z π/2 ´2
12
IOZ (Γ1 ) = 2k t 4− −1 √ 2
dt = 8k sen2 θ (2 cos θ − 1)2 dθ
t2
0 4−t 0
µ µ ¶ µ ¶ µ ¶¶ µ ¶
3 3 3 1 3 32
= k 16 B , − 16 B 1 , +4B , = k 4π − .
2 2 2 2 2 3
donde se han vuelto a utilizar las funciones Gamma y Beta de Euler en forma totalmente análoga al
caso anterior. (1 punto).
Nota: También se pude utilizar como parámetro la coordenada z.

2
2. Para obtener la expresión de la densidad definida sobre Σ es necesario determinar la ecuación del
plano a tangente a Σ en cada uno de sus puntos. Si n(x, y, z) representa un vector normal cualquiera
a la superficie en el punto (x, y, z) de la misma, la ecuación cartesiana del plano tangente es:

n(x, y, z) · (X − x, Y − y, Z − z) = 0 ,

donde (X, Y, Z) representan las coordenadas cartesianas de los puntos del plano. La distancia de este
plano al origen y, por tanto, la densidad DΣ (x, y, z) definida sobre Σ, es entonces:

n(x, y, z) · (x, y, z)
DΣ (x, y, z) = C ,
kn(x, y, z)k

donde C > 0 es la constante de proporcionalidad.


En el caso que nos ocupa, la superficie de revolución Σ es el casquete (situado en el semiespacio
z ≥ 0) de la esfera de ecuación cartesiana:

x2 + y 2 + (z + 1)2 = 4 ,

que se obtendrı́a al hacer girar toda la circunferencia alrededor del eje OZ. Un vector normal al
punto (x, y, z) es entonces
³ ´
n(x, y, z) = grad x2 + y 2 + (z + 1)2 − 4 = 2 (x, y, (z + 1)) ,

con lo que la expresión de la densidad es:

n(x, y, z) · (x, y, z) 2 (x, y, (z + 1)) · (x, y, z) x2 + y 2 + z(z + 1)


DΣ (x, y, z) = C = C q = Cq .
kn(x, y, z)k 2 x2 + y 2 + (z + 1)2 x2 + y 2 + (z + 1)2

Ahora bien, dado que la densidad hay que evaluarla sobre Σ q y sus puntos (x, y, z) √
verifican la ecuación
2 2 2 2 2
cartesiana de la esfera, se tiene: x +y +z(z +1) = 3−z y x + y + (z + 1) = 4 = 2 sobre Σ, con
lo que, finalmente, la densidad definida sobre la superficie que debemos considerar es, simplemente:
3−z
DΣ (x, y, z) = C .
2
La masa de Σ viene dada entonces por la integral de superficie:
ZZ ZZ
3−z
M (Σ) = DΣ (x, y, z) dσ = C dσ .
Σ Σ 2
(1 punto).
Para calcularla es necesario parametrizar la superficie que se origina al hacer girar alrededor del
eje OZ el arco de ecuación cartesiana:
q √
x = 0, z= 4 − y2 − 1 , 0≤y≤ 3,

donde el valor y = 3 corresponde al corte del arco con el plano z = 0.
Una de las posibles parametrizaciones, s(r, θ) = (x(r, θ), y(r, θ), z(r, θ)), de Σ viene dada por:
√ √
x(r, θ) = r cos θ , y(r, θ) = r sin θ , z(r, θ) = 4 − r2 − 1 , θ ∈ [0, 2π] , r ∈ [0, 3] .

Resulta entonces:
ZZ ZZ
M (Σ) = DΣ dσ = √ DΣ (s(r, θ)) kn(r, θ)k dθ dr
Σ [0,2π]×[0, 3]

3
ZZ Ã !
3 − z(r, θ)
= C √ kn(r, θ)k dθ dr ,
[0,2π]×[0, 3] 2

donde n(r, θ) es el vector normal asociado a la parametrización; es decir:


à !
∂s(r, θ) ∂s(r, θ) r2 cos θ r2 sen θ 2r
n(r, θ) = × = √ , √ ,r =⇒ kn(r, θ)k = √ .
∂r ∂θ 4 − r2 4 − r2 4 − r2

Por otra parte, la densidad evaluada sobre la superficie tiene la expresión:


à ! √
3 − z(r, θ) 4 − 4 − r2
DΣ (s(r, θ)) = C =C ,
2 2

con lo que la integral doble que hay que calcular es


ZZ ³ √ ´
r
M (Σ) = C √ √ 4 − 4 − r 2 dr dθ

ZZ
[0,2π]×[0, 3] 4 − r2 ZZ
r
= 4C √ √ dr dθ − √ r dr dθ
[0,2π]×[0, 3] 4 − r2 [0,2π]×[0, 3]
 ¯√ 
√ ¯√3 r 2¯ 3
¯ ¯
= 2πC −4 4 − r2 ¯ − ¯  = 5π C .
0 2¯ 0

(1 punto).
3. Unas ecuaciones que describen el dominio Ω ∈ R3 son:
½ q ¾
3 2 2
Ω = (x, y, z) ∈ R : 0 ≤ x + y ≤ 3 , 0≤z≤ 4− x2 − y2 −1 .

donde el cı́rculo 0 ≤ x2 + y 2 ≤ 3 es la proyección de Ω sobre el plano XY y z debe variar entre este


plano y Σ.
Puesto que la densidad definida en Ω es proporcional en cada uno de sus puntos a la distancia al
plano XY , es decir: DΩ (x, y, z) = A|z|, su masa viene dada por la integral triple:
ZZZ
M (Ω) = A|z| dx dy dz ,

donde A > 0 es la constante de proporcionalidad.


Para calcular su valor utilizamos coordenadas cilı́ndricas (r, θ, z):

x = r cos θ , y = r sen θ , z = z , con jacobiano: J = r .

El recinto Ω expresado en estas coordenadas es:


n √ √ o
Ω0 = (r, θ, z) ∈ R3 : 0 ≤ θ ≤ 2π , 0≤r≤ 3, 0≤z≤ 4 − r2 − 1

Como Ω está situado en el semiespacio z ≥ 0, se tiene:


ZZZ ZZZ Z 2π Z √3 Z √4−r2 −1
M (Ω) = A z dx dy dz = A r z dr dθ dz = A dθ r dr z dz
Ω Ω0 0 0 0
 ¯√3 
Z √3
r ³√ ´2 5r2 r4 2 ¯ 7 ¯
= 2π A 4 − r2 − 1 dr = π A  − + (4 − r2 )3/2 ¯¯  = πA.
0 2 2 4 3 0
12

(1 punto).

4
AMPLIACIÓN DE CÁLCULO (Curso 2008/2009) Convocatoria de febrero 03.02.09

PROBLEMA 3 (3 puntos)

Sea F un campo vectorial de clase 1 en un abierto Ω de R3 . Se dice que un campo escalar f : Ω → R de


clase 1 es un factor integrante de F en Ω si rot(f F) = 0. Sea F(x, y, z) = (yz, zx, −xy). Se pide:
1. Calcular un factor integrante f de F que sea solamente función de z, ¿cuál es su dominio de definición?
Calcular asimismo un potencial escalar de f F.
2. Sea Σ la superficie del paraboloide x2 + 9y 2 = z − 1 contenida en el semiespacio z ≤ 9 y orientada
de forma que en (0, 0, 1) su vector normal es k. Calcular el flujo de rot F a través de Σ.
3. Sea Γ la curva parametrizada por
2 π
ϕ(t) = (2 cos t, sen t, 5), t ∈ [0, ]
3 3
Calcular la circulación de F a lo largo de Γ.
Nota: Los apartados 2. y 3. pueden hacerse directamente, pero se valorará especialmente el hacerlos aplicando 1.

Respuesta:
1. Calculemos en primer lugar el rotacional de F:


i j k
rot F(x, y, z) = ∂/∂x ∂/∂y ∂/∂z = (−2x, 2y, 0).

−xy

yz zx
Sea f (x, y, z) = h(z) el factor integrante buscado y denotemos G = f F. Recordemos la fórmula
rot G = ∇f × F + f rot F (véase el problema 6.8 (f)). Calculemos el primer sumando:

i j k

∇f × F(x, y, z) = 0 0 h0 (z) = h0 (z)(−zx, yz, 0).

zx −xy

yz

Entonces
rot G = 0 ⇐⇒ h0 (z)(−zx, yz, 0) + h(z)(−2x, 2y, 0) = 0
⇐⇒ zxh0 (z) + 2xh(z) = yzh0 (z) + 2yh(z) = 0.
Suponiendo x e y no nulos e integrando se tiene
h0 (z) 2 c
= − ⇐⇒ h(z) = 2 , c 6= 0.
h(z) z z
La constante c es arbitraria, podemos tomarla igual a 1 por ejemplo, con lo que el factor integrante
queda f (x, y, z) = 1/z 2 y es válido siempre que z sea no nulo; esto es, en cualquier parte del espacio
que no atraviese el plano de ecuación z = 0.
Supongamos Ω ⊂ R3 \ {z = 0} abierto y simplemente conexo, con ello podemos asegurar que
G(x, y, z) = (y/z, x/z, −xy/z 2 ) admite un potencial escalar g, es decir G = ∇g. Integremos las
ecuaciones resultantes:
∂g y xy
= ⇒ g(x, y, z) = + k(y, z)
∂x z z
∂g x x ∂k ∂k
= = + ⇒ =0
∂y z z ∂y ∂y
∂g xy xy ∂k ∂k
=− 2 =− 2 + ⇒ =0
∂z z z ∂z ∂z
Se concluye que la función k es constante; en particular podemos escogerla nula y tomar g(x, y, z) =
xy/z como potencial escalar.
También podemos integrar directamente el campo G –si nos situamos, por ejemplo en el semiespacio
z > 0– a lo largo de una lı́nea poligonal de segmentos paralelos a los ejes y que una (0, 0, 1) con
(x, y, z); llamando Gj a las componentes de G:
Z x Z y Z z
g(x, y, z) = G1 (t, 0, 1)dt + G2 (x, t, 1)dt + G2 (x, y, t)dt
0 0 1
y z xy xy z xy
Z Z
= xdt + − 2 dt = xy + = .
0 1 t t 1 z

2. La superficie Σ está contenida en el semiespacio z ≥ 1 que es un conjunto estrellado y F es de clase


1, ası́ pues podemos aplicar el teorema de Stokes; sea B el borde de Σ orientado coherentemente; se
tiene
ZZ Z
rot F = F.
Σ B

Ahora bien, la curva B está situada en el plano z = 9, por tanto


1
G(x, y, z) = F(x, y, z), (x, y, z) ∈ B.
81
Se tiene entonces:
Z Z ZZ
F = 81 G = 81 rot G = 0,
B B Σ

donde se ha aplicado de nuevo el teorema de Stokes, esta vez al campo G.


2. (Razonamientos alternativos, sin utilizar el primer apartado)
(a) Consideremos la superficie D que cierra la superficie Σ en el plano z = 9, esto es

D = {(x, y, z) ∈ R3 , x2 + 9y 2 ≤ 8, z = 9}

En virtud del Teorema de Stokes, si un campo de clase 1 es solenoidal en una parte estrellada de R3 ,
su integral de superficie depende de un potencial vector y del borde de la superficie, no de la superficie
propiamente dicha. En nuestro caso Σ y D tienen el mismo borde (la elipse x2 + 9y 2 = 8, z = 9),
el vector unitario normal a D coherente con la orientación que Σ induce en su contorno es k, pero
resulta que rot F · k = (−2x, 2y, 0) · (0, 0, 1) = 0. Concluimos por tanto:
ZZ ZZ
rot F = rot F · k = 0.
Σ D

El razonamiento anterior puede leerse también en clave del Teorema de Gauss: sea Σ− ∪D la superficie
frontera del paraboloide sólido P (comprendido entre la superficie del paraboloide y el plano z = 9),
orientada por el vector normal exterior –obsérvese que Σ y Σ− tienen orientación opuesta–, entonces
ZZ ZZZ ZZ ZZ
rot F = div(rot F) = 0 ⇒ rot F = − rot F = 0.
Σ− ∪D P Σ− D

(b) También puede realizarse directamente la integral de superficie, sin tomar en consideración los teo-
rema de Stokes o Gauss, en tal caso:
Consideremos la parametrización dada por la ecuación implı́cita H(x, y, z) = x2 + 9y 2 − z + 1 = 0
entre z = 1 y z = 9. El vector normal es
∇H
= (−2x, −18y, 1),
∂H/∂z
este vector, al tener la tercera componente positiva, orienta el paraboloide como se nos indica en el
enunciado. El flujo pedido es:
ZZ ZZ ZZ
rot F = (−2x, 2y, 0) · (−2x, −18y, 1) dx dy = (4x2 − 36y 2 ) dx dy
Σ x2 +9y 2 ≤8 x2 +9y 2 ≤8

Hacemos cambio de variables a coordenadas elı́pticas:


1 √ 1
x = ρ cos t, y = ρ sen t, (t, ρ) ∈ [0, 2π] × [0, 8] J= ρ
3 3
y obtenemos:
ZZ
4 ZZ 3 2 2
rot F = √ ρ (cos t − sen t)dρ dt = 0.
Σ 3 [0,2π]×[0, 8]

3. Busquemos una representación implı́cita de la curva Γ (es decir, su traza) eliminando el parámetro
t,

x2 + 9y 2 = 4, z = 5.

Notamos que Γ está sobre la intersección de Σ con el plano z = 5, por tanto


1
G(x, y, z) = F(x, y, z), (x, y, z) ∈ Γ.
25
Podemos calcular la integral de lı́nea con ayuda del potencial g obtenido en el primer apartado,
veamos cuáles son los extremos de la curva:

ϕ(0) = (2, 0, 5), ϕ(π/3) = (1, 3/3, 5)

Entonces:

Z Z Z √ 3
F = 25 G = 25 ∇g = 25(g(1, 3/3, 5) − g(2, 0, 5)) = 5 .
Γ Γ Γ 3

3. (Cálculo directo de la integral de lı́nea)


Z Z π/3 20 Z π/3 20 Z π/3
F= F(ϕ(t)) · ϕ0 (t)dt = (cos2 t − sen2 t)dt = cos 2tdt
Γ 0 3 0
√ 3 0
20 sen 2t π/3
" #
10 3
= = sen(2π/3) = 5 .
3 2 0
3 3

Gabriela Sansigre.
AMPLIACIÓN DE CÁLCULO (Curso 2008/2009) Convocatoria de Junio 09.06.09

NOMBRE . . . . . . . . . . . . . . . . . . . . . . . . . APELLIDOS . . . . . . . . . . . . . . . . . . . . . . . . . . . . . . . . . . . . . . . . . . .
Número de matrı́cula. . . . . . . . . . . . . . . . . . . . . . . .

PROBLEMA 1 (3 puntos)
Este problema tiene por objeto utilizar el teorema de Green para calcular una conocida integral
impropia, que puede verse más abajo, al final de este enunciado.
Se considera el campo vectorial F de clase C 1 definido en R2 por y
Γ3 Γ
 ! R
 e −xy
sen x
,0 si x 6= 0


F (x, y) := x

(1, 0)
Γ4 D Γ2
si x = 0 .

Dado R > 0 arbitrario, sea la curva cerrada Γ = Γ1 ∪ Γ2 ∪ Γ3 ∪ Γ4 de la 0 Γ1 R x


Figura. Se pide:
(1) (0,5 puntos) Mediante una aplicación del teorema de Green, calcular una función f (x, y) tal
que Z Z Z R R
x F · dr = dy f (x, y) dx .
Γ 0
|0 {z }
=:I1 (R,y)

(2) (0,5 puntos) Calcular la integral I1 (R, y).


(3) (0,25 puntos) Integrando I1 (R, y) respecto de la variable y en el intervalo [0, R], obtener dos
funciones g(R) y h(R, y) tales que
Z Z R
x F · dr = g(R) − h(R, y) dy .
Γ 0

(4) (0,5 puntos) Hallar la circulación del campo F a lo largo de cada uno de los segmentos Γ1 ,
Γ2 , Γ3 y Γ4 , todos ellos con la orientación indicada en la Figura. En algunos casos se obtendrá un
valor numérico y en otros una integral en [0, R] que no es posible calcular en términos de funciones
elementales.
(5) (0,25 puntos) Utilizando, si se desea, la desigualdad |(sen x)/x| < 1 para x > 0, obtener una
función ξ(R) que verifique
Z

F · dr ≤ ξ(R) y tal que lı́m ξ(R) = 0 .
R→+∞

Γ3

(6) (0,5 puntos) Encontrar una función η(R) que verifique


Z
R

h(R, y) dy ≤ η(R) y tal que lı́m η(R) = 0 .
0 R→+∞

(7) (0,5 puntos) Como consecuencia inmediata de todo lo anterior, probar la igualdad
∞ sen x π
Z
dx = .
0 x 2
Este problema está tomado de R. L. Robertson, An improper application of Green’s theorem,
College Math. J. 38 (2007), 142–145.

Respuesta: Se entregará esta hoja y, a lo sumo, una hoja adicional.


(1) Haciendo F = (P, Q), tenemos por los teoremas de Green y de Fubini:
∂P
Z ZZ
x F · dr = − dxdy
Γ D ∂y
ZZ
= e−xy sen x dxdy
D
Z R Z R
= dy e−xy sen x dx .
0 0

(2) Integrando dos veces por partes:


!
u = e−xy ⇒ du = −ye−xy dx
dv = sen x dx ⇒ v = − cos x
h ix=R Z R
I1 (R, y) = −e−xy cos x −y e−xy cos x dx
x=0 0
Z R
−Ry
= 1−e cos R − y e−xy cos x dx
|0 {z }
=:I2 (R,y)

(dv = cos x dx ⇒ v = sen x)


h ix=R Z R
I2 (R, y) = e−xy sen x +y e−xy sen x dx
x=0 0
= e−Ry sen R + yI1(R, y) ,

de donde

I1 (R, y) = 1 − e−Ry cos R − y[e−Ry sen R + yI1(R, y)]


1 − e−Ry cos R − ye−Ry sen R
⇒ I1 (R, y) = .
1 + y2

(3)
Z
1 − e−Ry cos R − ye−Ry sen R
Z R
x F · dr = dy
Γ 0 1 + y2
Z R −Ry
e (cos R + y sen R)
= arc tg R − dy .
0 1 + y2

(4) Es claro que


Z Z Z Z Z
x F · dr = F · dr + F · dr + F · dr + F · dr .
Γ Γ1 Γ2 Γ3 Γ4

Parametrizando cada intervalo del modo obvio, se tiene


Z Z Z R sen x
F · dr = P (x, 0) dx = dx ,
Γ1
Z
Γ1
Z
0 x
F · dr = P (R, y) dx = 0 ya que x es constante,
Γ2 Γ2
Z Z 0 e−Rx sen x Z R −Rx
e sen x
F · dr = dx = − dx ,
Γ3
Z
R x 0 x
F · dr = 0 por ser x constante.
Γ4

2
(5) Obtenemos fácilmente
Z
R e−Rx sen x
Z

F · dr =
dx

Γ3 0 x
sen x
Z R
≤ e−Rx dx
0 x
Z R
≤ e−Rx dx
0
1 −Rx x=R
 
= − e
R x=0
−R2
1−e
= −−−−→ 0 .
R R→+∞

(6) Teniendo en cuenta que |cos R + y sen R| ≤ y + 1 para todo y ≥ 0, y que


1
≤ 1 para todo y,
1 + y2
resulta
Z
R e−Ry (cos R + y sen R) R
Z
dy ≤ e−Ry (y + 1) dy

1 + y2

0 0
y=R
1 1 R −Ry
  Z
= − e−Ry (y + 1) + e dy
R y=0 R 0
2 2
1 − e−R (1 + R) 1 − e−R
= + −−−−→ 0 .
R R2 R→+∞

(7) Sustituyendo los valores obtenidos en (4) en la fórmula del apartado (3), resulta:
R sen x R e−Rx sen x R e−Ry (cos R + y sen R)
Z Z Z
dx = arc tg R + dx − dy .
0 x 0 x 0 1 + y2

Tomando lı́mites para R → +∞ en ambos miembros y, teniendo en cuenta los resultados de (5), se
obtiene
sen x π
Z ∞
dx = lı́m (arc tg R) = .
0 x R→+∞ 2

3
AMPLIACIÓN DE CÁLCULO (Curso 2008/2009) Convocatoria de junio 09.06.09

NOMBRE . . . . . . . . . . . . . . . . . . . . . . . . . APELLIDOS . . . . . . . . . . . . . . . . . . . . . . . . . . . . . . . . . . . . . . . . . . .
Número de matrı́cula. . . . . . . . . . . . . . . . . . . . . . . .

PROBLEMA 2 (4 puntos)

Sea Σ la porción de la superficie de ecuación


¡ ¢3/2
x2 + y 2 = (1 − z)y (1)

que pertenece al primer octante de R3 y verifica que z ≤ 1.

1. (1 pto.) Sea A el sólido limitado por Σ y por los 3 planos coordenados. Sobre A hay una
distribución de masa de forma que la densidad en cada punto de A es proporcional a la distancia
del punto al plano xy. Se pide determinar el momento de inercia de A respecto del eje z.

2. (1.5 ptos.) Calcular el flujo φ del campo F(x, y, z) = (y, −x, 0) a través de Σ orientada de
forma que el vector normal en cada punto tenga su primera componente positiva.

3. (1.5 ptos.) Sea Γ la curva intersección de Σ con los planos coordenados. Calcular el campo
escalar g que cumple que g(0, 0, z) = z 2 /2 y que el flujo φ del apartado 2 es igual a la circulación
del campo H(x, y, z) = (x, y, g(x, y, z)) sobre la curva Γ orientada adecuadamente. Calcular
explı́citamente dicha circulación para comprobar el resultado.

Respuesta: Se entregará esta hoja y, a lo sumo, una adicional.

Obsérvese que los puntos de la forma (0, 0, z) son soluciones de la ecuación, lo que quiere decir
que todo el eje z es solución de la ecuación. Para estudiar la forma de la superficie Σ podemos pasar
a cilı́ndricas. Se obtiene
ρ3 = (1 − z)ρsenθ

es decir, ρ = 0 (que corresponde al eje z) ó


√ √
ρ= 1 − z senθ (2)

que corresponde a la superficie Σ. La variación de los parámetros es θ ∈ [0, π/2] y z ∈ [0, 1].
Para ver qué forma tiene esta superficie cortamos por planos z igual a√constante, z = C, 0 ≤ C ≤ 1
obteniendo que (2) define una curva que une los puntos (0, 0, C) y (0, 1 − C, C) cuya ”amplitud”
va decreciendo cuando aumenta z y se reduce al punto (0, 0, 1) cuando z = 1.
Estudiemos también la curva Γ intersección de Σ con los planos coordenados. La intersección Γ1
con el plano XY la podemos obtener√ o bien de (2) o bien de (1) haciendo z = 0. Ası́ obtenemos la
curva con ecuación polar ρ = senθ, θ ∈ [0, π/2] que une los puntos (0, 0, 0) y (0, 1, 0). Haciendo
x = 0 en la (2) se obtiene que la ecuación de su intersección con el plano Y Z es y 3 = (1 − z)y, es
decir, la curva y 2 = 1 − z con z ∈ [0, 1] a la que denominamos Γ2 y la curva Γ3 definida por y = 0,
es decir, la porción del eje z con z ∈ [0, 1]. En cuanto a su intersección con el plano Y Z se obtiene,
haciendo y = 0, que debe ser x = 0 con lo que de nuevo volvemos a obtener la curva Γ3 .
1

z
0

0
0

y
1 x

Apdo. 1. La densidad es ρ(x, y, z) = Kz por lo que


Z Z Z
¡ ¢
I=K z x2 + y 2 dxdydz
A

Procedimiento 1. Fijando z e integrando en x, y. Fijando primero la z e integrando en x y en y


se tiene Z 1 Z Z
¡ 2 ¢
I=K zdz x + y 2 dxdy
0 Πz

donde ΠC es la intersección de A con el plano z = C. Haciendo un cambio a polares (x, y) = Ψ(ρ, θ)


tenemos Z 1 Z Z
I=K zdz ρ2 ρdρdθ
0 Ψ−1 (Πz )

Para obtener Ψ−1 (Πz ) entramos con el cambio a polares


√ en la ecuación de Σ y obtenemos (2) con lo

que, para cada θ ∈ [0, π/2], ρ varı́a entre 0 y ρ = 1 − z senθ. Por tanto
Z Z Z √ √
1 π/2 1−z senθ
I = K zdz dθ ρ3 dρ =
0 0 0
Z 1 Z π/2 µZ 1 ¶ ÃZ π/2
!
1 1
= K zdz (1 − z)2 sen2 θdθ = K z(1 − z)2 dz sen2 θdθ =
4 0 0 4 0 0
Z π/2
K 1 − cos 2θ K 1 π Kπ
= β(2, 3) dθ = =
4 0 2 4 12 4 192

Procedimiento 2. Fijando x, y e integrando en z. La superficie Σ (salvo los puntos para los que
y = 0, que tienen contenido nulo en R2 ) se puede escribir en forma explı́cita mediante

(x2 + y 2 )3/2
z = h(x, y) = 1 − (x, y) ∈ B (3)
y
donde n ¡ ¢3/2 o
B = (x, y) : x2 + y 2 ≤ y y y 6= 0

(excluimos los puntos con y = 0 puesto que en ellos h no está definida, aunque a efectos de integración
es irrelevante incluirlos o no)
Por tanto
Z Z Z Z Z Z 1−
(x2 +y 2 )3/2
¡ ¢ ¡ ¢ y
I = K z x2 + y 2 dxdydz = K dxdy x + y 2 2
zdz =
A B 0
Z Z µ ¶2
K ¡ 2 2
¢ (x2 + y 2 )3/2
= x +y 1− dxdy
2 B y
(x2 +y 2 )3/2
Obsérvese que la integral doble que se obtiene es una integral en sentido propio, pues y
está acotada en B. Haciendo un cambio a polares se obtiene
Z π/2 Z √
senθ µ ¶2
K 3 ρ2
I= dθ ρ 1− dρ
2 0 0 senθ

Para poder calcular la primera integral hacemos un cambio ρ = w(u) = senθu, y se tiene
Z Z ³√ ´3 ¡
K π/2 1 ¢2 √
I = dθ senθu 1 − u2 senθdu =
2 0 0
ÃZ ! µZ ¶ Z
π/2 1 ¡ ¢ 1 ¡ ¢2
K 2 3 2 2 Kπ
= sen θdθ u 1−u du = u3 1 − u2 du
2 0 0 2 4 0

3
Para calcular esta última integral se puede desarrollar√(1 − u2 ) o, de forma más elegante, relacionarla
con una β. Para ello hacemos el cambio u = h(t) = t y se obtiene
Z 1 Z 1 Z 1
¡ ¢2 1 2 −1/2 1 1 1
u 1 − u2 du =
3 3/2
t (1 − t) t dt = t (1 − t)2 dt = β(2, 3) =
0 2 0 2 0 2 24

con lo que
Kπ 1 Kπ
I= =
2 4 24 192
Apdo. 2.
Primer procedimiento. En primer lugar tenemos en cuenta que F es de clase 1 y solenoidal en R3 .
La superficie Σ no es cerrada por lo que no se puede asegurar que el flujo valga cero, pero sı́ se puede
”cerrar” la superficie con otras superficies más cómodas y aplicar Gauss, con lo que pasarı́amos a
integrar sobre dichas superficies. Sea Σ1 la porción de A que pertenece al plano xy (orientada con
la normal n = (0, 0, 1)) y sea Σ2 la porción de A que pertenece al plano yz (orientada con la normal
n = (1, 0, 0)). Entonces podemos escribir
Z Z Z Z Z
φ− F · ds− (y, −x, 0) · ds = divFdxdydz = 0
Σ1 Σ2 A

luego
Z Z
φ = (y, −x, 0) · ds+ (y, −x, 0) · ds =
Σ1 Σ2
Z Z
= (y, −x, 0) · (0, 0, 1)ds+ (y, −x, 0) · (1, 0, 0)ds =
ZΣ 1 Σ2

= yds
Σ2

donde se ha utilizado que (y, −x, 0) · (0, 0, 1) = 0. Claramente esta integral es más cómoda que
calcular el flujo integrando sobre Σ.
Ya hemos visto que la ecuación de la intersección de Σ con el plano x = 0 es y = 0 (que
corresponde al eje z) y y 2 = 1 − z, que es la curva buscada. Por ello Σ2 se puede parametrizar en la
forma

x = 0
© ª
y = y (y, z) ∈ T = (y, z) : 0 ≤ z ≤ 1 − y 2 , y ∈ [0, 1]
z = z
y por tanto
Z Z Z Z 1
1
φ= yds = ydydz = y(1 − y 2 )dy =
Σ2 T 0 4
Segundo procedimiento. Si no nos damos cuenta de lo anterior, debemos calcular el flujo integrando
directamente sobre Σ. Para ello en primer lugar necesitamos parametrizar la superficie.
Primera forma de parametrizar. Usando (2) tenemos, utilizando que x = ρ cos θ, y = ρsenθ, la
parametrización (x, y, z) = Φ(θ, z) , (θ, z) ∈ [0, π/2] × [0, 1] dada por
√ √
x = 1 − z senθ cos θ (4)
√ √ √
y = 1 − z senθsenθ = 1 − zsen3/2 θ (θ, z) ∈ [0, π/2] × [0, 1]
z = z

Ahora ¯ ¯
¯ i j k ¯
∂Φ ∂Φ ¯ √ √ √ ¯
¯ z 1−3sen
2 θ 3 senθ cos θ ¯
N(θ, z) = ± × = ±¯ 1 − √
2 senθ
1 − z 2
0 ¯
∂θ ∂z ¯ √ 3/2 ¯
¯ − √ senθ cos θ
− sen
√ θ
1 ¯
2 1−z 2 1−z

Nótese que como la tercera componente del campo F es idénticamente nula no hace falta calcular la
tercera componente de N. Ası́
µ ¶
3√ √ √ 1 − 3sen2 θ
N=± 1 − z senθ cos θ, − 1 − z √ , N3
2 2 senθ

La orientación del enunciado para Σ hace que tomemos el signo + y por ello se tiene
Z
φ = (y, −x, 0) · ds =
Σ
Z Z ³√ √ √ ´ µ3√ √ √ 1 − 3sen2 θ
3/2
= 1 − zsen θ, − 1 − z senθ cos θ, 0 · 1 − z senθ cos θ, − 1 − z √ , N
[0,π/2]×[0,1] 2 2 senθ
Z Z µ ¶
3 2 1 2 operando
= (1 − z) sen θ cos θ + (1 − z)(1 − 3sen θ) cos θ dθdz =
[0,π/2]×[0,1] 2 2
µZ 1 ¶ ÃZ π/2 !
cos θ 1
= (1 − z)dz dθ =
0 0 2 4

Segunda forma de parametrizar. También podemos utilizar la expresión explı́cita (3) de la super-
ficie, con lo que se tiene la parametrización (x, y, z) = Θ(x, y) , (x, y) ∈ B dada por

x = x n o
¡ ¢3/2
y = y (x, y) ∈ B (x, y) : x2 + y 2 ≤ y y y 6= 0
(x2 + y 2 )3/2
z = h(x, y) = 1 −
y

Entonces
µ ¶
∂Θ ∂Θ ∂h ∂h
N(θ, z) = ± × = ± − (x, y), − (x, y), 1 =
∂x ∂y ∂x ∂y
à !
1/2 1/2
3 (x2 + y 2 ) (x2 + y 2 ) (2y 2 − x2 )
= ± , ,1
y y2
donde elegimos el signo + que es el correspondiente a la orientación definida en el enunciado. Ası́
Z Z Z Ã !
1/2 1/2
3 (x2 + y 2 ) (x2 + y 2 ) (2y 2 − x2 ) operando
φ = (y, −x, 0) · ds = (y, −x, 0) · , 2
, 1 dxdy =
Σ B y y
Z Z p µ ¶
x3 polares
= x2 + y 2 x + 2 dxdy =
B y
Z π/2 Z √senθ µ ¶ Z π/2 µ ¶
3 cos3 θ cos3 θ sen2 θ
= dθ ρ cos θ + dρ = cos θ + dθ =
0 0 sen2 θ 0 sen2 θ 4
Z Z
1 π/2 ¡ 2 3
¢ 1 π/2 1
= cos θsen θ + cos θ dθ = cos θdθ =
4 0 4 0 4
Apdo. 3. Obsérvese que el borde de Σ es precisamente la curva Γ. Puesto que se pide relacionar
el flujo de F con la circulación de H sobre su borde Γ, si H fuese un potencial vector de F de clase
uno en R3 se tendrı́a, usando el teorema de Stokes
Z Z
H · dr = F · ds
Γ Σ

donde la orientación de Γ es la coherente con la de Σ.


Ya sabemos que F admite potencial vector de clase 1 en R3 , pues F es de clase 1 y solenoidal
en el conjunto estrellado R3 . En definitiva, buscamos encontrar un potencial vector H de F de la
forma H(x, y, x) = (x, y, g(x, y, z)) y que cumpla que g(0, 0, z) = z 2 /2. Imponiendo que rotH = F se
obtiene
∂g ∂g
=y ; − = −x ; 0 = 0
∂y ∂x
De la primera igualdad se deduce que g(x, y, z) = y 2 /2 + α(x, z) y entrando en la segunda obtenemos
∂α
∂x
= x luego α(x, z) = x2 /2 + γ(z), es decir, g(x, y, z) = x2 /2 + y 2 /2 + γ(z). Imponiendo g(0, 0, z) =
z 2 /2 se obtiene finalmente g(x, y, z) = x2 /2+y 2 /2+z 2 /2, luego H(x, y, x) = (x, y, x2 /2+y 2 /2+z 2 /2).
Para calcular la circulación sobre Γ, que debemos orientar de forma coherente con Σ, expresamos
Γ en la forma Γ = Γ1 ∪ (−Γ2 ) ∪ (−Γ3 )

Γ1 : intersección de Σ con el plano XY orientada en sentido antihorario (si se mira desde arriba)
Γ2 : intersección de Σ con el plano Y Z (salvo eje z) orientada en sentido horario
(si se mira desde la parte positiva del eje x)
Γ3 : intersección de Σ con el eje z orientada hacia arriba

Entonces Z Z Z Z
H · dr = H · dr − H · dr − H · dr
Γ Γ1 Γ2 Γ3

Parametrización de Γ1 : se puede obtener de la parametrización de Σ (4) haciendo z = 0



x = α(θ) = senθ cos θ (5)
y = β(θ) = sen3/2 θ θ ∈ [0, π/2]
z = γ(θ) = 0

Como γ 0 (θ) = 0 no es necesario escribir la tercera componente de H. Por ello


Z Z Z π/2 ³√ Ã √ !
´ 1 − 3sen 2
θ 3 senθ cos θ operando
H · dr = (x, y, H3 ) · dr = senθ cos θ, sen3/2 θ, H3 · √ , , 0 dθ =
Γ1 Γ1 0 2 senθ 2
Z π/2
cos θ 1
= dθ =
0 2 2
Parametrización de Γ2 : Ya hemos obtenido que la ecuación cartesiana de Γ2 como curva en el
plano x = 0 es y 2 = 1 − z, y ∈ [0, 1]. Por ello podemos parametrizar en la forma

x = 0
y = y y ∈ [0, 1]
z = 1 − y2

Por ello
Z Z Z 1
2 2 2 x=0 en Γ2 ¡ ¢2
H · dr = (x, y, x /2 + y /2 + z /2) · dr = (0, y, y 2 /2 + 1 − y 2 /2) · (0, 1, −2y)dy =
Γ2 Γ 0
Z 21
¡ ¢2 1 1 1 1
= (y − y 3 − y 1 − y 2 )dy = − − =
0 2 4 6 12
Parametrización de Γ3 :

x = 0
y = 0 z ∈ [0, 1]
z = z

y por tanto Z Z 1
1
H · dr= (0, 0, z 2 /2) · (0, 0, 1)dz =
Γ3 0 6
En definitiva Z
1 1 1 1
H · dr= − − =
Γ 2 12 6 4
como debı́amos obtener.
Obsérvese que podrı́amos haber simplificado un poco las operaciones dándonos cuenta de que H
se puede escribir en la forma

H = (x, y, x2 /2 + y 2 /2 + z 2 /2) = (x, y, z 2 /2) + (0, 0, x2 /2 + y 2 /2)

El primer campo es conservativo en R3 (pues es irrotacional en R3 que es un abierto estrellado) con


lo que su circulación a lo largo de una curva cerrada es nula. Por ello
Z Z
H · dr = (0, 0, x2 /2 + y 2 /2) · dr
Γ Γ
R R
y ahora Γ1
(0, 0, x2 /2 + y 2 /2) · dr = Γ3
(0, 0, x2 /2 + y 2 /2) · dr = 0 por lo que
Z Z Z 1
2 2
H · dr = − (0, 0, x /2 + y /2) · dr = − (0, 0, y 2 /2) · (0, 1, −2y)dy =
Γ Γ2 0
Z 1
1
= y 3 dy =
0 4
AMPLIACIÓN DE CÁLCULO (Curso 2008/2009) Convocatoria de Junio 09.06.09

PROBLEMA 3 (3 puntos)

Se considera el cono Ωh de base x2 + y 2 ≤ 1, z = 0 y vértice (0, 0, h) .


1. Calcular mediante una integral el área de su superficie lateral cónica Σh .
2. Sea F (x, y, z) = y 2 z − z 4 , −x2 z, 2z − 8y 2 el campo de velocidades de un fluido. Calcular el flujo que


sale por la base de Ωh .


3. ¿Cuál debe ser la altura h del cono Ωh para que el flujo de F que entra a través de la superficie cónica
Σh sea π?

Respuesta: Se entregará esta hoja y, a lo sumo, una adicional.


1. La superficie cónica está formada por segmentos que parten de la circunferencia (cos t, sen t, 0) y
terminan en el vértice (0, 0, h) , luego sus ecuaciones paramétricas son

x = a cos t 

y = asen t t ∈ [0, 2π], a ∈ [0, 1]
z = h − ah


z
de donde eliminando el parámetro a = 1 − h también puede extraerse la ecuación de la superficie
cónica: 2
z

x2 + y 2 = 1 − .
h
Para calcular su área, usaremos la parametrización obtenida:
φ (a, t) = (a cos t, asen t, h − ha)
∂φ
= (−asen t, a cos t, 0)
∂t
∂φ
= (cos t, sen t, −h)
∂a
∂φ ∂φ p
∂t × ∂a
= a 1 + h2

por tanto
Z 2π Z 1 Z 2π  Z 1
∂φ ∂φ p  p
2 ada = π 1 + h2
área (Σh ) = ∂t × ∂a dadt = 1 + h dt

0 0 0 0

2. La base es B = {(x, y, 0) x2 + y 2 ≤ 1} orientada con normal (0, 0, −1), que es la normal saliente del
sólido Ωh por su base B. En los puntos de B el campo toma el valor F (x, y, 0) = 0, 0, −8y 2 ; entonces


el flujo pedido es
Z Z Z   Z Z Z 2π  Z 1 
F ds = 0, 0, −8y 2 ◦(0, 0, −1) dxdy = 8y 2 dxdy = 8 sen 2 tdt r3 dr = 2π.
B x2 +y 2 ≤1 x2 +y 2 ≤1 0 0

3. El campo es de clase C 1 (Ωh ) y div (F) = 2 luego por el Teorema de Gauss el flujo a través de la
superficie exterior de Ωh es
π
Z Z Z Z Z
F ds − F ds = divF dxdydz = 2vol (Ωh ) = 2 h.
B Σh Ωh 3
R
donde la integral de superficie Σh F ds está orientada hacia el interior de Ωh tal y como se nos pide.
Entonces
2π 2π
Z Z
F ds = F ds − h=2π − h.
Σh B 3 3
Por tanto, la altura pedida h debe ser tal que
2π 3
π = 2π − h ⇐⇒ h= .
3 2
AMPLIACIÓN DE CÁLCULO (Curso 2008/2009) Convocatoria de septiembre 15.09.09

PROBLEMA 1 (3 puntos)

Estudiar la convergencia de las siguientes integralesZ ∞ impropias y calcularlas explı́citamente en caso


sen t π
de ser convergentes. Se supone conocido el resultado dt = .
0 t 2
Z ∞ 1 − cos t
1. (0,5 puntos) dt,
0 t2
Z 1 dt
2. (0,5 puntos) q ,
0 t log(1/t)
ZZ
2
3. (1 punto) e−y dxdy,
{(x,y)∈R2 / 0≤x<∞, x≤y<∞}
ZZ
x
4. (1 punto) dxdy.
{(x,y)∈R2 / 0<x2 +y 2 ≤1} (x2 + y 2 )3/2

Respuesta: (Se entregará esta hoja y, a lo sumo, una adicional.)


Z
1 − cos t

1. dt. La función integrando no está definida en el origen pero dado que
0 t2
1 − cos t sen t 1
lı́m 2
= lı́m = ,
t→0 t t→0 2t 2
podemos extenderla con continuidad y establecer que la integral es impropia de primera especie.
Podemos razonar la convergencia por comparación (por ejemplo en la semirrecta [1, ∞) comparar
con la función f (t) = 1/t3/2 ) pero, puesto que nos piden calcularla, vamos a integrar por partes y
tener presente el resultado:

Integración por partes: Sean f, g : (a, ∞) → R funciones de clase 1, entonces


Z ∞ Z ∞
f g 0 = lı́m f (t)g(t) − lı́m+ f (t)g(t) − f 0g
a t→∞ t→a a

si tres de los cuatro términos son convergentes, también lo es el cuarto.

Integremos por partes:


dt 1
u = 1 − cos t =⇒ du = sen tdt, dv = 2
=⇒ v = −
t t
∞ 1 − cos t 1 − cos t ∞ Z ∞ sen t
Z
dt = − + dt.
0 t2 t 0 0 t
La integral es convergente ya que
1 − cos t sen t 1 − cos t
lı́m = lı́m = 0, lı́m = 0 (función acotada por función que tiende a cero)
t→0 t t→0 1 t→∞ t
por tanto:
Z ∞ 1 − cos t Z ∞
sen t π
dt = dt = .
0 t2 0 t 2
Z 1 dt
2. q . La integral es impropia en ambos lı́mites, vamos a transformarla mediante un
0t log(1/t)
cambio de variable y razonaremos conforme al resultado:

Cambio de variable: Sea f : (a, b) → R una función continua; sea ϕ : (α, β) → R de clase 1,
ϕ0 6= 0 y tal que ϕ((α, β)) = (a, b). Entonces se da igualdad
Z b Z β
f (t)dt = f (ϕ(x))|ϕ0 (x)|dx.
a α

Es decir, ambas integrales convergen o divergen simultáneamente.

Hagamos el cambio ϕ(x) = e−x que es una función monótona decreciente que transforma biyec-
tivamente la semirrecta (0, ∞) en el intervalo (0, 1). Se tiene:

1 ex/2 ex/2
f (t) = q =⇒ f (ϕ(x)) = f (e−x ) = √ , f (ϕ(x))|ϕ0 (x)| = √ e−x = x−1/2 e−x/2 .
t log(1/t) x x

Al hacer ahora el cambio x = 2y, dx = 2dy la integral queda:


Z 1 dt Z ∞ √ Z ∞ √ √
q =2 (2y)−1/2 e−y dy = 2 y −1/2 e−y dy = 2Γ(1/2) = 2π.
0 t log(1/t) 0 0

Para los apartados 3. y 4. utilizaremos los siguientes resultados:

Sea S ⊂ Rn y f una función definida en S. Entonces


Z Z
f es convergente ⇐⇒ |f | es convergente.
S S
Z
Si f es no negativa en S y existe una sucesión básica {Mk }k∈N tal que existe el lı́m f,
Z Z k→∞ Mk

entonces f es integrable en S y además f = lı́m f.


S k→∞ Mk

ZZ
2
3. e−y dxdy.
{(x,y)∈R2 / 0≤x<∞, x≤y<∞}

La función subintegral es no negativa, el recinto de integración (no acotado) consiste en la parte


del primer cuadrante que está por encima de la bisectriz. Para cada k ∈ N podemos tomar como Mk
el interior del triángulo de vértices (0, 0), (0, k) y (k, k). Entonces hay que calcular la integral
ZZ
2
e−y dxdy
Mk

2
Dado que no existe una primitiva elemental de e−y nos interesa integrar primero en la variable x, es
decir, proyectar el recinto sobre el eje OY :

Mk = {(x, y) ∈ R2 / 0 ≤ y ≤ k, 0 ≤ x ≤ y}

con lo que se tiene:


ZZ Z k Z y  Z k 1 2 k
1
−y 2 −y 2 2 2
e dxdy = e dx dy = ye−y dy = − e−y = (1 − e−k )
Mk 0 0 0 2 0 2
2
y, como lı́m e−k = 0, la integral es convergente y vale 1/2.
k→∞
ZZ
x
4. dxdy.
{(x,y)∈R2 / 0<x2 +y 2 ≤1} (x2 + y 2 )3/2

En este caso tomamos como sucesión básica:


1
Mk = {(x, y) ∈ R2 / ≤ x2 + y 2 ≤ 1}, k = 1, 2, . . .
k2
Tomamos la función subintegral en valor absoluto y pasamos a coordenadas polares (x = ρ cos θ, y =
ρ sen θ, J = ρ)
ZZ
|x| ZZ
|ρ cos θ| Z 1
dρ Z 2π
dxdy = ρ dρdθ = | cos θ|dθ
Mk (x2 + y 2 )3/2 [1/k,1]×[0,2π] ρ3 1/k ρ 0

Calculemos cada una de las integrales simples:


Z 1 dρ
= log ρ|11/k = log k,
1/k ρ

Z 2π Z π/2
| cos θ|dθ = 4 cos θdθ = 4 sen θ|π/2
0 = 4.
0 0

Dado que el logaritmo en infinito diverge a infinito, la integral no es absolutamente convergente y


por tanto diverge. Obsérvese que
Z 1
ZZ
x dρ Z 2π
dxdy = cos θdθ = 0
Mk (x2 + y 2 )3/2 1/k ρ 0

Esto no contradice el resultado anterior, solo nos indica que, de ser convergente la integral, su valor
tendrı́a que ser cero, cosa que obviamente no sucede.
Gabriela Sansigre.
AMPLIACIÓN DE CÁLCULO (Curso 2008/2009) Convocatoria de Septiembre 15.09.09

NOMBRE . . . . . . . . . . . . . . . . . . . . . . . . . APELLIDOS . . . . . . . . . . . . . . . . . . . . . . . . . . . . . . . . . . . . . . . . . . .
Número de matrı́cula. . . . . . . . . . . . . . . . . . . . . . . .

PROBLEMA 2 (4 puntos)
Sea a un vector no nulo de R3 , sea α ∈ R y denotamos r = (x, y, z), r = krk.
1. (0.75 ptos.) Calcular la divergencia del campo
a·r α
H(r) = 3
r− a
r r
y estudiar para qué valores de α es solenoidal en su dominio de definición.
2. (1 pto.) Para los valores de α calculados en el apartado 1 estudiar si el campo H admite un
potencial vector de la forma F(r) = f (r)a × r donde f : (0, ∞) → R es una función de clase 1. En
caso afirmativo calcularlo.
3. (1.25 ptos.) Sea

x+y 1
G(x, y, z) = (x, y, z) + (1, 1, 0)
(x2 + y2 + z 2 )3/2 (x2 + y2 + z 2 )1/2

y sea S la porción de la superficie esférica de centro el origen y radio 2 que está situada por encima
del plano x = z, orientada con la normal ”saliente”. Calcular el flujo de G a través de S.
4. (1 pto.) Sea

x+y 1
T(x, y, z) = 2 (x, y, z) + (1, 1, 0)
(x2 2 2
+y +z ) (x + y 2 + z 2 )
2

Calcular el flujo de T a través de S.

Nota: a × (b × c) = (a · c)b − (a · b)c

Respuesta: Se entregará esta hoja y, a lo sumo, una adicional.


1. El campo H está definido en Ω := R3 − {0} y además es de clase infinito en dicho conjunto.
Utilizando que div(U V) = gradU · V + U divF, que grad(U/V ) = (V gradU − U gradV )/V 2 , que
grad(r) = r/r para r 6= 0 y la regla de la cadena se tiene
· ¸
a·r 1 a·r a·r 1 1
divH(r) = div( 3 r) − αdiv( a) = grad( 3 ) · r+ 3 div(r) − α grad( ) · a + diva =
r r r r r r
3 3
grad(a · r)r − a · r grad r a·r 1r
= 6
·r+3 3 +α 2 ·a=
r r r r
3 2r
r a − a · r 3r r a·r 1r a·r
= 6
· r + 3 3 + α 2 a = (1 + α) 3
r r r r r
donde también se ha utilizado que grad(a · r) = a.
Puesto que a · r no se anula salvo en los puntos del plano que pasa por el origen y es perpendicular
a a, para que divH(r) sea nula en Ω debe ser 1 + α = 0, es decir, α = −1.
2. Considerando el caso α = −1, sea
a·r 1
H(r) = 3
r+ a
r r
Obsérvese que aunque H es solenoidal en su dominio Ω, no es posible asegurar a priori que H admita
potencial vector en Ω, pues Ω no es estrellado.
Calcularemos rot F para ver si se cumple rot F = H para alguna función f . Veamos

rot F(r) = grad(f (r)) × (a × r) + f (r)rot(a × r)

Usando la regla de la cadena obtenemos grad(f (r)) = f 0 (r) rr y como a×r = (a2 z − a3 y, a3 x − a1 z, a1 y − a2 x)
calculando se obtiene rot(a × r) = 2a. Por ello
r
rot F(r) = f 0 (r) × (a × r) + 2f (r)a
r
Utilizando la fórmula para el doble producto vectorial a × (b × c) = (a · c)b − (a · b)c obtenemos
finalmente
f 0 (r) ¡ 2 ¢ f 0 (r)
rot F(r) = r a − (a · r) r + 2f (r)a = (rf 0 (r) + 2f (r)) a − (a · r) r
r r
Por tanto rot F = H en Ω si y solo si se cumple

1 f 0 (r) 1
rf 0 (r) + 2f (r) = y − = 3 para todo r > 0
r r r
es decir, debemos estudiar si existe alguna f tal que las dos ecuaciones diferenciales ordinarias

1 f 0 (t) 1
tf 0 (t) + 2f (t) = y − = 3 para todo t > 0
t t t
tienen solución. De la segunda ecuación tenemos f 0 (t) = − t12 por lo que integrando directamente se
obtiene que sus soluciones son
1
f (t) = + C
t
donde C es una constante arbitraria. Entrando en la primera ecuación se obtiene
µ ¶
1 1 1
−t 2 + 2 +C =
t t t

que se verifica si y sólo si C = 0. En definitiva, existe una única solución que es


1
F(r) = a × r
t
Obsérvese que hemos obtenido un potencial vector de H en Ω con lo que en este caso concreto
hemos demostrado que sı́ existe potencial vector en Ω a pesar de que no se cumplen las condiciones
suficientes (campo solenoidal en un conjunto abierto estrellado).
3. Del enunciado se deduce que G corresponde al campo H del apartado 1 para α = −1 cuando
a = (1, 1, 0). Por lo tanto, el campo es solenoidal en Ω y además, como resultado del aparatado
anterior, el campo F(r) = 1r a × r es un potencial vector de G cuando a = (1, 1, 0).
El cálculo del flujo pedido se puede enfocar:
a) A través de la definición calculando la integral de superficie.
b) Aplicando el teorema de Stokes,calculando la circulación de F sobre la curva frontera Γ de S,
es decir, Z Z
φ := Gds = Fdr
S Γ

c) Aplicando el teorema de Gauss. Puesto que G es solenoidal en Ω, si Σ es cualquier superficie


que no pase por el origen (pues ahı́ no está definido el campo) que se apoye sobre Γ (y esté orientada
de forma coherente con la misma) y tal que el reciento limitado por S ∪ Σ no contenga al origen, la
aplicación del teorema de Gauss permite afirmar que
Z
φ= Gds
Σ

La opción (a) presenta la dificultad de que no es fácil obtener una parametrización simple de la
superficie. Por ejemplo, un primer intento de parametrizar la superficie podrı́a ser
x = 2 cos θsenϕ ; y = 2senθsenϕ , z = 2 cos ϕ ; (θ, ϕ) ∈ A
pero el dominio A en el que varı́an θ y ϕ no es fácil de calcular. En efecto, A será de la forma
A = [0, 2π] × [0, ϕ(θ)] donde ϕ(θ) se obtiene imponiendo x = z, es decir, cos θsenϕ(θ) = cos ϕ(θ).
Como se ve, es complicado obtener ϕ(θ) a partir de esta ecuación.
Una parametrización más sencilla, aunque no tan evidente, es considerar que ϕ sea el ángulo que
el radio vector forma con el eje y, en cuyo caso
x = 2 cos θsenϕ ; y = 2 cos ϕ, z = 2senθsenϕ ; (θ, ϕ) ∈ A (1)
Dibujando la superficie es fácil ver que ahora el dominio en que varı́an los parámetros es A =
[ π4 , 5π
4
] × [0, π]. Con esta parametrización el flujo se podrı́a calcular sin mucha dificultad utilizando la
definición.
Estudiemos el camino (c). En nuestro caso la aplicación de este resultado no es muy útil, pues no
hay superficies ”fáciles” Σ que cumplan las condiciones anteriormente mencionadas. En un primer
momento nos podrı́amos ver tentados de tomar como superficie Σ la porción del plano x = z contenida
dentro de la esfera. Sin embargo dicha superficie pasa por el origen, donde G no está definido, y por
ello el teorema de Gauss no podrı́a aplicarse.
Procedamos por el camino (b). Consideremos la curva frontera Γ de la superficie S, es decir, la
circunferencia de ecuación
x2 + y 2 + z 2 = 4
x = z
orientada de forma coherente con S, es decir, de forma que su proyección sobre el plano xy se recorre
en sentido positivo. Como el campo F es de clase 1 en Ω y la superficie S está contenida en Ω, se
puede aplicar el teorema de Stokes obteniéndose
Z Z Z
1
φ := Gds = Fdr = a × r dr
S Γ Γ r

Puesto que Γ está contenida en la esfera y en ella la distancia al origen es 2 se puede simplificar la
expresión anterior Z Z
1 1
φ= a × r dr = a × r dr
Γ r 2 Γ
Ahora tenemos dos posibles caminos. (b.1) Calcular la circulación anterior utilizando la definición o
(b.2) aplicar el teorema de Stokes y calcular dicha circulación mediante una integral de superficie.
Camino (b.2). Sea Π la porción del plano x = z contenida dentro de la esfera orientada de forma
coherente con la curva. Puesto que a × r es de clase 1 en R3 podemos √ aplicar Stokes y escribir,
teniendo en cuenta que el vector normal unitario a Π es n = (−1, 0, 1)/ 2,
Z Z Z Z
1 1 1
φ = a × r dr = rot(a × r) ds = 2a ds = a · n ds =
2 Γ 2 Π 2 Π Π
Z Z Z
(−1, 0, 1) 1 1
= a· √ ds = √ (a3 − a1 )ds = √ (a3 − a1 ) ds =
Π 2 2 Π 2 Π
1 1 √
= √ (a3 − a1 )Área(Π) = √ (a3 − a1 )π22 = −2 2π
2 2
donde se ha usado que Π es un cı́rculo de radio 2.
Camino (b.1). Parametricemos Γ, para lo cual escribimos Γ en la forma

2x2 + y 2 = 4
x = z

donde la primera
√ ecuación define la curva proyección de Γ sobre el plano xy. Parametrizamos dicha
proyección (x/ 2)2 + (y/2)2 = 1 en la forma

x = 2 cos t ; y = 2 sint , t ∈ [0, 2π]

(obsérvese que la orientación es la correcta) y por ello una posible parametrización de Γ es


√ √
x = 2 cos t ; y = 2 sint ; z = 2 cos t , t ∈ [0, 2π]

Aplicando la definición se tiene


Z Z
1 1
φ = a × r dr = (z, −z, y − x) dr =
2 Γ 2 Γ
Z
1 2π ³√ √ √ ´ ³ √ √ ´
= 2 cos t, − 2 cos t, 2 sint − 2 cos t · − 2sin t, 2 cos t, − 2sin t dt =
2 0
Z
1 2π ³ √ ´ √
= −2 2 dt = −2 2π
2 0
4. El campo T se puede escribir en la forma
a·r 1
T(r) = 4
r+ 2 a
r r
con a = (1, 1, 0). Este campo no es solenoidal (se puede calcular su divergencia para comprobarlo)
con lo que en principio la forma más inmediata de calcular el flujo pedido
Z Z µ ¶
a·r 1
α := Tds = r+ 2 a ds
S S r4 r
es calculando la integral de superficie aplicando la definición. Nótese que, como ya se vio en el
apartado 3, el cálculo de dicha integral tiene el inconveniente de que no es fácil parametrizar la
superficie. Además, si aplicásemos el teorema de Gauss tendrı́amos que calcular un flujo sobre una
superficie en principio más difı́cil que el casquete esférico (como ya se vio en el punto (c) del apartado
anterior, no se puede utilizar el plano Π) y además una integral triple, por lo que no parece el método
más aconsejable.
Como resultado de los razonamientos anteriores, intentaremos relacionar el flujo α con el flujo φ
calculado en el apartado anterior, pues la superficie es la misma y el campo, si bien es distinto, tiene
una forma muy parecida. Si no lo conseguimos entonces calcularemos α aplicando la definición.
Puesto que sobre la esfera el vector normal unitario es r = 2 tenemos que
Z µ ¶ Z µ ¶
1 a·r 1 1 a·r 1 1 √
α := 3
r + a ds = 3
r + a ds = φ = − 2π
S r r r 2 S r r 2
Si no nos damos cuenta de esta circunstancia y no somos capaces de relacionar α con φ aplicamos
la definición usando la parametrización (1). Como el vector unitario es n = r/R = r/2 se tiene
Z µ ¶ Z µ ¶ Z
a·r 1 r 1 a·r 1 1
α = r + 2 a · ds = + 2 a · r ds = 3 (a · r + a · r) ds =
S r4 r 2 2 S r2 r 2 S
Z µZ Z ¶
1 a=(1,1,0) 1
= a · rds = xds + yds
4 S 4 S S
R
Como S es simétrica respecto del plano y = 0 y y es una función impar en y s sigue que S
yds = 0
con lo que, usando (1),
Z Z 5π Z π Z 5π Z π
1 1 4 4
α = xds = (2 cos θsenϕ) (4senϕ) dθdϕ = 2 cos θdθ sen2 ϕdϕ =
4 S 4 π
0 π
0
√ 4 4

= − 2π
AMPLIACIÓN DE CÁLCULO (Curso 2008/2009) Convocatoria de septiembre 15.09.09

NOMBRE . . . . . . . . . . . . . . . . . . . . . . . . . APELLIDOS . . . . . . . . . . . . . . . . . . . . . . . . . . . . . . . . . . . . . . . . . . .
Número de matrı́cula. . . . . . . . . . . . . . . . . . . . . . . .

Problema 3 (3 puntos): Se considera el sólido Ω interior al cilindro (x − a)2 + y 2 = a2 (a > 0)


limitado por la superficie cónica z 2 = x2 + y 2 , z ≥ 0, y el plano XOY.

1. Calcular el volumen de Ω. (1 punto)

2. Hallar el área de su superficie lateral. (Sugerencia: relacionarla con una integral curvilı́nea).
(1 punto)
 √ 
3. Sea F (x, y, z) = 0, 0, − x2 + y 2 el campo de velocidades de un fluido. Calcular el flujo que
entra por la base superior de Ω. (1 punto)

Respuesta: (Se entregará esta hoja y, a lo sumo, una adicional)

1. El sólido es q
Ω = {(x, y, z) : (x − a)2 + y 2 ≤ a2 0≤z≤ x2 + y 2 }
luego es el espacio proyectable sobre el disco plano D : (x − a)2 + y 2 ≤ a2 . Ası́ pues:
 √ 
Z Z Z Z Z Z Z Z x2 +y 2 q
vol (Ω) = dxdydz =  dz  dxdy = x2 + y 2 dxdy.
Ω D 0 D

Para calcular esta integral doble, podemos cambiar a coordenadas polares en D, donde se
verifica que

(x − a)2 + y 2 ≤ a2 ⇔
x2 + y 2 ≤ 2ax ⇔
0 ≤ ρ ≤ 2a cos θ

ası́ que, para cada ángulo θ con cos θ ≥ 0 (es decir, para θ ∈ [−π/2, π/2]) se tiene que ρ ∈
[0, 2a cos θ]. Por tanto, la integral queda
Z π/2 Z 2a cos θ
1 Z π/2 16a3 Z π/2
vol (Ω) = (2a cos θ)3 dθ =
ρ2 dρdθ = cos3 θdθ
−π/2 0 3 −π/2 3 0
16a3 Z π/2   32a3
= cos θ 1 − sin2 θ dθ = .
3 0 9

2. La superficie lateral√Σ de Ω es la superficie cilı́ndrica comprendida entre las gráficas de las


funciones h (x, y) = x2 + y 2 y g (x, y) = 0, definidas sobre la curva borde C del dominio D.
Por ello, puede relacionarse el área de Σ con una integral curvilı́nea sobre C :
Z Z q
area (Σ) = (h − g) dl = x2 + y 2 dl.
C C

Por otro lado, la curva C es la circunferencia plana (x − a)2 +y 2 = a2 luego viene parametrizada
como
(x (t) , y (t)) = (a + a cos t, a sin t) t ∈ [0, 2π]
y su vector derivada es (x0 (t) , y 0 (t)) = (−a sin t, a cos t) que tiene módulo a. Por todo ello,
Z q Z 2π q
area (Σ) = x2 + y 2 dl = a x2 (t) + y 2 (t)dt =
C 0
Z 2π q
2 2
Z 2π √
= a (a + a cos t) + a2 sin tdt = a 2a2 + 2a2 cos tdt =
0 0
√ Z
2
2π √ 2
Z 2π
t
 
2
Z π
t
 
dt = 8a2 .

= a 2 1 + cos tdt = 2a cos
dt = 4a cos
0 0

2 0 2

3. El fluido F es adivergente y de clase C 1 en el sólido Ω, luego


RRR
Ω div (F) dxdydz = 0; y
podemos aplicar el Teorema de Gauss, con lo que:
Z Z Z
F ds = F ds + F ds
Base superior Base inferior Σ

donde el primer miembro es el flujo pedido, y las superficies del segundo miembro están ori-
entadas hacia el exterior de Ω. Obsérvese que Σ es una superficie cilı́ndrica recta, y F es un
campo de dirección vertical, luego el flujo de F a través de Σ es nulo (en otras palabras, F
pertenece al plano tangente de Σ en cada
R
punto, o bien F es ortogonal al vector normal a Σ en
cada punto). Por todas esas razones, Σ Fds = 0.
Basta, pues, calcular el flujo de F en la base inferior de Ω, que es el dominio plano D, con
vector normal (0, 0, −1) se considera el flujo que sale de Ω. Dicha integral de superficie queda
Z Z Z  q  Z Z q
F ds = 0, 0, − x2 + y 2 ◦ (0, 0, −1) dxdy = x2 + y 2 dxdy
D D D

que resulta ser la integral doble ya calculada en el apartado 1. Finalmente,


Z
32a3
F ds = vol (Ω) = .
Base superior 9
AMPLIACIÓN DE CÁLCULO (Curso 2009/2010) Convocatoria de Febrero 26.01.10

NOMBRE . . . . . . . . . . . . . . . . . . . . . . . . . APELLIDOS . . . . . . . . . . . . . . . . . . . . . . . . . . . . . . . . . . . . . . . . . . .
Número de matrı́cula. . . . . . . . . . . . . . . . . . . . . . . .

PROBLEMA 1 (3 puntos)
1. (1.25 ptos.) Estudiar la convergencia de, y en su caso calcular, la integral impropia
∫ ∞
1
dx
0 (1 + a2 x2 )(1 + x2 )
donde a ≥ 0.
2. (1.75 ptos.) Sea a ≥ 0. Estudiar la convergencia de
∫ ∞
log(1 + a2 x2 )
dx
0 x2 (1 + x2 )
y calcularla con todo rigor cuando sea convergente.
Respuesta: Se entregará esta hoja y, a lo sumo, una adicional.
1. Sea I(a) la integral del enunciado. Para todo a ≥ 0 la función
1
f (x, a) :=
(1 + a2 x2 )(1 + x2 )

es continua como función de x en los compactos de la forma [0, T ], T > 0 y por tanto está acotada
en los mismos. Por ello, la única causa por la que la integral es impropia es porque el intervalo
de integración es no acotado. Puesto que para todo a ≥ 0 la función f (x, a) es positiva para
x ∈ [0, ∞), podemos aplicar los criterios para integrales impropias de funciones que no cambian de
signo. Apliquemos el criterio del cociente. Puesto que
1
(1+a2 x2 )(1+x2 )
lim 1
x→∞
x2
∫∞
vale
∫ ∞ cero si a > 0 y 1 si a = 0, y puesto ∫que 1 1/x2 dx es convergente (atención, obsérvese que

0
1/x2 dx no es convergente) tenemos que 1 f (x, a)dx, y por ello I(a), son convergentes para todo
a ≥ 0.
Puesto que la función subintegral es racional, para calcular la integral impropia descompondremos
dicha función en fracciones simples. Si a ̸= 0 y a ̸= 1 las raı́ces del denominador son ±i y ± a1 i
imaginarias y simples. Por ello en ese caso (a ∈ / {0, 1}) podemos escribir
1 A + Bx C + Dx
= +
(1 + a2 x2 )(1 + x2 ) 1 + x2 1 + a2 x2

Poniendo común denominador y dando valores a x (por ejemplo x = i y x = a1 i) se obtiene

1 a2
A= ; B = 0 ; C = ; D=0
1 − a2 a2 − 1
por lo que
∫ ∞ [ ] ∫ ∞ ∫ ∞
1 1 a2 1 1 dx a2 dx
I(a) = + dx = +
0 1 − a2 1 + x2 a2 − 1 1 + a2 x2 1 − a2 0 1 + x2 a2 − 1 0 1 + a2 x2
donde podemos separar en suma de integrales porque las dos integrales del miembro de la derecha
son convergentes. Ası́

1 ∞ a2 1
arctan(ax) |∞
al ser a>0
I(a) = arctan x |0 + 2 =
1−a 2 a −1a 0

1 π a π a−1 π π 1
= + 2 = 2 = (1)
1−a 2 a −12
2 a −12 2a+1
Nótese que para obtener la expresión anterior hemos usado que a > 0.
En principio esta expresión se ha deducido para el caso a ̸= 1 y a ̸= 0. Para calcular I(0) e I(1)
podemos sustituir directamente en la definición de I
∫ ∞
1 π
I(0) = 2
dx =
(1 + x ) 2
∫0 ∞
1
I(1) = dx
0 (1 + x2 )2

de donde vemos que el cálculo de I(1) es en principio laborioso. Por ello, veamos otro enfoque: si
I(a) fuese continua en [0, ∞), la expresión (1) también serı́a válida para a = 1 y a = 0, pues el valor
de la función en a = 1 concidirı́a con el valor del lı́mite cuando a → 1 y lo mismo sucederı́a en el caso
a = 0. Demostremos que I(a) es continua en [0, ∞), para lo cual, puesto que f (x, a) es continua en
[0, ∞) × [0, d] para todo d > 0, sólo tenemos que demostrar que la integral que define a I converge
uniformemente en [0, d] para todo d > 0. Para ello usaremos el teorema de Weierstrass: Sea d > 0.
Por un lado, para todo a ∈ [0, d] y toda x > 0,

1 1

(1 + a2 x2 )(1 + x2 ) ≤ 1 + x2
∫ ∞ dx
y por otro lado 0 1+x 2 converge, por lo que se ha probado la convergencia uniforme que se buscaba.

En definitiva, I(a) es continua en [0, ∞) y por ello

π 1
I(a) =
2a+1

para todo a ∈ [0, ∞). Nota: en vez de demostrar que hay convergencia uniforme en [0, d] para todo
d > 0. también podrı́amos haber demostrado directamente que hay convergencia uniforme para
a ∈ [0, ∞).
2. Sea J(a) la integral del enunciado y sea g(x, a) la función subintegral. Puesto que
a2 2x
log(1 + a2 x2 ) log(1 + a2 x2 ) L’Hopital 1+a2 x2
lim = lim+ = lim+ = a2 ,
x→0+ x2 (1 + x2 ) x→0 x2 x→0 2x

g(x, a) está acotada en el entorno de cero. Como para cada valor de a la función g es continua
como función de x en los compactos de (0, ∞), la integral J sólo es impropia porque el intervalo de
integración es no acotado. La función subintegral g es no negativa para todo x > 0 y toda a ≥ 0.
Aplicando el criterio del cociente, como para todo a ≥ 0
log(1+a2 x2 )
x2 (1+x2 )
lim 1 = 0,
x→∞
x2
∫∞
y 1 1/x2 dx converge, se sigue que J(a) es convergente para todo a ≥ 0. Si se cumpliesen las condi-
ciones del teorema de derivación de integrales impropias dependientes de un parámetro, tendrı́amos
que
∫ ∞
′ d log(1 + a2 x2 )
J (a) = dx
da x2 (1 + x2 )
∫ ∞0
1 ∂
= 2 2
log(1 + a2 x2 )dx =
x (1 + x ) ∂a
∫0 ∞
2a a
= dx = 2aI(a) = π (2)
0 (1 + a2 x2 )(1 + x2 ) a+1

Vamos a demostrar que dichas condiciones se cumplen en el intervalo a ∈ [0, d] para todo d > 0,
con lo que entonces J será derivable con continuidad en [0, ∞) y su derivada estará dada por (2).
Sea d tal que 0 < d. La función g(x, a) es continua en (0, ∞) × [0, d] y además ∂a
∂g
(x, a) también es
continua en dicho recinto. Sólo falta por demostrar que
∫ ∞
2a
dx
0 (1 + a x2 )(1 + x2 )
2

converge uniformemente en [0, d], que se demuestra de forma muy parecida a la empleada en el
apartado 1. Por Weierstrass, para todo a ∈ [0, d] y toda x > 0,

2a 2d

(1 + a2 x2 )(1 + x2 ) ≤ 1 + x2
∫ ∞ 2d
y por otro lado 0 1+x 2 dx converge, por lo que se ha probado la convergencia uniforme que se

buscaba.
Como conclusión, J es de clase 1 en [0, ∞) y J ′ (a) = 2aI(a) = π a+1 a
. Ahora calculando una
primitiva de esta función, se tiene que para todo a ≥ 0
∫ ∫ ∫ ( )
a a+1−1 1
J(a) = π da = π da = π 1− da = πa − π log |1 + a| + C =
a+1 a+1 a+1
= πa − π log(1 + a) + C

Para determinar C usaremos que J(0) = 0, de donde resulta que C = 0. En definitiva

J(a) = πa − π log(1 + a) , a ≥ 0

NOTA: aunque no se pide el valor de J(a) para a < 0, éste es fácil de obtener pues J es una
función par. Por ello
J(a) = π |a| − π log(1 + |a|) , a ∈ R
AMPLIACIÓN DE CÁLCULO (Curso 2009/2010) Convocatoria de febrero 26.01.10

PROBLEMA 2 (3 puntos)
Sea el campo vectorial
1
F(x, y) = (y, −x), (x, y) ∈ R2 \ {(0, 0)}.
x2 + y2
I
1. Calcular F, para cada una de las siguientes curvas (supuestas simples y recorridas en sentido
Γ
positivo):
x2 y 2
a) (1 punto) Γ ≡ + = 1,
4 9
x2 (y + 3)2
b) (0,5 puntos) Γ ≡ + = 1.
9 4
2. Sea D = {(x, y) ∈ R2 / (x − 1)2 + (y − 1)2 ≤ 3/2}.
a) (1 punto) Determinar un potencial escalar f de F en D tal que f (0, 1) = 0.
(¡Atención! f tiene que ser continuo en D.)
Z √
b) (0,5 puntos) Calcular F, para cualquier curva γ en D de extremo inicial (1, 0) y final ( 3, 1).
γ

Respuesta:

1. El campo F = (P, Q) es de clase 1 en su dominio de definición, estudiemos si es conservativo en


alguna parte del plano; para ello calculemos en primer lugar ∂Q/∂x y ∂P/∂y:
à !
∂P ∂ y x2 − y 2
= = ,
∂y ∂y x + y 2
2 (x2 + y 2 )2
à !
∂Q ∂ −x x2 − y 2
= = ,
∂x ∂x x2 + y 2 (x2 + y 2 )2

se cumple la igualdad de ambas derivadas parciales por lo que F es conservativo en dominios


simplemente conexos contenidos en su dominio de definición, es decir, que excluyan el origen.
a) La elipse Γ es frontera de un dominio que contiene el origen por lo cual F no es conservativo. El
campo F parece fácilmente integrable en una circunferencia centrada en el origen, sin embargo
una integración directa en la elipse nos conducirá probablemente a una integral pesada de cal-
cular por métodos elementales. Una alternativa entonces consiste en aplicar el teorema de Green
generalizado a dominios múltiplemente conexos del plano. Para ello, Sea Cr una circunferencia de
centro el origen y radio r ∈ (0, 2) (observemos que está contenida en el interior geómetrico de la
elipse) y sea Ω la región encerrada entre Γ y Cr , supóngase la circunferencia recorrida en sentido
positivo, entonces
ZZ Ã ! Z Z
∂Q ∂P
− dxdy = F− F = 0,
Ω ∂x ∂y Γ Cr

por tanto
Z Z
1 Z 2
F= F= 2
ydx − xdy = − 2 (área encerrada por Cr ) = −2π.
Γ Cr r Cr r
b) La elipse no encierra el origen, en consecuencia el campo F es conservativo en una región simple-
mente conexa del plano que contiene a la curva cerrada Γ, por lo que la integral vale 0.

2.a) El origen no √
está incluido en el disco D ya que la distancia del centro de D al origen es 2, mayor
que el radio 6/2 por ello F es conservativo en D y admite potencial escalar.

Figura 1: El disco D corta los ejes

Integramos
∂f y
= P (x, y) = 2 ⇒ f (x, y) = arc tg(x/y) + C(y), y 6= 0
∂x x + y2
∂f −x
= 2 2
+ C 0 (y) = Q(x, y) ⇒ C 0 (y) = 0, f (0, 1) = 0 ⇒ C = 0.
∂y x +y
El campo hallado es válido para y > 0, lo extendemos con continuidad para los (x, y) ∈ D con
y < 0, como para los y próximos a cero es x > 0,
π π
lı́m+ f (x, y) = , lı́m− f (x, y) = − .
y→0 2 y→0 2
Para cualquier (x, y) ∈ D definimos


 arc tg(x/y)
y>0
π
f (x, y) =  2
y=0

arc tg(x/y) + π y < 0
Método alternativo:
Dado que nos proporcionan el dato inicial en (0, 1), podemos integrar el campo F a lo largo de la
poligonal que une (0, 1), (0, y), (x, y), con y 6= 0:
Z (x,y) Z y Z x Z x
y
f (x, y) = F= Q(0, t)dt + dt = arc tg(x/y).
P (t, y)dt =
(0,1) 1 0 0 + y2 t2
Los comentarios sobre la extensión continua se completan como en el método anterior.
2.b) Por ser F un campo gradiente en D, basta con evaluar el potencial hallado en el apartado anterior
en los extremos de la curva:
Z √ √ π π π π
F = f ( 3, 1) − f (1, 0) = arc tg 3 − = − = − .
γ 2 3 2 6
Método alternativo:
También puede calcularse el valor de la integral escogiendo
√ una curva √concreta,√por ejemplo la
lı́nea poligonal formada por los segmentos S1 = [(1, 0), ( 3, 0)] y S2 = [( 3, 0), ( 3, 1)]. En S1 la
integral es nula porque la segunda componente es nula (dy = 0) y P (t, 0) = 0. Entonces:
Z Z 1 Z 1 √
√ − 3 √ 1 √ π
F= Q( 3, t)dt = 2
dt = − arc tg(t/ 3)| 0 = − arc tg(1/ 3) = − .
γ 0 0 t +3 6

Gabriela Sansigre.
AMPLIACIÓN DE CÁLCULO (Curso 2009/2010) Convocatoria de febrero 26.01.10

NOMBRE . . . . . . . . . . . . . . . . . . . . . . . . . APELLIDOS . . . . . . . . . . . . . . . . . . . . . . . . . . . . . . . . . . . . . . . . . . .
Número de matrı́cula. . . . . . . . . . . . . . . . . . . . . . . .

PROBLEMA 3 (4 puntos)
NOTA: Este problema consta de dos apartados independientes.
1) Sea F el campo vectorial definido por:
³ ´
(1 − y)z , 2xz , x(1 − y)
F := F(x, y, z) =
(x2 + (y − 1)2 + z 2 )3/2
1.a) Demuéstrese que el flujo de F a través de cualquier superficie regular y cerrada que no
pase por el punto (0, 1, 0) es nulo. (1,5 puntos)

1.b)√Calcúlese el flujo de F a través de la superficie, Σ, generada por revolución de la curva


z = 2y , con 0 ≤ y ≤ 1, alrededor del eje OY y orientada de forma que la segunda componente
de su vector normal sea negativa. (1 punto)

2) Sea S la superficie generada por revolución de la curva de ecuación cartesiana y = z alrededor
del eje OZ. Se considera el sólido homogéneo, Ω ⊂ R3 , limitado por S y por el plano z = 2x + 2cy + 2.
Determinar todos los valores posibles de c ∈ R para los que la distancia del centro de gravedad de Ω
al eje OZ es 2. (1,5 puntos)

Respuesta: Se entregará esta hoja y, a lo sumo, una adicional.


1) En primer lugar observamos que el campo vectorial F está definido en R3\ {(0, 1, 0)}. Además,
dado que se trata de calcular un flujo, es conveniente calcular su divergencia para saber cómo se puede
aplicar el teorema de Gauss. Con la notación F(x, y, z) := (F1 (x, y, z), F2 (x; y; z), F3 (x, y, z)), se tiene:
∂F1 ∂F2 ∂F3 1
∇·F = + + =³ ´ 5 (3 x (y − 1) z − 6 x (y − 1) z + 3 x (y − 1) z) = 0 .
∂x ∂y ∂z 2 2
x2 + (y − 1) + z2

1.a) En estas condiciones, dado que el campo está definido, es de clase C 1 y es solenoidal en el
conjunto R3 \ {(0, 0, 1)}, podemos aplicar el teorema de Gauss que nos permite afirmar que el flujo
de F a través de una superficie regular y cerrada, Σ, que no pase por el punto (0, 1, 0) puede tomar
los siguientes valores:
ZZ ZZZ ZZ
F · dσ = ∇ · F · dσ = 0 = F · dσ, si la superficie Σ encierra un volumen,
Σ+ V (Σ) Σ−
V (Σ), que no contiene el punto (0, 1, 0). Aquı́, Σ+ y Σ− representan las caras exterior e interior
de Σ, respectivamente.

Si el volumen acotado limitado por la superficie, que no pasa por (0, 1, 0), si contiene este punto,
entonces el teorema de Gauss nos permite afirmar que el flujo pedido es el mismo para todas las
superficies regulares que cumplan esta condición y estén orientadas de la misma forma (según
el vector normal saliente o entrante). Para calcular su valor, elegimos una de ellas. Por ejemplo,
la cara, E + , de la esfera de centro el punto (0, 1, 0) y radio 1 orientada según la normal saliente.
Se tiene: ZZ ZZ µ ¶
F · dσ = (1 − y)z , 2xz , x(1 − y) ,
E+ E+

1
dónde el campo de la segunda integral es el valor de F sobre la superficie E + . Nótese que el
denominador (x2 + (y − 1) ∗ 2 + z 2 )( 3/2) es igual a 1 sobre esta superficie.
Ahora bien, este último campo está definido y es solenoidal en todo R3 con lo que se puede
aplicar de nuevo el teorema de Gauss para concluir que el flujo a través de E + (regular y
cerrada) es también nulo en este caso. También lo será, por tanto, el flujo a través de la cara
interior E − .

En resumen, el flujo pedido es siempre nulo.

1.b) Existen diversas alternativas para calcular el flujo en este apartado. Exponemos a conti-
nuación tres de ellas:
Primera alternativa: Usar el teorema de Gauss. La superficie Σ que aquı́ se considera no es
cerrada y tiene por borde la circunferencia x2 + z 2 = 2, y = 1. Sea ahora S una superficie cualquiera
regular y con el mismo borde que Σ que no pase por el punto (0, 1, 0). La unión de las dos, Σ ∪ S,
es una superficie cerrada para la que el flujo de F es nulo (según se ha demostrado en el apartado
anterior). Ası́ pues:
ZZ ZZ ZZ ZZ ZZ
F · dσ = F · dσ + F · dσ = 0 =⇒ F · dσ = F · dσ
(Σ∪S)+ Σ+ S+ Σ+ S−

dónde (Σ ∪ S)+ representa la cara exterior de Σ ∪ S, Σ+ la cara de Σ con la orientación dada en el


enunciado y S − la cara de S con orientación contraria a la que define sobre S la normal exterior a
Σ ∪ S.
Elegimos ahora S − como el casquete de la esfera de ecuación cartesiana x2 + (y − 1)2 + z 2 = 2
situado en y ≥ 1. Un vector unitario en la dirección de la normal considerada es el gradiente de la
función g(x, y, z) := −x2 − (y − 1)2 − z 2 + 2; es decir:

∇g −1
=q (x, y − 1, z) .
k∇gk x2 + (y − 1)2 + z 2

Pero el campo F es tangente en cada punto a S − , ya que:


∇g
F· = 0.
k∇gk

Por tanto, el flujo de F a través de S − es también nulo y coincide con el que se pretende calcular a
través de la superficie de revolución considerada.
Segunda alternativa: Usar la simetrı́a de revolución. La superficie Σ se genera por revolu-
ción de una curva alrededor del OY y, por tanto, es simétrica con respecto a ese eje. Esto significa,
en particular, que también es simétrica con respecto a cualquier plano que contenga dicho eje, lo que
ocurre con el plano Y Z.
Una ecuación cartesiana de Σ es x2 + y 2 = 2y. Tal ecuación se puede obtener si más que razonar
que cada punto de la curva que gira alrededor del eje OY describe una circunferencia de radio

2y situada en el plano paralelo al XZ y que pasa por el punto (0, y, 0). Por tanto, si se define
h := h(x, y, z) = x2 + z 2 − 2y, un vector unitario en la dirección de la normal a Σ definida en el
enunciado es: Ã !
∇h x 1 z
= {√ ,− √ ,√ },
k∇hk 1+x +z 2 2 1+x +z2 2 1 + x2 + z 2
con lo cual:
∇h −2 x y z
F· =√ ³ ´3
k∇hk
1 + x2 + z 2 x2 + (−1 + y)2 + z 2 2

2
El flujo es entonces: ZZ ZZ
∇h
F · dσ = F· dσ .
Σ+ Σ k∇hk
Pero en esta última integral de superficie, el campo escalar que se integra es impar en la variable x
como muestra su expresión ya obtenida, mientas que la superficie es simétrica con respecto al plano
Y Z. Por tanto, la integral es nula.
Tercera alternativa: Parametrizar Σ y aplicar la definición. Una parametrización de la
superficie con la orientación dada es: S(t, θ) = (x(t, θ), y(t, θ), z(t, θ)) dónde:
√ √
x(t, θ) = 2t cos θ , y(t, θ) = t , z(t, θ) = 2t cos θ , t ∈ [0, 1] , θ ∈ [0, 2π] ,

cuyo vector normal es:

∂S(t, θ) ∂S(t, θ) √ √
n(t, θ) := × = { 2t cos(u), −1, 2t sin(u)} .
∂t ∂u
Por tanto:
−4 t2 cos(θ) sin(θ)
F(S(t, θ)) · n(t, θ) = 3 .
(1 + t2 ) 2
Finalmente:
   
ZZ Z 2π Z 1 2 µZ 2π ¶ Z 1
4 t cos(θ) sin(θ) 4 t2
F · dσ = − dθ  3 dt = cos(θ) sin(θ)dθ  3 dt = 0 .
Σ+ 0 0 (1 + t2 ) 2 0 0 (1 + t2 ) 2

2) Sea V (Ω) el volumen de Ω y (xg (Ω), yg (Ω), zg (Ω)) su centro de gravedad, de forma que:
K ZZZ K ZZZ K ZZZ
xg (Ω) = xdxdydz , yg (Ω) = ydxdydz , zg (Ω) = zdxdydz ,
V (Ω) V (Ω) V (Ω)

dónde la densidad definida sobre Ω se considera constante e igual a K > 0, puesto que el sólido es
homogéneo.
La distancia desde el centro de gravedad al eje OZ es:
q
dOZ (x, y, z) = xg (Ω)2 + yg (Ω)2 ,

que debe ser igual a 2. Por tanto, para determinar el valor del parámetro c que se pide bastará obtener
los valores de xg (Ω) e yg (Ω); es decir, hay que calcular las tres integrales triples siguientes:
ZZZ ZZZ ZZZ
V (Ω) = K dxdydz , V (Ω)xg (Ω) = K x dxdydz , V (Ω)yg (Ω) = K y dxdydz .
Ω Ω Ω

Para ello, unas ecuaciones que describen Ω son:


n o
Ω = (x, y, z) ∈ R3 : (x − 1)2 + (y − c)2 ≤ 3 + c2 , (x2 + y 2 ) ≤ z ≤ 2x + 2cy + 2 ,

dónde el disco (x − 1)2 + (y − c)2 ≤ 3 + c2 es la proyección de Ω sobre el plano XY , que se obtiene


eliminando z entre las ecuaciones de las dos superficies dadas y completando cuadrados. Para cada
punto de este disco, la coordenada z varı́a entre la superficie de revolución, que tiene por ecuación
cartesiana z = x2 + y 2 , y el plano z = 2x + 2cy + 2.
La descripción de Ω que se acaba de dar aconseja utilizar coordenadas cilı́ndricas desplazadas:

x = 1 + ρ cos θ , y = c + ρ sen θ , z=z

3
con jacobiano dado por J(ρ, θ) = ρ. En estas coordenadas, el nuevo dominio Ω∗ viene dado por:

n √
Ω∗ = (ρ, θ, z) ∈ R3 : 0 ≤ θ ≤ 2π , 0 ≤ ρ ≤ 3 + c2 ,
o
(1 + c2 + ρ2 + 2 ρ (cos θ + c sen θ)) ≤ z ≤ 4 + 2c2 + 2ρ(cos θ + c sen θ)

Entonces, usando estas coordenadas y efectuando primero la integral en z, las integrales a calcular
son:

ZZZ Z 2π (Z √ 2 ) 2
3+c h i K (3 + c2 ) π
2 2
V (Ω) = K ρdρdθdz = K dθ ρ 3 + c − ρ dρ =
Ω∗ 0 0 2
ZZZ Z 2π (Z √ 2 )
3+c h i
V (Ω)xg (Ω) = K (1 + ρ cos θ)ρ dρdθdz = K dθ (1 + ρ cos θ)ρ 3 + c2 − ρ2 dρ
Ω∗ 0 0
Z 2π (Z √ 2 ) Z 2π (Z √ 2 )
3+c h i 3+c h i
2 2 2 2 2
=K dθ ρ 3 + c − ρ dρ + K dθ ρ 3 + c − ρ cos θdρ
0 0 0 0
Z 2π (Z √ 2 ) 2
3+c h i K (3 + c2 ) π
2 2
=K dθ ρ 3 + c − ρ dρ = (como la anterior) .
0 0 2
ZZZ Z 2π (Z √ 2 )
3+c h i
V (Ω)yg (Ω) = K (c + ρ sen θ)ρ dρdθdz = K dθ (c + ρ sen θ)ρ 3 + c2 − ρ2 dρ
Ω∗ 0 0
Z 2π (Z √ 2 ) Z 2π (Z √ 2 )
3+c h i 3+c h i
2 2 2 2 2
=K dθ cρ 3 + c − ρ dρ + K dθ ρ 3 + c − ρ sen θdρ
0 0 0 0
Z 2π (Z √ 2 ) 2
3+c h i cK (3 + c2 ) π
2 2
=K dθ cρ 3 + c − ρ dρ = (como la anterior) .
0 0 2

Resulta entonces xg (Ω) = 1 e yg (Ω) = c. Por lo tanto:


q √
dOZ (x, y, z) = xg (Ω)2 + yg (Ω)2 = 2 =⇒ 1 + c2 = 4 =⇒ c = ± 3 ,

que son los posibles valores que se pedı́a calcular.

4
AMPLIACIÓN DE CÁLCULO (Curso 2009/2010) Convocatoria de junio 22.06.10

NOMBRE . . . . . . . . . . . . . . . . . . . . . . . . . APELLIDOS . . . . . . . . . . . . . . . . . . . . . . . . . . . . . . . . . . . . . . . . . . .
Número de matrı́cula. . . . . . . . . . . . . . . . . . . . . . . .

PROBLEMA 1 (4 puntos)
Dadas las constantes b > a > 0, se pide:
(1) Estudiar la convergencia de la integral impropia
Z ∞ 
−a2 x2 −b2 x2 dx
I := e −e .
0 x2
Nótese que el denominador del integrando se anula en x = 0. (1 punto)
(2) Estudiar la convergencia de la integral doble impropia
ZZ
2 2
J := ye−x y dxdy ,
D

siendo D := [0, ∞) × [a, b]. (0,75 puntos)


(3) Estudiar la convergencia uniforme de la integral impropia
Z ∞
2 2
F (y) := ye−x y dx
0

cuando y ∈ [a, b]. Calcular F (y). (0,75 puntos)


(4) Razonar si se cumple o no la igualdad:
Z ∞ Z b Z b Z ∞
−x2 y 2 2 y2
dx ye dy = dy ye−x dx .
0 a a 0

(0,25 puntos)
(5) Calcular el valor de J. (0,75 puntos)
(6) Calcular el valor de I. (0,5 puntos)
Nota. Se pueden usar sin demostración todos los resultados teóricos estudiados en el curso,
ası́ como la conocida fórmula: Z ∞ √
−x2 π
e dx = .
0 2
(La preacta se publicará en intranet el 9 de julio a las 18 h.; la revisión personal de este problema –previamente
solicitada– será el 13 de julio a las 11 h.)

Respuesta: Se entregará esta hoja y, a lo sumo, una adicional.


(1) Descomponemos la integral en suma de dos:
Z 1  Z ∞ 
−a2 x2 −b2 x2 dx −a2 x2 −b2 x2 dx
I= e −e + e −e =: I1 + I2 .
0 x2 1 x2
Aunque el denominador del integrando se anula en x = 0, este punto no es singular porque la función
tiene en él lı́mite finito:
2 2
e−a x − e−b x
2 2 2 2 2 2
−2a2 xe−a x + 2b2 xe−b x  
2 −b2 x2 2 −a2 x2
lı́m+ 2
= lı́m+ = lı́m+ b e −a e = b2 − a2 .
x→0 x x→0 2x x→0

1
Por tanto, la integral en [0, 1] existe como integral de Riemann.
En cuanto a la integral en [1, ∞), se tiene para todo x ∈ [1, ∞):
2 x2 2 x2
e−a − e−b 1
0< <
x2 x2
y es bien sabido que la integral Z ∞
dx
1 x2
es convergente. Por tanto, la integral I2 converge, y la integral I es convergente.
(2) Sea Dn := [0, n] × [a, b] ⊂ R2 (Figura 1).
y
D
b
Dn
a

0 n x

Figura 1: Dominios D = [0, ∞) × [a, b] y Dn = [0, n] × [a, b]

Consideramos la integral:
ZZ
2 y2
Jn : = ye−x dxdy
Dn
Z n Z b
2 2
= dx ye−x y dy
0 a
Z n y=b
1 −x2 y2
= − 2e dx
0 2x y=a
Z
1 n  −a2 x2 2 2
 dx
= e − e−b x .
2 0 x2
Se tiene entonces: Z ∞  2 2  dx
1 2 2 1
lı́m Jn = e−a x − e−b x = I.
n→∞ 2 0 x2 2
Ya hemos establecido en (1) que I existe (y es finito) luego, teniendo en cuenta que el integrando de
J no cambia de signo, se obtiene que J es convergente y se verifica
1
J = lı́m Jn = I.
n→∞ 2
(3) Para y ∈ [a, b], se tiene:
2 y2 2 x2
0 < ye−x ≤ be−a .
La integral Z ∞
2 x2
e−a dx
0
es convergente, como es bien sabido. Por el criterio de Weierstrass, la integral
Z ∞
2 2
F (y) = ye−x y dx
0

2
converge uniformemente para y ∈ [a, b].
Calculemos el valor de F (y). Haciendo el cambio de variable x 7→ u, siendo u = xy, se tiene:
Z ∞ √
−u2 π
F (y) = e du = .
0 2
2 2
(4) Como la función e−x y es continua en [0, ∞) × [a, b], y la integral F (y) es uniformemente
convergente para y ∈ [a, b], se verifica que F (y) es integrable en [a, b] y, además,
Z b Z b Z ∞ Z ∞ Z b
−x2 y 2 2 y2
F (y) dy = dy ye dx = dx ye−x dy .
a a 0 0 a

(5) De las igualdades obtenidas en (2), resulta:


Z ∞ Z b Z n Z b
−x2 y 2 2 y2
dx ye dy = lı́m dx ye−x dy = lı́m Jn = J .
0 a n→∞ 0 a n→∞

Por (4) y (3),


Z b Z ∞ √ Z b √
−x2 y 2 π π
J= dy ye dx = dy = (b − a) .
a 0 2 a 2
(6) Se tiene √
I = 2J = (b − a) π .

3
AMPLIACIÓN DE CÁLCULO (Curso 2009/2010) Convocatoria de junio 22.06.10

PROBLEMA 2 (3 puntos)

Sea F un campo vectorial continuo en un abierto conexo D de R2 .

1.1 (0,5 puntos) Defı́nase el concepto de campo conservativo.

1.2 (0,5 puntos) Enúnciese una condición necesaria y suficiente para que F sea conservativo en D.
1
Sea F(x, y) = (x, y) y D = R2 \ {(0, 0)}.
x2 + y 2
2.1 (1 punto) Estúdiese si F es conservativo en D.
Z
2.2 Calcúlese F en los siguientes casos:
Γ

(a) (0,5 puntos) Γ es la circunferencia de centro (1, 0) y radio 2 recorrida en sentido positivo.

(b) (0,5 puntos) Γ es el arco de la parábola de ecuación y = (x − 1)2 − 4 recorrido desde (−1, 0) hasta
(3, 0).

(La preacta se publicará en intranet el 9 de julio a las 18 h.; la revisión personal –previamente solicitada– será el 13 de
julio a las 18:30 h.)

Respuesta:
2
Z un conjunto abierto D ⊂ R es conservativo en D
1.1 Se dice que un campo vectorial F continuo en
si para toda curva cerrada γ contenida en D es F = 0.
γ

1.2 Si el abierto D es conexo, la definición anterior equivale a la existencia de una función potencial
escalar de clase 1 en D, es decir ∃f : D → R, f ∈ C 1 (D), ∇f = F.

2.1 El campo F = (P, Q) es de clase 1, en ese caso, una condición necesaria para la existencia de
potencial escalar es ∂Q/∂x = ∂P/∂y, que en nuestro caso se cumple:
   
∂P ∂ x −2xy ∂ y ∂Q
= 2 2
= 2 2 2
= 2 2
=
∂y ∂y x + y (x + y ) ∂x x + y ∂x
Vamos a calcular un candidato a potencial integrando F a lo largo de una lı́nea poligonal que no pase
por el origen, sabemos que tal función será un potencial escalar de F en cualquier conjunto simplemente
conexo que contenga a la lı́nea poligonal –condición necesaria, pero no suficiente para que el campo sea
conservativo en D–. Dado (1, 0) (valdrı́a cualquier otro punto no nulo del plano) fijemos un (x, y), x > 0
y consideremos la poligonal de vértices (1, 0), (x, 0), (x, y):
Z x Z y Z x Z y
t t
f (x, y) = P (t, 0)dt + Q(x, t)dt = 2
dt + 2 2
dt =
1 0 1 t 0 x +t
1 1 1 p
log t|x1 + log(x2 + t2 )|y0 = log x + log(x2 + y 2 ) − log(x2 ) = log x2 + y 2 .
2 2 2
p
La función f (x, y) = log x2 + y 2 es continua en R2 \ {(0, 0)} y su gradiente es F, podemos concluir
que F es conservativo en todo su dominio de definición.
2.1 Alternativamente, podemos estudiar si el campo es conservativo en D aplicando la definición. La
condición ∂Q/∂x = ∂P/∂y nos permite afirmar que el campo es conservativo en dominios simplemente
conexos de D; equivalentemente, la circulación de F a lo largo de cualquier curva cerrada que no rodee
el origen es nula. Tomemos ahora una curva γ cerrada y simple que contenga el origen en su interior
geométrico. Conforme al Teorema de Green en dominios múltiplemente conexos del plano la integral a lo
largo de γ coincide con la circulación a lo largo de una circunferencia C de radio arbitrario R y centrada
en el origen siempre y cuando ambas curvas se recorran en el mismo sentido (horario o antihorario):
Z Z Z
1
F= F= 2 xdx + ydy = 0.
γ C R C

Que la última integral es nula puede comprobarse directamente mediante parametrización de la circun-
ferencia, o bien observando que el campo G = (x, y) cumple ∂y/∂x = ∂x/∂y = 0 en todo R2 por lo que
es conservativo. Lo cual permite concluir que el campo F es conservativo en D ya que su circulación a
lo largo de cualquier curva cerrada (encierre esta el origen o no) es nula.

2.2(a) La circunferencia Γ es una curva cerrada, contenida en el dominio de definición del campo con-
servativo F, por tanto la integral es nula.

2.2(b) Como el campo es conservativo la integral de lı́nea depende exclusivamente de los extremos de la
curva; dado que hemos calculado un potencial, lo más sencillo es utilizarlo:
Z Z
F = ∇f = f (3, 0) − f (−1, 0) = log 3.
Γ Γ

2.2(b) Alternativamente, podemos calcular la integral sustituyendo la curva que nos proponen por una
lı́nea poligonal contenida en D, obsérvese que el segmento que une los extremos de la curva no es
apto para la integración ya que atraviesa el origen, en el cual no está definido el campo. Tomemos,
por ejemplo, la poligonal P de vértices (−1, 0), (−1, 1), (3, 1), (3, 0). Entonces:
Z Z 1 Z 3 Z 1
F= Q(−1, t)dt + P (t, 1)dt − Q(3, t)dt
P 0 −1 0
Z 1 Z 3 Z 1
t t t
= 2
dt + 2
dt − 2
dt
0 1+t −1 1 + t 0 9+t
1 1 1
= log(1 + t2 )|10 + log(1 + t2 )|3−1 − log(9 + t2 )|10 = log 3.
2 2 2

Observación: El ejemplo que se muestra en este ejercicio es uno de los más sencillos de campo central
(véase el ejercicio 6.7); los campos centrales son conservativos en su dominio de definición: si el campo es
de la forma F(r) = g(krk)r, r ∈ D(F) ⊂ Rn con g continua, entonces un potencial escalar es de la forma
f (r) = h(krk) siendo h una primitiva de la función t 7→ tg(t). En el problema planteado, g(t) = t−2 por
lo que h es una primitiva de 1/t, por ejemplo h(t) = log t y un potencial escalar es f (r) = log(krk).

Gabriela Sansigre.
AMPLIACIÓN DE CÁLCULO (Curso 2009/2010) Convocatoria de junio 22.06.10

NOMBRE . . . . . . . . . . . . . . . . . . . . . . . . . . APELLIDOS. . . . . . . . . . . . . . . . . . . . . . . . . . . . . . . . . . . . . . . . . . . . .
Número de matrı́cula . . . . . . . . . . . . . . . . . . . . . . . . .

PROBLEMA 3 (3 puntos)
Sea F el campo vectorial definido por:
1 ³ 2 2 2
´
F(r) := xy , x + 2y + z , yz ,
krk3
donde r = (x, y, z).
(1) Determı́nense las funciones f y h, de clase C 1 en R, para las que el campo vectorial
1
G(r) := (f (z) , 0 , h(x))
krk
es un potencial vector de F, indicando el mayor conjunto de R3 en el que se cumple esta propiedad.
(0,75 Puntos.)
En R3 se considera la curva Γ definida por la intersección de la esfera de centro el√origen
√ y radio 1
y el plano y = x, orientada de forma que su vector tangente unitario en el punto (1/ 2, 1/ 2, 0) es el
vector k = (0, 0, 1).
Sea Σ la superficie cónica de vértice el punto (0, 1/2, 0) que se apoya en Γ y Σ+ la superficie Σ con
la orientación que se corresponde con la de su borde Γ.
(2) Sabiendo que Σ+ cumple las hipótesis del teorema de Stokes, utilı́cese razonadamente este teorema
para calcular el flujo del campo F a través de Σ+ . (1 Punto.)
(3) Utilı́cese razonadamente el teorema de Gauss para determinar una superficie orientada, S − , tal que
los flujos del campo F a través de Σ+ y S − coincidan y calcúlese este último flujo mediante una integral
doble. (1,25 Puntos.)
(La preacta se publicará en intranet el 9 de julio a las 18 h.; la revisión personal de este problema –previamente solicitada–
será el 15 de julio a las 18:30 h.)

Respuesta: (Se entregará esta hoja y, a lo sumo, una adicional.)


(1) Para determinar las funciones f y h se debe imponer la condición ∇ × G = F (el rotacional
de G es F) lo que significará, por definición, que el campo G es un potencial vector de F. Para ello,
representamos por (F1 , F2 , F3 ) y por (G1 , G2 , G3 ) los campos F y G, respectivamente. Entonces, basta
tener en cuenta que las tres igualdades:
 

 ∂G3 ∂G2 




− = F1 ⇐⇒ −h(x) = x , 



 ∂y ∂z 


 ∂G 

1 ∂G3 x h(x)−z f (z)+(x +y +z ) (f (z)−h (x))
2 2 2 0 0 2
x +2y +z2 2
∇ × G = F ⇐⇒  − = F2 ⇐⇒ 3 = 3 ,
 ∂z ∂x (x2 +y 2 +z 2 ) 2 (x2 +y 2 +z 2 ) 2 


 


 ∂G ∂G 


 2 1 

 − = F3 ⇐⇒ f (z) = z , 
∂x ∂y
se cumplen si y solamente si f (z) = z y h(x) = −x, que son las funciones f y h cuya expresión se pide
determinar en el enunciado. Puesto que estas dos funciones son de clase C ∞ en R, resulta que ∇ × G
y F están definidos en R3 \{(0, 0, 0)} que es, por tanto, el mayor conjunto de R3 en el que G es un
potencial vector de F.
(0,75 Puntos.)
(2) La superficie reglada Σ+ cumple las hipótesis que exige el teorema de Stokes. Además, siempre
existe un abierto de R3 \{(0, 0, 0)} que contiene a Σ+ y en el que el potencial vector G es de clase C 1 ;
por tanto se cumple:
ZZ ZZ Z
F · dσ ≡ ∇ × G · dσ = (Teorema de Stokes) = G · dr .
Σ+ Σ+ Γ

Ası́ pues, el teorema de Stokes permite calcular el flujo del campo F a través de Σ+ (superficie que no
es necesario (y no se pide) parametrizar) como la circulación de su potencial vector G a lo largo del
borde orientado Γ de esta superficie. Para ello, se dispone de las ecuaciones cartesianas de Γ:
x2 + y 2 + z 2 = 1 , y = x,
con lo que una de sus parametrizaciones, r(ϑ) = (x(ϑ), y(ϑ), z(ϑ)), viene dada por:
1
x(ϑ) = y(ϑ) = √ cos ϑ , z(ϑ) = sen ϑ , ϑ ∈ [0, 2π] ,
2
y à !
10 1
r (ϑ) = − √ sen ϑ , − √ sen ϑ , cos ϑ .
2 2
√ √
Nótese que el punto (1/ 2, 1/ 2, 0) se obtiene para ϑ = 0 y r0 (0) = (0, 0, 1) ≡ k; es decir, las ecuaciones
paramétricas consideradas constituyen, en efecto, una parametrización de la curva con la orientación
dada.
Entonces, utilizando la definición de circulación, se tiene:
ZZ Z Z 2π
0 1 Z 2π √
F · dσ = G · dr = G (r(ϑ)) · r (ϑ) dϑ = − √ dϑ = − 2 π ,
Σ+ Γ 0 2 0
que es el resultado pedido. (1 Punto.)

(3) Se trata de calcular el flujo de F a través de la superficie Σ+ que es de clase C 1 a trozos pero
no es cerrada. Para poder calcularlo aplicando el teorema de Gauss procedemos a “cerrar” Σ utilizando
otra superficie, S, con el mismo borde y tal que la divergencia de F esté definida en el recinto acotado
S
Ω ∈ R3 limitado por la unión, Σ S, de ambas superficies.
Dado que F está definido y es C 1 en R3 \{(0, 0, 0)}, la superficie S (C 1 a trozos y con borde Γ) se
puede elegir de cualquier forma con tal de que Ω no contenga el origen. En tal caso, el teorema de Gauss
permite escribir: ZZ ZZZ
S F · dσ = ∇ · F dV = 0,
(Σ S)+ Ω
S
donde (Σ S)+ representa la cara exterior de la frontera de Ω, que definirá la orientación de las dos
superficies que la forman. Además, la integral es cero porque el campo F es solenoidal (∇ · F = 0) en
Ω, ya que en ese conjunto F es un campo de rotores (deriva de un potencial vector).
A la hora de hacer una √elección concreta para S, tenemos en cuenta la forma del campo. Este
contiene la expresión krk = x2 + y 2 + z 2 , lo que sugiere, como elección adecuada para S, el casquete
de la esfera unidad que tiene por borde la curva Γ y está situado en el semiespacio y > x, de forma que
Ω no contenga el origen. Teniendo en cuenta la orientación de Σ dada por Σ+ y denotando por S + la
cara de S orientada según la normal exterior a la esfera, se tiene:
ZZ ZZ ZZ ZZ ZZ ZZ
S F · dσ = F · dσ + F · dσ = 0 =⇒ F · dσ = − F · dσ = F · dσ ,
(Σ S)+ Σ+ S+ Σ+ S+ S−

donde el último flujo es el que se pide calcular mediante una integral doble y, obviamente, S − representa
la cara interior del casquete esférico mencionado, que es una de las posibles superficies a las que se refiere
el enunciado.
(0,5 Puntos.)
Para calcular el flujo conviene tener en cuenta que krk = 1 sobre la superficie S − (que es lo que ha
motivado la elección de S − ) y que el vector unitario en la dirección de su normal es −r/krk. Por tanto:
ZZ ZZ ZZ ZZ
r 2y
F · dσ = − F· dσ = − dσ = − 2y dσ .
S− S− krk S − krk S−

Utilizando coordenadas esféricas, una parametrización de S − , r(φ, θ) = (x(φ, θ), y(φ, θ), z(φ, θ)), viene
dada por las ecuaciones paramétricas:
· ¸
π π
x(φ, θ) = cos θ sen φ , y(φ, θ) = sen θ sen φ , z(φ, θ) = cos φ , θ∈ , + π , φ ∈ [0, π],
4 4
con lo que la norma del vector normal asociado a esta parametrización es
° °
° ∂r(φ, θ) ∂r(φ, θ) °°
°
° × ° = sen φ .
° ∂θ ∂φ °

Luego, finalmente y por definición de integral de superficie:


ZZ ZZ ZZ ° °
° ∂r(φ, θ) ∂r(φ, θ) °°
°
Fdσ = − 2y dσ = −2 y(φ, θ) °° × ° dθ dφ
S− S− [ π , π +π]×[0,π]
4 4
∂θ ∂φ °
Z π/4+π Z π √
= −2 sen θ sen2 φ dθ dφ = − 2 π ,
π/4 0

que es el resultado final pedido.


(0,75 Puntos.)
AMPLIACIÓN DE CÁLCULO (Curso 2009/2010) Convocatoria de julio 22.07.10

NOMBRE . . . . . . . . . . . . . . . . . . . . . . . . . APELLIDOS . . . . . . . . . . . . . . . . . . . . . . . . . . . . . . . . . . . . . . . . . . .
Número de matrı́cula. . . . . . . . . . . . . . . . . . . . . . . .

PROBLEMA 1 (4 puntos)
Este problema proporciona un método sencillo para calcular la conocida integral de probabilidades
Z ∞ 2
I := e−x dx .
0

Se pide:
(1) Siendo n ∈ N, estudiar la convergencia uniforme de la integral impropia
Z ∞ 2 (y 2 +1)
F (y) := xe−x dx
0

para y ∈ [0, n]. En caso de que F (y) exista, calcular su valor. (0,75 puntos)
(2) Discutir la existencia de la integral
Z n
Hn := F (y) dy
0

y calcular su valor, en caso de que la integral exista. (0,5 puntos)


(3) Estudiar la convergencia de la integral doble impropia
ZZ
2 (y 2 +1)
J := xe−x dxdy
D

siendo D := [0, ∞) × [0, ∞). Se sugiere para ello considerar, para cada n ∈ N, la integral doble sobre
[0, n] × [0, n] y analizar su comportamiento cuando n crece.
En caso de que J exista, calcular su valor. (1,25 punto)
(4) Estudiar la convergencia de la integral impropia
Z ∞ 2 (y 2 +1)
G(x) := xe−x dy
0

para cada x ≥ 0. Calcular su valor para aquellos x para los que sea convergente. Se sugiere dar este
valor en función de I. (0,75 puntos)
(5) Estudiar la convergencia de la integral impropia
Z ∞
G(x) dx
0

y, en caso de que exista, calcular su valor. √ (0,25 puntos)


(6) Usando los resultados obtenidos hasta ahora, demostrar que I = π/2. (0,5 puntos)
Nota. El procedimiento expuesto en este problema está tomado de H. Iwasawa, Gaussian integral puzzle, Math.
Intelligencer 31 (2009), 38–41.

Respuesta: Se entregará esta hoja y, a lo sumo, una adicional.


(1) Para todo y ∈ [0, n] se tiene:
2 (y 2 +1) 2
0 ≤ xe−x ≤ xe−x .
Esta última función tiene integral convergente, con valor:
∞ ∞
1 1
Z 
−x2 2
xe dx = − e−x = .
0 2 0 2
El criterio de Weierstrass implica entonces que F (y) es uniformemente convergente para y ∈ [0, n].
Su valor es Z ∞ " #x=∞
−x2 (y 2 +1) 1 −x2 (y 2 +1) 1 1
F (y) = xe dx = − 2 e = .
0 2(y + 1) x=0
2 y2 + 1
(2) Como F (y) es una función continua en [0, n], es integrable de Riemann en ese intervalo y su
integral es: Z n
1 Z n dy 1 n 1
Hn = F (y) dy = = [arc tg y]0 = arc tg n .
0 2 0 y2 + 1 2 2
(3) Siendo Dn := [0, n] × [0, n], se verifica:
ZZ
2 (y 2 +1)
Jn : = xe−x dxdy
Dn
Z n Z n 2 (y 2 +1)
= dy xe−x dx
Z0 n Z0 ∞
2 (y 2 +1)
≤ dy xe−x dx (por ser el integrando ≥ 0)
Z 0n 0

= F (y) dy
0
1 π
= arc tg n ≤ .
2 4
Claramente, Jn crece cuando n crece y, como la sucesión (Jn ) está acotada, existe
π
lı́m Jn ≤ .
n→∞ 4
Por tanto, usando de Rnuevo la positividad del integrando, concluimos que J es convergente. Como
las integrales J, F (y) y 0∞ F (y) dy son convergentes, la integral doble es igual a la integral iterada,
luego:
Z ∞ Z ∞ 2 (y 2 +1)
Z ∞ Z n 1 π
J= dy xe−x dx = F (y) dy = n→∞
lı́m F (y) dy = lı́m arc tg n = .
0 0 0 0 2 n→∞ 4
(4) Obviamente, G(0) = 0. Para cada x > 0 fijo, se tiene:
Z ∞ 2 y2 2 2
Z ∞ 2 y2
G(x) = xe−x e−x dy = xe−x e−x dy .
0 0

Se sabe que esta última integral es convergente porque no es más que una sencilla modificación de
la integral I de probabilidades.
Haciendo el cambio y 7→ t dado por t := xy, se tiene
Z ∞ 2 y2
Z ∞ 2 dt 1 Z ∞ −t2 I
e−x dy = e−t = e dt = ,
0 0 x x 0 x
de donde
2
G(x) = e−x I .
(5) Se tiene Z ∞ Z ∞ 2
G(x) dx = I e−x dx = I 2 .
0 0
(6) Como las integrales G(x), 0∞ G(x) dx y J son convergentes, se verifica la igualdad de la
R

integral doble con la integral iterada, luego:


Z ∞ Z ∞ 2 (y 2 +1)
Z ∞
J= dx xe−x dy = G(x) dx = I 2 ,
0 0 0

de donde √
√ π
I= J= .
2
AMPLIACIÓN DE CÁLCULO (Curso 2009/2010) Convocatoria de julio 22.07.10

PROBLEMA 2 (3 puntos)
 q 
3 2 2
Sea Ω = (x, y, z) ∈ R / ( x2 + y2 − 1) + z ≤ 1 y Σ su frontera.

1. (1,5 puntos) Calcular el volumen de Ω.

2. (1,5 puntos) Calcular el área de Σ.

Respuesta: vamos a resolver el ejercicio de varias formas. En primer lugar contemplando la superficie
y el sólido como área y volumen de revolución y aplicando el Teorema de Guldin; en segundo lugar
haremos la integral de volumen mediante un cambio de coordenadas que sugerirá una representación
paramétrica de la superficie y, por último, contemplando el sólido como conjunto simple, proyectable
sobre el plano de ecuación z = 0 y la superficie con la parametrización natural obtenida a partir de
la ecuación implı́cita.


Primer método: Obsérvese que la superficie Σ de ecuación implı́cita ( x2 + y 2 − 1)2 + z 2 = 1 es el
toro que se obtiene al girar la circunferencia (x − 1)2 + z 2 = 1, y = 0 alrededor del eje z; a su vez Ω
es el toro sólido que se obtiene al girar el disco (x − 1)2 + z 2 ≤ 1, y = 0. En efecto, comprobémoslo.
Una parametrización de la circunferencia es x = 1 + cos α, y = 0, z = sen α, α ∈ [0, 2π], giremos un
punto genérico alrededor del eje z:
      
x cos θ − sen θ 0 1 + cos α cos θ(1 + cos α)
 y  =  cos θ sen θ 0  0  =  sen θ(1 + cos α) 
      

z 0 0 1 sen α sen α

eliminando los ángulos α y θ se obtiene la ecuación implı́cita de Σ: ( x2 + y 2 − 1)2 + z 2 = 1.
Para calcular volumen y área podemos aplicar el teorema de Guldin en sus dos formas:

Teorema de Guldin

Para volúmenes: El volumen del sólido engendrado por un recinto plano que gira alrededor
de un eje no secante y situado en su mismo plano es igual al producto del área del recinto por
la longitud de la circunferencia descrita por su centroide.

Para superficies: El área de la superficie engendrada por una curva plana que gira alrededor
de un eje no secante y situado en el mismo plano es igual al producto de la longitud de la curva
por la longitud de la circunferencia descrita por su centroide.

En nuestro caso, el centroide tanto del disco como de la circunferencia es su centro que describe
una circunferencia de longitud 2π. El área del disco es π y su perı́metro 2π ası́ pues V (Ω) = 2π 2 y
A(Σ) = 4π 2 .

Segundo método:
1. Expresamos Ω en coordenadas cilı́ndricas, x = ρ cos θ,
n y = ρ sen θ, z = λ. El jacobiano de la trans- o
formación es J3 = ρ ≥ 0 y la contraimagen de Ω es A = (ρ, θ, λ) ∈ R3 / (ρ − 1)2 + λ2 ≤ 1, 0 ≤ θ ≤ 2π ,
por tanto el volumen pedido es:
ZZZ ZZZ ZZ
V (Ω) = dxdydz = ρdρdθdλ = 2π ρdρdλ.
Ω A (ρ−1)2 +λ2 ≤1
Para resolver esta última integral hacemos el cambio de coordenadas ρ = 1 + r cos α, λ = r sen α con
(r, α) ∈ [0, 1] × [0, 2π] y el jacobiano es J2 = r, entonces
ZZ
V (Ω) = 2π (1 + r cos α)rdrdα = 2π 2 .
[0,1]×[0,2π]


2. La superficie Σ viene representada por la ecuación implı́cita ( x2 + y 2 −1)2 +z 2 = 1. Aprovechando
los cambios de coordenadas que hemos realizado en el apartado previo, podemos parametrizar:

x = (1 + cos α) cos θ, x = (1 + cos α) sen θ, z = sen α, (α, θ) ∈ [0, 2π] × [0, 2π].

El área buscada es

ZZ ∂(x, y, z) ∂(x, y, z)
A(Σ) = × dαdθ


[0,2π]×[0,2π] ∂α ∂θ

Calculemos la norma del vector normal a Σ:


∂(x, y, z)
= (− sen α cos θ, − sen α sen θ, cos α)
∂α
∂(x, y, z)
= (−(1 + cos α) sen θ, (1 + cos α) cos θ, 0)
∂θ
Estos dos vectores son ortogonales por lo que la norma de su producto vectorial es el producto de
las normas:

∂(x, y, z) ∂(x, y, z)

× = 1 + cos α.
∂α ∂θ

Por último el área es


ZZ
A(Σ) = (1 + cos α)dαdθ = 4π 2 .
[0,2π]×[0,2π]

Tercer método:
1. Expresamos Ω en coordenadas esféricas, x = ρ sen φ cos θ, y = ρ sen φ sen θ, z = ρ cos φ. El jaco-
biano es J = ρ2 sen φ y la contraimagen de Ω es:
n o
A = (ρ, θ, φ) ∈ R3 / 0 ≤ θ ≤ 2π, 0 ≤ φ ≤ π, 0 ≤ ρ ≤ 2 sen φ .

Entonces el volumen se calcula:


ZZZ ZZZ
V (Ω) = dxdydz = ρ2 sen φdρdθdφ.
Ω A

Integramos en primer lugar en la variable ρ,


Z 2 sen φ 8
ρ2 dρ = sen3 φ
0 3
y sustituimos

8Z π 4 16π Z π/2 16π 16π 3/2 1/2 Γ(1/2)2


V (Ω) = 2π sen φ dφ = 2 sen4 φ dφ = β(5/2, 1/2) = = 2π 2 .
3 0 3 0 3 3 2
2. También puede calcularse le área utilizando coordenadas esféricas. La ecuación de Σ es ρ = 2 sen φ,
lo que da lugar a la parametrización: x = 2 sen2 φ cos θ, y = 2 sen2 φ sen θ, z = 2 sen φ cos φ, (φ, θ) ∈
[0, π] × [0, 2π]. La norma del vector normal asociado (complétense los cálculos) es N = 4 sen2 φ y el
área:
ZZ Z π
A(Σ) = 4 sen2 φ dφ dθ = 8π sen2 φ dφ = 4π 2 .
[0,π]×[0,2π] 0

Cuarto método:
1. Alternativamente, puede calcularse el volumen expresando Ω como sólido proyectable sobre el
plano de ecuación z = 0; sea D = {(x, y) ∈ R2 / x2 + y 2 ≤ 4}:
 q 
3 2 2
Ω = (x, y, z) ∈ R / (x, y) ∈ D, z ≤ 1 − ( x2 + y2 − 1)

entonces:
ZZZ ZZ r q
V (Ω) = dxdydz = 2 1 − ( x2 + y 2 − 1)2 dxdy.
Ω D

Para calcular esta integral doble pasamos a coordenadas polares:


ZZ q Z 2 q
V (Ω) = 2 ρ 1 − (ρ − 1)2 dρdθ = 4π ρ 1 − (ρ − 1)2 dρ,
[0,2]×[0,2π] 0

y, por último, en la integral simple hacemos el cambio ρ = 1 + cos α, α ∈ [0, π] y se obtiene:


Z π
V (Ω) = 4π sen2 α(1 + cos α)dα = 2π 2 .
0

2. La superficie Σ viene representada por la ecuación implı́cita


q
G(x, y, z) := ( x2 + y 2 − 1)2 + z 2 − 1 = 0;

la función G cumple G(x, y, z) = G(±x, ±y, ±z) por tanto A(Σ) = 8A(Σ+ ) donde hemos denotado
por Σ+ la parte de la superficie que yace en el primer octante. Si G ∈ C 1 y ∂G/∂z 6= 0 puede
despejarse z implı́citamente como función de (x, y). La parametrización en este caso es Φ(x, y) =
(x, y, z(x, y)) con (x, y) ∈ {x2 + y 2 ≤ 4, x ≥ 0, y ≥ 0}. El vector normal asociado es
∂Φ ∂Φ 1
× = ∇G
∂x ∂y ∂G/∂z
y el área:
ZZ
k∇Gk
A(Σ+ ) = dxdy.
D |∂G/∂z|
Hagamos los cálculos:
!
q x q y
∇G(x, y, z) = 2( x2 + y2 − 1) √ 2 2
, 2( x2 + y 2 − 1) √ 2 , 2z ,
x +y x + y2

k∇Gk 1
= .
|∂G/∂z| |z|

+
ZZ
1
A(Σ ) = q √ 2 dxdy.
D 1 − ( x + y 2 − 1)2
Cambiamos a coordenadas polares:
ZZ
ρ πZ2 ρ
A(Σ+ ) = q dρdθ = q dρ,
[0,2]×[0,π/2] 1 − (ρ − 1)2 2 0 1 − (ρ − 1)2

y en esta última integral hacemos el cambio ρ = 1 + cos α, α ∈ [0, π], con lo que se llega a

πZπ π2
A(Σ+ ) = (1 + cos α)dα = ,
2 0 2
y se concluye que el área pedida es A(Σ) = 8A(Σ+ ) = 4π 2 .

Observación: Este método, utilizando la ecuación implı́cita, tiene algunos pasos delicados que no
he detallado. La parametrización vale en el interior de D, y ahı́ el vector normal está perfectamente
definido. La derivada parcial de G respecto de z se anula para z = 0; es decir, el vector normal
se anula en parte de la frontera de D. La integral doble que representa el área es impropia pero
convergente y los cálculos realizados pueden justificarse con todo rigor.

El volumen utilizando el principio de Cavalieri: √


Para cada z fijo, la sección correspondiente es Dz = {(x, y) / ( x2 + y 2 − 1)2 ≤ 1 − z 2 }, como z
varı́a entre −1 y 1, el volumen es
Z 1
V (Ω) = A(z)dz,
−1

siendo A(z) el área de Dz . Si expresamos Dz como:


√ q √
1− 1 − z2 ≤ x2 + y 2 ≤ 1 + 1 − z2
√ √
observamos que es una corona circular, luego su área es A(z) = π((1 + 1 − z 2 )2 − (1 − 1 − z 2 )2 ) =

4π 1 − z 2 . Para finalizar (haciendo, por ejemplo, el cambio z = sen t) se llega a:
Z 1 √ Z π/2
V (Ω) = 4π 1 − z 2 dz = 4π cos2 t dt = 2π 2 .
−1 −π/2

Gabriela Sansigre.
AMPLIACIÓN DE CÁLCULO (Curso 2009/2010) Convocatoria de julio 22.07.10

PROBLEMA 3 (3 puntos)
Se consideran los siguientes campos vectoriales, definidos para todo vector no nulo r = (x, y, z),
1 (a · r)
F (r) = (0, −z, y) G (r) = r
x2 + y2 + z2 krk2
siendo a un vector fijo del espacio. Se pide:

1. Determinar el vector a para el cual G es potencial vector de F en un dominio de R3 que se


especificará. (1 punto)
2. Demostrar que el flujo de F a través de cualquier porción de una superficie esférica centrada
en el origen es nulo. (0’5 puntos)
3. Se considera la pirámide de vértice (2, 1/2, 1) y de base el triángulo de vértices (1, 0, 0) , (3, 0, 0) ,
(3, 2, 0) . Sea Σ la superficie de dicha pirámide formada por las caras que no son base, orientadas
de forma que su normal tenga tercera componente positiva. Calcular el flujo de F a través de
Σ. (1’5 puntos)

Respuesta: (Se entregará esta hoja, y a lo sumo una adicional.)

1. Imponemos que F = rot (G) . Usando la propiedad rot (f V) = ∇f × V + f rot (V) , escribimos
1
G (r) = (a · r) V (r) con V (r) = krk 2 r (campo central). Ası́ pues,

1
rot (G (r)) = a × V (r) + (a · r) rot (V (r)) = (a × r)
krk2
donde se ha utilizado que rot (V (r)) = 0 ya que todo campo central V (r) es irrotacional.
Finalmente, si a = (a1 , a2 , a3 ) ,
1
rot (G (r)) = (a2 z − a3 y, a3 x − a1 z, a1 y − a2 x)
x2 + y2 + z2
y se observa que, para a = (1, 0, 0) , rot (G (r)) coincide con F (r) en el dominio R3 \ {(0, 0, 0)}.
2. Si S es una porción de una superficie esférica de centro el origen (es decir, que krk = R), se
observa que el vector n normal a S es proporcional a r = (x, y, z) , con lo cual el campo F es
ortogonal a dicha normal, puesto que
1
F (r) · r = (0, −z, y) · (x, y, z) = 0.
x2 + y 2 + z 2
Ası́ pues, el flujo de F que atraviesa S es nulo, ya que n = r/R y
Z Z Z
Fds = (F · n) ds= 0 ds = 0.
S S S

1
(Otra forma de verlo es que F (r) = krk2
(1, 0, 0) ×r es un campo ortogonal a r).

3. En vez de parametrizar las tres caras planas de la superficie Σ, utilizando el hecho que F es
adivergente en R3 \ {(0, 0, 0)}, podemos tanto aplicar el Teorema de Gauss (pasando a calcular
el flujo en otra superficie que tenga el mismo borde) como aplicar el Teorema de Stokes (pues
conocemos un potencial vector G).
• MÉTODO 1 (Teorema de Gauss): Cerramos Σ con el triángulo D de vértices (1, 0, 0) ,
(3, 0, 0) , (3, 2, 0) , que está contenido en el plano z = 0. Esto es válido pues en el espacio
delimitado entre Σ y D no contiene al origen, luego en ese espacio el campo F es de clase
C 1 , y al ser adivergente, se tiene por el Teorema de Gauss que
Z Z
Fds = Fds
Σ D

siempre que la orientación de Σ y D sea coherente, con lo cual basta que el vector normal
al triángulo plano D sea (0, 0, 1) . Por tanto,
Z Z
1
Fds = (0, −z, y) · (0, 0, 1) dxdy =
D x + y2 + z2
2
ZD
y
= 2 2
dxdy.
D x +y

El triángulo D puede parametrizarse como D = {(x, y, 0) : 1 ≤ x ≤ 3, 0 ≤ y ≤ x − 1}.


Por integrales iteradas se llega a que el valor de la integral es
Z 3 Z x−1 Z 3 
y 1 2 2
dydx = log x + (x − 1) − log (x) dx
1 0 x2 + y 2 1 2
5 1 1
= log 13 − 3 log 3 − π + arctg 5.
4 8 2
• MÉTODO 2 (Teorema de Stokes): por el primer apartado, disponemos de un potencial
vector G que es de clase C 1 (Σ) , luego por el Teorema de Stokes
Z Z Z
x
Fds = Gdl = 2 2 2
(x, y, z) dl
Σ C C x +y +z

donde C es la curva borde de Σ con la orientación adecuada, a saber: C es la curva borde


del triángulo de vértices (1, 0, 0) , (3, 0, 0) , (3, 2, 0) , orientada positivamente en el plano
z = 0. Hay que parametrizar cada uno de los tres lados de C :
En C1 , el lado que va de (1, 0, 0) a (3, 0, 0) , parametrizado por (t, 0, 0) , t ∈ [1, 3], el vector
derivada es (1, 0, 0) : Z Z 3
x
2 2
(x, y, 0) dl = 1dt = 2.
C1 x + y 1
En C2 , el lado que va de (3, 0, 0) a (3, 2, 0) , parametrizado por (3, t, 0) , t ∈ [0, 2], el vector
derivada es (0, 1, 0) :
Z Z 2
x 3t 3
2 2
(x, y, 0) dl = 2
dt = log 13 − 3 log 3.
C2 x + y 0 9+t 2
En C3 , el lado que va de (1, 0, 0) a (3, 2, 0) , parametrizado por (t, t − 1, 0) , t ∈ [1, 3], la
derivada es (1, 1, 0) (obsérvese que hemos considerado la orientación contraria):
Z 3 Z 3
t (2t − 1) 2t2 − t
Z
x
2 2
(x, y, 0) dl = 2 dt = 2
dt =
C3 x + y 1 2t − 2t + 1
2
1 t + (t − 1)
Z 3
t−1 1 1 1
= 1+ 2 dt = 2 + π + ln 13 − arctg 5
1 2t − 2t + 1 8 4 2
Finalmente, la solución es la suma de las dos primeras integrales curvilı́neas, menos la
tercera: Z Z
5 1 1
Fds = Gdl = log 13 − 3 log 3 − π + arctg 5.
Σ C 2 8 2

You might also like